PDA

View Full Version : Got a Real-World Weapon, Armor or Tactics Question? Mk. XVIII



Pages : [1] 2 3 4 5 6

Brother Oni
2015-06-15, 01:54 AM
Real World Weapon, Armour and Tactics Thread XVIII

This thread is a resource for getting information about real life weapons, armour and tactics. The concept has always been that the information is for RPG players and DMs so they can use it to make their games better, thus it's here rather than in Friendly Banter.

A few rules for this thread:


This thread is for asking questions about how weapons, armour and tactics really work. As such, it's not going to include game rule statistics. If you have such a question, especially if it stems from an answer or question in this thread, feel free to start a new thread and include a link back to here. If you do ask a rule question here, you'll be asked to move it elsewhere, and then we'll be happy to help out with it.

Any weapon or time period is open for questions. Medieval and ancient warfare questions seem to predominate, but since there are many games set in other periods as well, feel free to ask about any weapon. This includes futuristic ones - but be aware that these will be likely assessed according to their real life feasibility. Thus, phasers, for example, will be talked about in real-world science and physics terms rather than the Star Trek canon. If you want to discuss a fictional weapon from a particular source according to the canonical explanation, please start a new thread for it.

Please try to cite your claims if possible. If you know of a citation for a particular piece of information, please include it. However, everyone should be aware that sometimes even the experts don't agree, so it's quite possible to have two conflicting answers to the same question. This isn't a problem; the asker of the question can examine the information and decide which side to go with. The purpose of the thread is to provide as much information as possible. Debates are fine, but be sure to keep it a friendly debate (even if the experts can't!).

No modern real-world political discussion. As the great Carl von Clausevitz once said, "War is merely the continuation of policy by other means," so politics and war are heavily intertwined. However, politics are a big hot-button issue and one banned on these boards, so avoid political analysis if at all possible (this thread is primarily about military hardware). There's more leeway on this for anything prior to about 1800, but be very careful with all of it, and anything past 1900 is surely not open for analysis (These are arbitrary dates but any dates would be, and these are felt to be reasonable).

No graphic descriptions. War is violent, dirty, and horrific, and anyone discussing it should be keenly aware of that. However, on this board graphic descriptions of violence (or sexuality) are not allowed, so please avoid them.


With that done, have at and enjoy yourselves!
Thread I (http://www.giantitp.com/forums/showthread.php?24294-Got-A-Weapon-or-Armor-Question)
Thread III (http://www.giantitp.com/forums/showthread.php?21318-Got-a-Real-World-Weapon-or-Armor-Question-III)
Thread IV (http://www.giantitp.com/forums/showthread.php?18302-Got-a-Real-World-Weapon-or-Armor-Question-Mk-IV)
Thread V (http://www.giantitp.com/forums/showthread.php?80863-Got-a-Real-World-Weapon-or-Armor-Question-Mk-V)
Thread VI (http://www.giantitp.com/forums/showthread.php?124683-Got-a-Real-World-Weapon-or-Armor-Question-Mk-VI)
Thread VII (http://www.giantitp.com/forums/showthread.php?168432-Got-a-Real-World-Weapon-or-Armor-Question-Mk-VII)
Thread VIII (http://www.giantitp.com/forums/showthread.php?192911-Got-a-Real-World-Weapon-or-Armor-Question-Mk-VIII)
Thread IX (http://www.giantitp.com/forums/showthread.php?217159-Got-a-Real-World-Weapon-or-Armor-Question-Mk-IX)
Thread X (http://www.giantitp.com/forums/showthread.php?238042-Got-a-Real-World-Weapons-or-Armour-Question-Mk-X)
Thread XI (http://www.giantitp.com/forums/showthread.php?255453-Got-a-Real-World-Weapons-or-Armour-Question-Mk-XI)
Thread XII (http://www.giantitp.com/forums/showthread.php?282471-Got-a-Real-World-Weapons-or-Armour-Question-Mk-XII)
Thread XIII (http://www.giantitp.com/forums/showthread.php?308462-Got-a-Real-World-Weapons-or-Armour-Question-Mk-XIII)
Thread XIV (http://www.giantitp.com/forums/showthread.php?327994-Got-a-Real-World-Weapon-or-Armor-Question-Mk-XIV)
Thread XV (http://www.giantitp.com/forums/showthread.php?347806-Got-a-Real-World-Weapon-or-Armour-Question-Mk-XV)
Thread XVI (http://www.giantitp.com/forums/showthread.php?371623-Got-a-Real-World-Weapon-Armor-or-Tactics-Question-Mk-XVI)
Thread XVII (http://www.giantitp.com/forums/showthread.php?392804-Got-a-Real-World-Weapon-Armor-or-Tactics-Question-Mk-XVII)

Mr. Mask
2015-06-15, 02:28 AM
Carl: I think horse can kneel/lie down quickly enough with practice. I'm also assuming remarkable flexibility where the human body connects with the horse body, so most tasks humans perform should be possible to centaurs.

Crawling, however, seems like it would be difficult for them. The changes in structure you suggest sound interesting, and I was wondering about such changes myself. Would those changes make a centaur very different from a horse? That is, would it look like a horse, and still have abilities similar to that of a horse, with the suggested changes?

goto124
2015-06-15, 02:32 AM
(Quotes from previous thread)


I'm currently playing a combat-trained person from a fictional medieval place. Yes, I know 'medieval' covers a wide range of ages, no I don't know which period of medieval I'm actually working with. For now assume a generic fantasy medieval europe setting. Except since my char is on the poorer side, she has most experience with bronze and some experience with iron. She mainly uses cheaply-made swords, shields, clubs, maces, and a few other weapons. I don't know how (un)realistic bronze armor and weapons are IRL, but in her (sorta fantasy) setting it's commonplace, so that's one thing we're sticking to.

Now, she's possessed (taken over the body of) someone (we'll call this person a guard) who's essentially wearing the using whatever a S.W.A.T. team member would use. How would the outfit feel like to one who's worn bronze and a bit of iron armor? How would she attempt to hold and use the gun, in this case a Uzi (because that's what the GM ruled)? Will she point the muzzle at her own body?

She's surrounded by guards, so she's trying to act normal, like everyone else. She could look at the other guards to figure out how to hold the gun. Is shooting even something reasonable for her?


If they're from a medieval setting... they've never worn bronze, that stuff was largely phased out before Jesus was born. As for a flak vest compared to a medieval breastplate. The flak vest is more comfortable (kevlar may be stiff but it's still not as stiff as steel or iron), and weighs less... Actually it'd feel a lot like wearing a brigantine with a helmet (which is more Renaissance era, but whatever).

It'd probably be relatively easy to adjust too... unless they're used to full-plate armor or chainmail like a knight would use. Then it might feel a bit vulnerable to them.

You're right- I suppose the setting my char is from is very different from actual medieval ages. She rarely wears full-plate and chainmail, if ever (too expensive and beyond her reach). Interesting info!


Honestly she shouldn;t have a clue what the weapons for. She's no reason to know anything about it. And just what the hell are SWAT doing in a bronze age setting?

Some wierd stuff happened. Time and dimensional travel was involved. AKA the SWAT are not in a bronze age setting, it's a person ripped out of a bronze age setting and thrown into a modern world. She still hasn't figured out how lifts work.


If she's reasonably intelligent, then she's likely to pick up on how they work (point and pull) in terms familiar to her (a metal sling that throws bullets by pulling this lever), or they may be already be familiar with hand cannons (https://en.wikipedia.org/wiki/Hand_cannon) and recognise the basic principles already. The fact that it works via a trigger should be familiar from crossbows, although modern weapon safety (yes, they probably would look down the barrel of a loaded weapon when it doesn't fire) and trigger discipline wouldn't be, neither would be being prepared for the recoil.

Reloading, clearing blockages and stoppages and how best to hold the weapon would be beyond them though. The fire selector/safety switch is also something they're unlikely to figure out, not to mention the uzi has a second grip safety in case the weapon is dropped or the user loses a firm grip on the weapon.

In addition, a lack of familiarity with modern combat, both in terms of seeking cover and range ("don't worry, we're well out of bow shot..." *Headshot*) is going to be an issue, although they would pick it up quite quickly if they survive their first firefight.

I'm fairly sure she hasn't so much as seen a sling, hand cannon, or a crossbow, due to the homebrew setting she's from. Even longbows are rare there somehow.

She won't figure out the trigger for sure. Though I think she won't point the muzzle at herself. Will she fire the thing at all, even by accident?

How will she hold the Uzi? Even I, someone with little knowledge of guns, knows that shooting involves pulling the trigger. She's also used to swords and clubs stuff. I think she'll try to copy what her 'friends' are doing with their guns, but get some details off.

Chances are the only way she can use this Uzi is to smack people upside the head with it like a club :smallbiggrin: How good/bad of an idea is this? Probably the best given the situation, but still curious.

Anyway, thanks for everyone's help! Keep it going!

Brother Oni
2015-06-15, 03:01 AM
I'm fairly sure she hasn't so much as seen a sling, hand cannon, or a crossbow, due to the homebrew setting she's from. Even longbows are rare there somehow.

She won't figure out the trigger for sure. Though I think she won't point the muzzle at herself. Will she fire the thing at all, even by accident?

How will she hold the Uzi? Even I, someone with little knowledge of guns, knows that shooting involves pulling the trigger. She's also used to swords and clubs stuff. I think she'll try to copy what her 'friends' are doing with their guns, but get some details off.

Chances are the only way she can use this Uzi is to smack people upside the head with it like a club :smallbiggrin: How good/bad of an idea is this? Probably the best given the situation, but still curious.

It would have to be a pretty odd homebrew setting, given that slings date back to the Stone Age. I'm also curious how full plate developed without gunpowder weapons to spur its development.

It's highly unlikely that she will know how to use the weapon at all - you say you have a little knowledge of guns, but your character has no idea of guns or even the underlying technology, if even triggers are beyond her (it's a very subtle movement and it's easy to miss before the earth shattering kaboom as it fires), let alone get past the two external safeties that an uzi has.
While she would pick up how to hold the weapon from the guards around her, she will end up looking down the muzzle ("Maybe's my one is blocked?") at some point, if not outright cower in terror when she hears one fire for the first time (listening to recorded gunfire is no comparison to the real thing, since microphones often can't record the supersonic crack).

Clubbing someone with an uzi is a bad idea, but the grip safety will stop it from slamfiring (https://en.wikipedia.org/wiki/Slamfire), so it's not as bad as it could be. This is assuming that the SWAT force doesn't disable their grip safeties of course - there's a story of the Secret Service taping down their grip safeties; one agent accidentally dropped his weapon and it went off. On Airforce One. Ooops.

That said, hitting somebody hard enough repeatedly with an uzi would probably break the weapon.

goto124
2015-06-15, 03:21 AM
Fascinating! It's great fun discussing these sort of things. My char merely fails to encounter a sling, and full plate (however it's defined) may or may not exist in her world. But yea, the setting is a bit odd, and follows the tradition of not having any gunpowder.

Not sure if it's related to the topic, but what swear words and phrases would a bronze-age person use? The f-word seems too modern here...

Mr. Mask
2015-06-15, 03:43 AM
Well, they wouldn't be speaking in modern English (or French, or German, or Arabic) to begin with. If they magically possess a knowledge of the language, either their whole accent and vocabulary will change with this, or they'll have some more literal translation of given words and phrases from their original language. A lot of swearing was religion based, but what kind of curses you'd use depends heavily on the region, language and culture. Interestingly, I haven't seen a grammatical equivalent to several popular forms of modern swearing, in records from the past (I may not have looked hard enough).

Carl
2015-06-15, 06:01 AM
@Goto124: You literally cannot, and i mean cannot make bows and slings rare in any setting without some pretty massive changes of another kind. They weren't developed as weapons of war, they were your primary food gathering tool, a vital part of basic survival for literally millennia.

@Mr Mask: Hard to say, i'd argue their would be some loss of top speed with such changes, but handle rough terrain better but it probably wouldn't be any less of an equine breed than a Gorillia is a breed of Ape same as Human's. if you want an equally odd example note that the Okapi (https://en.wikipedia.org/wiki/Okapi) is of the same Genus as the Giraffe.

Also such changes would be in line with a six legged animal eventually developing the front third into a distinct portion with tool handling capabilities, (I.E hands), though that might require something other than traditional hoeves to develop, which would be a large change but subject to explaining that it's plausible from an evolutionary standpoint subject to the right circumstances.

Brother Oni
2015-06-15, 06:33 AM
@Goto124: You literally cannot, and i mean cannot make bows and slings rare in any setting without some pretty massive changes of another kind. They weren't developed as weapons of war, they were your primary food gathering tool, a vital part of basic survival for literally millennia.

Not necessarily. The Australian Aboriginal people used the woomera (https://en.wikipedia.org/wiki/Woomera_(spear-thrower)),which was much like an atlatl, for hunting not to mention the far more famous boomerang.

Hunting sticks that were thrown to incapacitate prey were also found outside Australia, although they were supplanted by slings and bows.

That said, the setting wouldn't need hunting at a distance unless they developed an agrarian society and technology very early on, but that would significantly change development of the setting. Given the effectiveness of being able to kill at a distance, I would be very surprised if the technology didn't spread rapidly once discovered, so there must be some other reason why missile weapons (not just gunpowder) are rare.

Spiryt
2015-06-15, 08:50 AM
Honestly, 'bronze age' human transported into modern setting would simply freak out/panic, if we want to be realistic.

Wondering what weapon and curses can they use is bit superfluous.

They would use whatever curses corded ware cultures, Ancient Egyptians, or whoever were using. :smallwink:

Kiero
2015-06-15, 09:48 AM
If they're from a medieval setting... they've never worn bronze, that stuff was largely phased out before Jesus was born.

No it wasn't. Bronze continued to be used for armour for a long time after that. Both as cuirass and scales on leather. There were Romans assaulting Jerusalem decades after Jesus' death who would have worn bronze armour (officers usually). Some might even have had heirloom bronze swords (well-worked bronze is better than low-grade steel).

warty goblin
2015-06-15, 10:17 AM
It would have to be a pretty odd homebrew setting, given that slings date back to the Stone Age. I'm also curious how full plate developed without gunpowder weapons to spur its development.


Bronze plate armor developed absent firearms, so it's entirely plausible that a person from a bronze age culture would be familiar with the stuff. It wouldn't be exactly like the medieval stuff, but relatively similar.

Storm_Of_Snow
2015-06-15, 10:30 AM
IMO, anyone coming from medieval period to the modern day would be suffering from the most unimaginable culture shock - buildings made of metal belching smoke and moving by magic (buses), shiny metal dragons flying through the air (aircraft), magic talking picture boxes (TV), rocks reaching up to the gods (skyscrapers) and everything else we take for granted.

Maybe if there's some merging of the hosts mind with theirs they'll be able to do something other than find the nearest hole in the ground, crawl into it and whimper. And if they've got that, they may have some of the skills they'll need, even if they don't actually have the knowledge around it.

Kiero
2015-06-15, 10:32 AM
Oh dear, we seem to have another rendition of the "bronze is inferior to iron" meme, which is patent nonsense. Bronze didn't fall out of use because iron (or low-grade steel) is better, it fell out of use because bronze is more expensive than iron. The reason for that is the relative shortage of one of it's two vital components: tin. Well-worked bronze is superior to iron and low-grade steel, especially for armour plates, but also for edges on blades too. What iron/steel has over bronze is that iron ore is common, and so are sources of carbon to heat it (charcoal or coal). Therefore, once you have the infrastructure to process it, it's much cheaper than making iron. It's also heavier than the same volume of steel, but only by about 10%.

It's no coincidence that even while the common, rank-and-file Republican Roman legionary might be wearing iron mail and using an low-grade steel sword, the officers leading them might still have bronze swords, and even be wearing bronze cuirasses.

Brother Oni
2015-06-15, 10:59 AM
Bronze plate armor developed absent firearms, so it's entirely plausible that a person from a bronze age culture would be familiar with the stuff. It wouldn't be exactly like the medieval stuff, but relatively similar.

Really? The bronze armour with the most coverage I know of, is greek hoplite panoply, but I didn't think it got as heavy duty as say gothic plate or its successors.

Do you have any pictures or examples?

goto124
2015-06-15, 11:06 AM
find the nearest hole in the ground, crawl into it and whimper.

Sounds like a plan.

warty goblin
2015-06-15, 11:23 AM
Really? The bronze armour with the most coverage I know of, is greek hoplite panoply, but I didn't think it got as heavy duty as say gothic plate or its successors.

Do you have any pictures or examples?

There's the Dendra armor, which IIRC dates from around 1400 or 1450 BC:

http://catherinemayoauthor.com/a/wp-content/uploads/2013/06/Dendra-Armour-Mylonas.jpg
This is often claimed as being armor for a chariot archer or driver, but I've also seen people who own accurate recreations say it's practical for infantry combat as well.

There's fragments of armor from the Theben arsenal, suggesting that the Dendra style armor developed into something smaller and simpler by the 1300's BC, but it's rather hard to figure exactly what it looked like.

Kiero
2015-06-15, 12:02 PM
Really? The bronze armour with the most coverage I know of, is greek hoplite panoply, but I didn't think it got as heavy duty as say gothic plate or its successors.

Do you have any pictures or examples?

Scythian leather-backed scale covered the entire body, if a lord was rich enough to own such a suit.

Stellar_Magic
2015-06-15, 02:48 PM
Very true... and I didn't ever say it was bad stuff, I just said that bronze was largely phased out by the time of Christ. It was phased out due to expense, as there really wasn't a material better then bronze for weapons until carbon steel. Iron just was easier to forge, with higher hardness but relatively brittle.

Okay... well, full-plate would be really rare for bronze as that stuff is cast, ground, and polished... tailoring it to properly fit an individual would be a royal pain. That's probably why stuff akin to scale mail was a common way to make bronze and early iron full-body armors.

Some of the stuff you've said are rather confusing... Just for clarity.

Stone Age > Bronze Age > Iron Age > Medieval Period > Renaissance

Bows and Arrows date from the neolithic period (stone age), and are nearly universal in their distribution with only a few exceptions like the aborigines of Australia and so forth. Bone tipped, stone tipped, and even arrowheads made from fish scales (Alligator Gar scales in the Americas) were used before the advent of bronze and even long after.

Full-plate armor, as we know it, dates from the late medieval era where it was actually pitted up to and measured against firearms (until firearm advancements made full-plate armor insufficient). This doesn't mean that rather elaborate plate armor did not exist or wasn't used earlier, but usually such armor wasn't nearly as protective as what we think of as 'full-plate', mostly because bronze is really much harder to articulate, and so areas like the legs and arms were more often left free of protection.

The sling is arguably one of the oldest, if not the oldest weapon on the planet. Alongside the spear, atlatl, and bow the sling dates from the stone age... though it quite likely was used by the earliest humans alongside fire hardened spears and flint. This doesn't mean it wasn't a deadly weapon... especially into the early iron age period. A heavy cast iron, steel, or bronze bullet is actually surprisingly effective against armor when thrown from a sling. In fact, early Roman legions relied primarily on the sling instead of the bow for long ranged weapons.

Just to be clear... Bronze age is not medieval... In fact the whole of Roman history separates the Bronze Age from the Medieval Period.

Gnoman
2015-06-15, 03:21 PM
@Regarding the Bronze-age warrior bodyswapped into a modern SWAT team:

The crucial thing that everyone is overlooking is "muscle memory", which would almost have to remain with the body (else the person would quite literally be unable to stand without a great deal of practice.) Anyone who works with weapons regularly (maybe not a security guard or civilian with a "just in case" revolver, but definitely any soldier or SWAT member) drills proper handling into themselves so that they do things like keeping their finger off the trigger or practicing muzzle safety in complete absence of conscious thought (much in the same way you don't think about the details of moving your own arm and knowing exactly how far you can reach). I find it highly likely that the transplantee would handle everything well until they thought about it, then be completely lost.

Brother Oni
2015-06-15, 06:12 PM
There's the Dendra armor, which IIRC dates from around 1400 or 1450 BC:


Scythian leather-backed scale covered the entire body, if a lord was rich enough to own such a suit.

Interesting - thanks for the information.


This doesn't mean that rather elaborate plate armor did not exist or wasn't used earlier, but usually such armor wasn't nearly as protective as what we think of as 'full-plate', mostly because bronze is really much harder to articulate, and so areas like the legs and arms were more often left free of protection.

Also much thinner - reproductions of the Dendra armour that warty mentioned have the breastplate between 1-1.3mm thick and a report of the original armour indicates the greaves and forearm armour were 'paper thin'. While there's obviously degradation of the original armour, it should be remembered that they typically fought with a shield back in the Bronze Age, thus arm armour was a bit excessive (not to mention redundant weight - one reproduction Dendra panoply was 25kg without the helmet).



The sling is arguably one of the oldest, if not the oldest weapon on the planet. Alongside the spear, atlatl, and bow the sling dates from the stone age... though it quite likely was used by the earliest humans alongside fire hardened spears and flint. This doesn't mean it wasn't a deadly weapon... especially into the early iron age period. A heavy cast iron, steel, or bronze bullet is actually surprisingly effective against armor when thrown from a sling. In fact, early Roman legions relied primarily on the sling instead of the bow for long ranged weapons.

I think the club (or other big stick), spear and throwing stick would probably pre-date the sling by a bit, but I would otherwise agree.

As an aside about Roman slingers, in typical soldier humour, they used to engrave little messages in their bullets, from their unit or home city, to insults or simply "Catch!": link (https://militaryhistoryblog.wordpress.com/2008/02/12/aerial-insults-the-tradition-of-inscribing-lead-sling-bullets-in-antiquity/).



Anyone who works with weapons regularly (maybe not a security guard or civilian with a "just in case" revolver, but definitely any soldier or SWAT member) drills proper handling into themselves so that they do things like keeping their finger off the trigger or practicing muzzle safety in complete absence of conscious thought (much in the same way you don't think about the details of moving your own arm and knowing exactly how far you can reach).

You'd think SWAT would know better, but then pictures show up like that NYPD SWAT officer who had his sight mounted backwards on his weapon.
I remember a little picture quiz someone once did of various military/paramilitary/police units, where the aim was to guess whether they were military or police. Pretty much every police unit was guessed correctly due to their lack of trigger discipline.

While I agree that muscle memory is most certainly a thing, it depends on the type of possession. Assuming that her host body did retain its muscle memory, it would suggest that she didn't bring her own muscle memory with her (as that's still in her old body), thus making her ineffective in melee combat until she manages to reintegrate the training in her new host body ("I know how to do this, why isn't this stupid body responding how it should?!").

Knowing how far you can reach is a bit of a tricky example however; while I agree that reaching out for something is picked up during infancy, knowing your reach in combat (particularly melee combat) is something else and definitely takes some training (the concept is known as maai in Japanese martial arts (https://en.wikipedia.org/wiki/Maai)).

rs2excelsior
2015-06-15, 07:01 PM
Regarding the Bronze Age person in the modern day... I think she would hold the Uzi correctly (in a general sense), just because that's the easiest way to hold it. Y'know, since it's designed to be held that way. Her accuracy and trigger/fire discipline would probably be rather terrible, if she could figure out how to fire it at all. As to whether she would figure it out, that really depends on the cleverness of your character.

I doubt she'd look down the barrel, especially after she saw one shoot.

WRT muscle memory, despite the name, that's a mental thing. Subconscious, yes, but not stored in the muscles. So she should have all of her old muscle memory, and none of the new. Of course, we can't actually mindswap people to figure out exactly how true that is.

Gnoman
2015-06-15, 07:13 PM
While I agree that muscle memory is most certainly a thing, it depends on the type of possession. Assuming that her host body did retain its muscle memory, it would suggest that she didn't bring her own muscle memory with her (as that's still in her old body), thus making her ineffective in melee combat until she manages to reintegrate the training in her new host body ("I know how to do this, why isn't this stupid body responding how it should?!").

Knowing how far you can reach is a bit of a tricky example however; while I agree that reaching out for something is picked up during infancy, knowing your reach in combat (particularly melee combat) is something else and definitely takes some training (the concept is known as maai in Japanese martial arts (https://en.wikipedia.org/wiki/Maai)).

My point is that kinesthetics, the operation of one's limbs, proper control of one's strength, and knowing one's reach is a learned behavior, and is specific for a given body. Since we assume that the person is able to walk normally and manipulate objects normally; we must also assume that the other ingrained physical reflexes are with the body and not the conscious mind.



WRT muscle memory, despite the name, that's a mental thing. Subconscious, yes, but not stored in the muscles. So she should have all of her old muscle memory, and none of the new. Of course, we can't actually mindswap people to figure out exactly how true that is.

While muscle memory is mental, it is so necessary for the functioning of the body that, unless it remains in the body, the swapped individuals would be entirely unable to operate that body. The process of growth alone is a serious obstacle (witness the clumsiness of teenagers, or watch someone who gained or lost a large amount of weight very quickly and you'll see what I mean), and sudden replacement of the entire body would take weeks, if not years, to adapt to.

Mr. Mask
2015-06-15, 07:43 PM
A popular trope is that during WW2, the American Civil War, or some other famous historical conflict where the fighting got fierce and desperate, one side discovers alien technology, or magic( technology), or steampunk, or mcguffin artefacts, or weaponizable zombies.

My question is, if a military suddenly did find some kind of powerful, potentially game-changing weapon, how would they go about incorporating it into their military? In particular, if the weapons are very different from how the military's current weaponry and tactics are used (like incorporating zombies, or super anime mechs).

Gnoman
2015-06-15, 08:18 PM
A popular trope is that during WW2, the American Civil War, or some other famous historical conflict where the fighting got fierce and desperate, one side discovers alien technology, or magic( technology), or steampunk, or mcguffin artefacts, or weaponizable zombies.

My question is, if a military suddenly did find some kind of powerful, potentially game-changing weapon, how would they go about incorporating it into their military? In particular, if the weapons are very different from how the military's current weaponry and tactics are used (like incorporating zombies, or super anime mechs).

That depends greatly on how sustainable the weapon is - cartridges for an AK-47 can be stamped out in American Civil War era munitions plants and loaded with contemporary powder, keeping the weapon usable even though the performance would be greatly reduced, while if the Bonhomme Richard found a Harpoon launcher Jones would be completely incapable of producing additional missiles for it.

If the new weapon can be reliably supplied with ammunition, fuel, souls, or whatever it operates on, and can be manufactured or acquired in useable quantities (even with fuel and ammunition, one tank would be hard-pressed to win the Punic Wars for Carthage); then the incorporation would vary depending on just how game-changing it is. If it's something like the atomic bomb that can end the war with only a few uses, they would just throw it around right away and worry about formalizing the system later. If it is merely an equalizer or otherwise a modest improvement, it would go through much the same acquisition process used for any new weapon. It would be demonstrated for high ranking officials, those officials would authorize a testing program to work out any issues while they write directives on the use of the new equipment. From there a limited deployment would be made to test it under combat conditions, -if that proved satisfactory- it would be placed into general service. It is unlikely that the new weapon is not going to mesh in some manner with an existing branch - zombies would go in the infantry, they'd assign Gundams to the cavalry, and airships would probably find their home in the navy.

fusilier
2015-06-15, 08:19 PM
A popular trope is that during WW2, the American Civil War, or some other famous historical conflict where the fighting got fierce and desperate, one side discovers alien technology, or magic( technology), or steampunk, or mcguffin artefacts, or weaponizable zombies.

My question is, if a military suddenly did find some kind of powerful, potentially game-changing weapon, how would they go about incorporating it into their military? In particular, if the weapons are very different from how the military's current weaponry and tactics are used (like incorporating zombies, or super anime mechs).

Here's my ideas, I'm sure others will think of other issues:

One example would be atomic weaponry, which was used very sparingly to end WW2. After the war, a whole system of "non-warfare" developed around it.

There are other historical antecedents, but they usually involve both sides developing it (or developing a response to it). WW1 provides lots of examples. The development of fighter aircraft happened on both sides. Chemical Warfare was introduced by the Germans but quickly developed by the Allies, who were also quick to put countermeasures in place to prevent the Germans from capitalizing on their early advantage. The Allies developed tanks, but the Germans never felt pressured to make them in large numbers themselves*. On the other hand the Germans developed highly effective infiltration tactics, and the Allies developed defensive schemes that could counter it.

Now, you could argue that's because both sides were roughly on par in terms of general technology and tactical thinking -- but what if we could give one side something far more technologically advanced? The problem then becomes logistical. In a couple of senses: 1. The new tech has to be made and distributed, and 2. The new tech has to be used appropriately, which entails both discovering the appropriate use (a tactical problem), then training, or retraining, troops in the new tactics (a logistical issue).

Again returning to WW1. Italy fielded the first sub-machinegun in 1915 -- the Villar Perosa. It was an odd weapon. Double barreled and lacking a stock. Originally intended for aircraft use (for which it was not well suited due to it's short range), it was fielded by the army fixed to a ball mount in the middle of heavy armor shield! I.e. it was used like a heavy machine gun. In 1916 a bi-pod was fitted to the design, and it could be employed like a light machine gun. The arditi (assault troops), fitted a rig/sling to it so it could be fired on the move in 1917. Finally in 1918 Beretta designed the MAB 1918 around the basic action of the Villar-Perosa. It was now a proper sub-machine gun, single barrel, with a wood stock, and folding bayonet, perfect for assault use. (It was never deployed in numbers like the more well known German MP18 although it technically predates it). It took *years* to develop tactics which could properly exploit the advantages of the weapon -- and that was during wartime.

So if the weapon dovetails nicely with existing tactics, then it would be easier to employ it. But if it doesn't then there may be an awkward learning curve, possibly requiring the retraining of an entire army in wartime. That's not impossible - new troops are being recruited, and veterans were often pulled out of the line for more training anyway - but it takes time.

-----
* The effectiveness of tanks on the outcome of the war is debated. Some claim that they were a decided advantage for the Allies. Others point to the lack of reliability, and the fact that the Germans had some very good successes without them, that show that tanks were not "war-winning" weapons on their own. I think it's hard to tell, as WW1 was really a war of attrition, and the Allies won the war from that perspective and the Germans lost it.

Gnoman
2015-06-15, 08:25 PM
Here's my ideas, I'm sure others will think of other issues:
The Allies developed tanks, but the Germans never felt pressured to make them in large numbers themselves*. On the other hand the Germans developed highly effective infiltration tactics, and the Allies developed defensive schemes that could counter it.


The Germans put a crash program in place to develop tanks, and the only reason that they didn't employ them en masse is because the first prototype was not ready until late 1917, with production starting in October. They didn't reach the front until March due to teething problems, and the war ended before the replacement for the very poor A7V design could be finished.

Also, infiltration was a French invention, not a German one.

fusilier
2015-06-15, 08:31 PM
Also, infiltration was a French invention, not a German one.

Not sure what you are referring to. There were antecedents, but the Germans were the first to formalize and capitalize upon them on a truly large scale:

https://en.wikipedia.org/wiki/Infiltration_tactics

Gnoman
2015-06-15, 08:36 PM
Not sure what you are referring to. There were antecedents, but the Germans were the first to formalize and capitalize upon them on a truly large scale:

https://en.wikipedia.org/wiki/Infiltration_tactics

That very article states that the Germans didn't use the tactic until late 1916, and that it was first used by the French in April 1915.

fusilier
2015-06-15, 08:49 PM
That very article states that the Germans didn't use the tactic until late 1916, and that it was first used by the French in April 1915.

From the wikipedia article (emphasis mine):

A form of infiltration tactics was proposed by French headquarters . . . These were partially adopted on 9 May 1915. . . .

The Germans were already developing their own form of infiltration tactics: an experimental Pioneer unit commanded by Hauptmann Willie Rohr had been formed in the spring of 1915, over two months before Laffargue's pamphlet was published.[

If you want to say that the French invented "Infiltration Tactics" I suppose you can, but it's misleading. Regardless, the Germans *perfected* it, and they are generally given the credit for developing it into an effective and *potentially* game changing tactic. The Allies weren't as static in their tactical thinking as many claim, but when people refer to "Infiltration tactics" they usually mean the German version of it.

There's no equivalent to Caporetto or the Spring Offensive from the Western Allies point of view. While the Russian Brusilov offensive of 1916 shows certain aspects of what would become Infiltration tactics, it was not as developed.

EDIT --
Actually Gnoman you brought up a really good point. When people refer to WW1 infiltration tactics, they usually mean the German style that was employed very effectively (and without further qualification that's probably what is meant).

However, a result of the German success is that people tend to think that Allied tactical thinking was very simplistic and that's not true. As you noted the French were working on their own forms of infiltration tactics from an early point. The Brusilov offensive showed some of the more advanced Russian thinking. The Italians developed "Arditi" tactics in 1917 which developed from a couple of different converging branches going back to just before the war. They were reasonably successful but never had the huge impact that the German Infiltration tactics (also called "Hutier tactics") had.

goto124
2015-06-15, 09:16 PM
The crucial thing that everyone is overlooking is "muscle memory", which would almost have to remain with the body (else the person would quite literally be unable to stand without a great deal of practice.)

Maybe possession lets you have only a small amount of the muscle memory of the guy you possessed, just enough for you to walk around and pick up things and such.

Then she won't be able to operate a gun. She'll also be rather clumsy with the body in general, making martial combat difficult as well.

Like possession from Dishonored, I think. The control offered by possession is limited and does not allow possessed people to fight, jump, or climb (according to the wiki).

At this point, possession is useful only to blend in with the crowd and escape suspicion for just a bit. Which is still very useful.

Gnoman
2015-06-15, 09:34 PM
Then she won't be able to operate a gun. She'll also be rather clumsy with the body in general, making martial combat difficult as well.


To prevent misunderstanding, I am not claiming that she could operate the weapon from sheer muscle memory - that always requires a concious action that our Bronze-Ager would be compleletley incapable of initially. What I am saying is that things like simply carrying it, holding it in the right places, and not accidentally shooting anybody would not be a problem unless she looked at it and started wondering "what is this thing" and thinks about what she is doing.

Stellar_Magic
2015-06-15, 09:58 PM
One thing I've been thinking about recently is how to model the difference in effectiveness of something like... a Battleship gun, against a small human sized target (note this is about hitting directly... I know that the blast would be more than fatal), in a D20 system. Saga Edition had a -20 penalty for turbolasers used against targets smaller then Colossal size, for example.

I'm wondering if a similar rule could be applied in a sort of universal fashion. If the target is more than two size categories smaller then the person wielding it or the weapon mount's size category, then it incures a -20 penalty. Do you think this would be realistic, or a bit excessive?

Gnoman
2015-06-15, 11:47 PM
One thing I've been thinking about recently is how to model the difference in effectiveness of something like... a Battleship gun, against a small human sized target (note this is about hitting directly... I know that the blast would be more than fatal), in a D20 system. Saga Edition had a -20 penalty for turbolasers used against targets smaller then Colossal size, for example.

I'm wondering if a similar rule could be applied in a sort of universal fashion. If the target is more than two size categories smaller then the person wielding it or the weapon mount's size category, then it incures a -20 penalty. Do you think this would be realistic, or a bit excessive?

It's complicated, but essentially the answer is no. Heavy artillery is actually extremely accurate, it's a matter of finding the range, adjusting for motion of the carrier and the target, and knowing the individual shell characteristics. If stationary, a man is no more difficult to hit than anything else, and there are confirmed reports of 16-inch shells scoring first-round hits on trucks 20 miles away. What they are bad at is engaging moving targets - it is fairly difficult to move such a large barrel (the inertia alone is a problem), and trying to hit a moving vehicle is not easy, particularly on a ship (where the motion of the ocean has to be compensated for) or a self-propelled gun (where the vehicle has to settle down before firing). Not only that, but a man can easily disappear, making it hard to track and engage him.

That said, the biggest obstacle to using such heavy weapons against individuals is that it is wasteful - even machine guns very rarely target that way, instead focusing on filling a given area with bullets, and there is always a more valuable target for heavy arty than an individual, or even an individual squad.

Stellar_Magic
2015-06-16, 12:32 AM
Huh, how do you get direct first round hits, with a 16 inch shell? I could see one landing directly, but those 16-inch batteries don't converge at a point, do they? So wouldn't one shell hit left, another hit right, while the middle one hit directly?

Same thing with a .50 caliber ball turret. Wouldn't one hit the target in the left side, and the other on the right? If the guy wasn't as wide as the spacing between the two guns (like a small sized creature) then wouldn't you have to intentionally aim slightly left or right to hit?

If I went with this -20 penalty, I might make it so that it can be removed by taking time to aim. Playtesting it a bit, it seems to work well and even out encounters between widely disparate size categories (Hobbit vs. Troll... Troll has a -20 to hit, deals x4 damage and x4 times the HP of the hobbit), so it might not be completely realistic... but it does seem to work really well in terms of balancing.

It does seem to work really well... I mean a Torpedo boat actually can race into torpedo range when the main guns of the battleship have a -20 to hit em... Secondary and tertiary guns are another matter, but that's when a torpedo boat would launch it's torpedoes.

Brother Oni
2015-06-16, 02:22 AM
Huh, how do you get direct first round hits, with a 16 inch shell? I could see one landing directly, but those 16-inch batteries don't converge at a point, do they? So wouldn't one shell hit left, another hit right, while the middle one hit directly?


You don't have to fire out of all three guns in a turret at the same time. It's perfectly possible to fire one gun, rotate and fire the second, then adjust and fire the third - it's why firing solutions take some time to calculate.

As an example of how complex they can get, they can fire three full salvos out of a battery and have all three hit the target at the same time.

Regarding the accuracy of an Iowa class' main guns, during 1987 test firings with a new radar and FCS, they shot 15 shells out of the right gun of each turret (5 a piece) at a target area 220 yds in diameter at a range of 34,000 yards (19.3 miles). 14/15 shells landed within 250 yds of the target centre and 8 were within 150 yds, giving a total shell to shell dispersion of 123 yds.

While it sounds fairly inaccurate, bear in mind that the High Capacity shells fired could create crafters 50ft wide by 20ft deep - during the Vietnam War, the USS New Jersey used to fire off a single HC shell to create helicopter landing zones 200 yards in diameter (it also defoliated trees for an additional 300 yards beyond that).

Gnoman
2015-06-16, 02:51 AM
Huh, how do you get direct first round hits, with a 16 inch shell? I could see one landing directly, but those 16-inch batteries don't converge at a point, do they? So wouldn't one shell hit left, another hit right, while the middle one hit directly?

Same thing with a .50 caliber ball turret. Wouldn't one hit the target in the left side, and the other on the right? If the guy wasn't as wide as the spacing between the two guns (like a small sized creature) then wouldn't you have to intentionally aim slightly left or right to hit?

It does seem to work really well... I mean a Torpedo boat actually can race into torpedo range when the main guns of the battleship have a -20 to hit em... Secondary and tertiary guns are another matter, but that's when a torpedo boat would launch it's torpedoes.

1. Even if you fire every gun simultaneously, the shells would land so close together that the spacing literally has no effect - if you're small enough for the gap to matter, there's no functional difference between a 16" (or 8", or even 4") gun striking you directly and hitting slightly to the side - you're paste either way.

2. A ball turret, or any other multiple machine gun mount is, in fact, mounted to converge at a set distance (I believe that 300 meters was a common setting, but it was easily adjustable). Even if you are small enough to fit between the barrels in the first place, and close/far enough from the point of divergence that the bullets still pass on either side, the effect of recoil and traverse scatter the bullets enough that the illusionary safe zone simply doesn't exist. For that matter, if you're that small to begin with, a HMG or autocannon bullet could kill with passage alone, much as artillery shells have been known to do with humans.

3. This effect has nothing to do with the accuracy of the gun - the cause (and what the Star Wars turbolaser rules are intended to model) is that torpedo boats are very fast, and the heavy guns have great difficulty traversing fast enough to hit them. They can also get "under" the guns (where the guns physically can't aim low enough to shoot at them) if they get close enough. Finally, the big guns have such a slow fire rate that they can't get a hit through mere volume of fire. A torpedo boat or destroyer moving slowly at range is nothing more than an easily killed target.


Your hobbit vs. troll analogy proves why this has no basis in realism - how often does a mosquito's small size (and not the insect's power of flight) make it hard for you to swat it? That's a far greater size difference.

Mr. Mask
2015-06-16, 04:18 AM
If you could develop some mutant super soldiers with inhuman or superhuman qualities, what traits would you give them? And if there needed to be drawbacks, what drawbacks would you give them?

Note that these don't have to be infantry. They cant also be pilots, tank commanders, artillery spotters, whatever you like.


I've always felt high strength has had a lot of military quality to it, as it allows you to carry heavier armour, heavier weapons, more equipment, and lets you load heavier shells. For drawbacks, I've been wondering if lack of speed is a good drawback to have. So long as you can march steadily, and it doesn't effect your piloting abilities, modern troops don't need to be that fast, it appears.

snowblizz
2015-06-16, 04:26 AM
I've always felt high strength has had a lot of military quality to it, as it allows you to carry heavier armour, heavier weapons, more equipment, and lets you load heavier shells. For drawbacks, I've been wondering if lack of speed is a good drawback to have. So long as you can march steadily, and it doesn't effect your piloting abilities, modern troops don't need to be that fast, it appears.

Or in another way, Supermutants from Fallout series video game, are they realistic as the "supersoldiers" (maybe, maybe not suffering form quality issues) they are supposed to portray.

Gnoman
2015-06-16, 05:18 AM
If I were to design the perfect infantryman, he would be short, muscular, and possess infrared vision. I'd also give him burrower's feet to let him dig a hole to hide in easier, and very, very smart. For vehicle crew, someone small and dextrous would be most useful, although strength would be an asset for some maintenance tasks.

Brother Oni
2015-06-16, 06:59 AM
Your hobbit vs. troll analogy proves why this has no basis in realism - how often does a mosquito's small size (and not the insect's power of flight) make it hard for you to swat it? That's a far greater size difference.

Agreed. The difference between a good game system and real life, is that a good game system is fair and balanced. Real life competitive events have to enforce things like weight and gender limits to maintain that balance and fairness, but outside of that, it's everything goes.


So long as you can march steadily, and it doesn't effect your piloting abilities, modern troops don't need to be that fast, it appears.

It depends on the role. Light infantry and airborne need to be fast, mechanised infantry, not so much.

They still have to be fast enough though - during the Falklands conflict, a large number of British casualties were from the sinking of a troop ship. These troops were on the ship only because they couldn't keep up with the Paras already on the island and were being shipped along to keep up.

The Battle of Mogadishu was another oddity where troop marching speed was of critical importance and I can't remember whether it was on this thread or somewhere else that suggested fast troops may have been a better choice than the humvee convoy (the enemy milita often heard the convoy approaching and set up accordingly).


If I were to design the perfect infantryman, he would be short, muscular, and possess infrared vision. I'd also give him burrower's feet to let him dig a hole to hide in easier, and very, very smart.

But not too smart, otherwise he'll be wondering why he was an infantryman in the first place. :smalltongue:

Carl
2015-06-16, 07:22 AM
But not too smart, otherwise he'll be wondering why he was an infantryman in the first place.

So wish there was a like button right now.


And this discussion on soldiers is one of the reasons modular power armour a la honorverse would probably trump engineered soldiers in reality. You can turn a soldier from a big brute lugging around the firepower of a small AFV into something fast enough to outrun the AFv and still match or exceed the firepower of a non-power armoured soldier. Or anything in between to suit your needs.

Galloglaich
2015-06-16, 10:07 AM
IMO, anyone coming from medieval period to the modern day would be suffering from the most unimaginable culture shock - buildings made of metal belching smoke and moving by magic (buses), shiny metal dragons flying through the air (aircraft), magic talking picture boxes (TV), rocks reaching up to the gods (skyscrapers) and everything else we take for granted.

Maybe if there's some merging of the hosts mind with theirs they'll be able to do something other than find the nearest hole in the ground, crawl into it and whimper. And if they've got that, they may have some of the skills they'll need, even if they don't actually have the knowledge around it.

Medieval <> Cavemen


Keep in mind, if we are talking actual medieval world and not faux fantasy generic whatever, they had a lot of things that would be familiar.

For example, tall buildings ... were not at all unusual in the medieval world

http://upload.wikimedia.org/wikipedia/commons/3/3a/LichCathedral5.jpg

http://travelerfolio.com/travelerfolio/photos/prague_old_town_square_03.jpg

http://www.letsgo-europe.com/Belgium/bruges/belgium_175.jpg



Other things...

http://3.bp.blogspot.com/_IN51M4RrPGI/TKfzwPCF1NI/AAAAAAAAAIw/4HfR4SYe9ik/s1600/kite.gif

http://www.tinasdynamichomeschoolplus.com/wp-content/uploads/Konrad_Kyeser,_Bellifortis,_Clm_30150,_Tafel_21,_B latt_91v.jpg

They may never have seen airplanes but it would not be unusual for them to see things like kites, as well as war rockets and other flying devices


Devices which made noises and music were also not at all unusual... they had for example a lot of automata (clockwork robots)

http://i.kinja-img.com/gawker-media/image/upload/s--RGb1_f8P--/c_fit,fl_progressive,q_80,w_636/17m3qoiph37fjjpg.jpg

and all kinds of musical instruments

http://sitemaker.umich.edu/musicalinstruments/files/minstrels_instruments.jpg

https://ferrebeekeeper.files.wordpress.com/2010/06/ren.jpg


http://www.uh.edu/engines/foliotverge.jpg

Machines were very common

G

Spiryt
2015-06-16, 10:13 AM
Very true... and I didn't ever say it was bad stuff, I just said that bronze was largely phased out by the time of Christ. It was phased out due to expense, as there really wasn't a material better then bronze for weapons until carbon steel. Iron just was easier to forge, with higher hardness but relatively brittle.


Actually, most irons are not very hard and certainly not brittle.

They are in fact quite ductile.

It seems that pure iron in general is something really malleable, we just never see really 'pure' iron.

Everything used by the man during history had some naturally occurring carbon atoms that make structure harder and less giving.




Okay... well, full-plate would be really rare for bronze as that stuff is cast, ground, and polished... tailoring it to properly fit an individual would be a royal pain. That's probably why stuff akin to scale mail was a common way to make bronze and early iron full-body armors.

As mentioned, big bronze surfaces weren't unheard of... Even if indeed painful to make.

Kiero
2015-06-16, 12:08 PM
Very true... and I didn't ever say it was bad stuff, I just said that bronze was largely phased out by the time of Christ. It was phased out due to expense, as there really wasn't a material better then bronze for weapons until carbon steel. Iron just was easier to forge, with higher hardness but relatively brittle.

Which again is total nonsense. Roman legionary officers in the siege of Jerusalem in 70AD, several decades after the death of Christ, were still wearing bronze armour. It wasn't largely phased out, it was still in active use. The fact that there was a glut of iron armour in use by the legionaries doesn't change the fact that bronze armour was still being used. And continued to be used for centuries afterwards, too.

Should we talk about what warship rams were made out of throughout the period? I'll give you a clue, it wasn't iron or steel.


Okay... well, full-plate would be really rare for bronze as that stuff is cast, ground, and polished... tailoring it to properly fit an individual would be a royal pain. That's probably why stuff akin to scale mail was a common way to make bronze and early iron full-body armors.

Do you know what a hoplite panoply looks like? A cuirass is a single, large plate of bronze, attached to a second, large plate of bronze. It also has large plates to cover the shins, smaller ones for the front of the thighs and for forearms and upper arms. Which was tailored to fit an individual, if you had the money to get a suit custom-made. This was just as common as scale in the Iron Age. Though outside the Classical Era, it tended to be replaced with a textile cuirass.

How do you get the plate to fit? One of the techniques was planishing (https://en.wikipedia.org/wiki/Planishing). One of the earliest jobs a bronzesmith's apprentice would have is hammering out sheets of bronze into plates of equal thickness all over.


Stone Age > Bronze Age > Iron Age > Medieval Period > Renaissance

Where and when are you talking about? Because they lasted for different lengths of time in different places, and there was no nice, neat, clear delineation between one period and the next. This implies a rather naive understanding of the terms. Bronze Age and Iron Age overlapped a great deal if you travelled a few hundred miles.

The most cursory bit of reading on Wikipedia (https://en.wikipedia.org/wiki/Bronze_Age) would demonstrate this, the Bronze Age in Europe lasted til 600BC, but was finished in the (not that distant) Near East by 1200BC. And so on it goes.


Just to be clear... Bronze age is not medieval... In fact the whole of Roman history separates the Bronze Age from the Medieval Period.

"Bronze Age" has nothing whatsoever to do with whether or not people were using bronze to make armour from.

MrZJunior
2015-06-16, 01:30 PM
If you were supplying an insurgency in a jungle environment from the air with approximately late nineteenth century technology, what sort of things would you ship?

There's the obvious stuff like rifles, ammunition, and explosives. Less obvious stuff would be bicycles, possibly land mines, though I don't think those were in use in the actual late nineteenth century.

Stellar_Magic
2015-06-16, 01:34 PM
Lordy, you guys are misinterpreting what I said.


Which again is total nonsense. Roman legionary officers in the siege of Jerusalem in 70AD, several decades after the death of Christ, were still wearing bronze armour. It wasn't largely phased out, it was still in active use. The fact that there was a glut of iron armour in use by the legionaries doesn't change the fact that bronze armour was still being used. And continued to be used for centuries afterwards, too.

Should we talk about what warship rams were made out of throughout the period? I'll give you a clue, it wasn't iron or steel.

So 1/100 soldiers having bronze armor means it's not been 'largely phased out'? Sure officers had it, but no one else usually did by that time.

As for rams, they were bronze because they didn't have the technology to cast something that big out of iron. Same reason early cannons were usually made of bronze.


Do you know what a hoplite panoply looks like? A cuirass is a single, large plate of bronze, attached to a second, large plate of bronze. It also has large plates to cover the shins, smaller ones for the front of the thighs and for forearms and upper arms. Which was tailored to fit an individual, if you had the money to get a suit custom-made. This was just as common as scale in the Iron Age. Though outside the Classical Era, it tended to be replaced with a textile cuirass.

How do you get the plate to fit? One of the techniques was planishing (https://en.wikipedia.org/wiki/Planishing). One of the earliest jobs a bronzesmith's apprentice would have is hammering out sheets of bronze into plates of equal thickness all over.

I was comparing it to full-plate, with total articulation... That was something they didn't do.


Where and when are you talking about? Because they lasted for different lengths of time in different places, and there was no nice, neat, clear delineation between one period and the next. This implies a rather naive understanding of the terms. Bronze Age and Iron Age overlapped a great deal if you travelled a few hundred miles.

The most cursory bit of reading on Wikipedia (https://en.wikipedia.org/wiki/Bronze_Age) would demonstrate this, the Bronze Age in Europe lasted til 600BC, but was finished in the (not that distant) Near East by 1200BC. And so on it goes.

I was going with Europe, because I assumed they were talking about a fantasy world... which usually roughly follow European technological progression. My point was that a 'Medieval' character wasn't a 'Bronze Age' character.


"Bronze Age" has nothing whatsoever to do with whether or not people were using bronze to make armour from.

*Sigh* I'm sorry that saying that 90%+ of armor made during the Medieval Period would be made from either iron or carbon steel?

Gnoman
2015-06-16, 02:50 PM
If you were supplying an insurgency in a jungle environment from the air with approximately late nineteenth century technology, what sort of things would you ship?

There's the obvious stuff like rifles, ammunition, and explosives. Less obvious stuff would be bicycles, possibly land mines, though I don't think those were in use in the actual late nineteenth century.

This is a complex question, and one that depends heavily on the geopolitical situation. It's a bit later than your reference point (although nailing down the tech level would be appreciated, the line between late 19th century and WWI is kind of blurry), but the United States Forces In Philippines (a Filipino-American guerrilla force that was led by a Colonel (self-promoted to Brigadier General) that refused to surrender to the Imperial Japanese Army) is a pretty good example of those complications. The main supplies sent were small arms, machetes, ammunition, medical supplies (lots of these), rations, and radios. The specifics of these were chosen for political reasons - for example, the primary small arm sent in was the US Carbine, Caliber .30, M1; and it was selected less for practical reasons than because there wasn't a single carbine in the country when the Philippines fell, and their presence would be solid proof that they were getting supplies. Conversely, the weapons provided early in the Vietnam conflict were usually obsolete junk so that the USSR had plausible deniability as to their source.

TL:DR - we need more information.

Kiero
2015-06-16, 03:25 PM
Lordy, you guys are misinterpreting what I said.



So 1/100 soldiers having bronze armor means it's not been 'largely phased out'? Sure officers had it, but no one else usually did by that time.

The Romans weren't the only soldiers in the world in the Roman period. There were still Hellenistic soldiers making regular use of Bronze, not to mention plenty of other cultures.


As for rams, they were bronze because they didn't have the technology to cast something that big out of iron. Same reason early cannons were usually made of bronze.

Nope, they cast them out of bronze because the ram was in constant contact with salt water. Iron + salty water = rust. Nothing whatsoever to do with being able to forge something that size.


I was comparing it to full-plate, with total articulation... That was something they didn't do.

First it was "you can't make big enough plates", now it's "you can't have full articulation". Stop moving the goalposts.


I was going with Europe, because I assumed they were talking about a fantasy world... which usually roughly follow European technological progression. My point was that a 'Medieval' character wasn't a 'Bronze Age' character.

So Italy and Greece aren't in Europe now?


*Sigh* I'm sorry that saying that 90%+ of armor made during the Medieval Period would be made from either iron or carbon steel?

Probably not, people didn't stop using bronze and that's leaving out textile or organic armours. Really, quit while you're ahead, and stop throwing out these silly, baseless assertions.

Thiel
2015-06-16, 03:25 PM
If you were supplying an insurgency in a jungle environment from the air with approximately late nineteenth century technology, what sort of things would you ship?

There's the obvious stuff like rifles, ammunition, and explosives. Less obvious stuff would be bicycles, possibly land mines, though I don't think those were in use in the actual late nineteenth century.

Boots, rations, boots, uniforms, medical supplies, boots, ammunition, small arms, boots. Did I mention boots?

Mr. Mask
2015-06-16, 08:45 PM
Kiero: This makes me curious. Could they make full articulated plate out of bronze?

Kiero
2015-06-17, 03:30 AM
Kiero: This makes me curious. Could they make full articulated plate out of bronze?

It's certainly possible to do so, I don't think the tolerances of bronze are so wildly different to steel that it would be unsuitable for the job. Though it would be about 10% heavier, which is not inconsiderable for the amount of metal you'd be wearing.

I don't know, though, whether ancient armour-smiths had the knowledge of creating articulating joints, or whether they'd have considered it worthwhile.

Incanur
2015-06-17, 10:28 AM
In theory I think you could make a circa-1500-style full-plate harness out bronze. The Dendra panoply already comes fairly close, though it lacks the same sort of articulations as later suits and thus wasn't/isn't as comfortable or effective. Nearly all circa-1500 full harnesses also incorporated bits of mail to cover the gaps. Making a comfortable full harness required skills that didn't necessarily exist in ancient times, even though the materials might have.

Kiero
2015-06-17, 11:12 AM
In theory I think you could make a circa-1500-style full-plate harness out bronze. The Dendra panoply already comes fairly close, though it lacks the same sort of articulations as later suits and thus wasn't/isn't as comfortable or effective. Nearly all circa-1500 full harnesses also incorporated bits of mail to cover the gaps. Making a comfortable full harness required skills that didn't necessarily exist in ancient times, even though the materials might have.

Yeah, I mean mail didn't exist til around 300BC, by which time most armourers were moving away from covering a man with bronze anyway. A textile thorax was often good enough protection for the body when you had a large shield, helmet and greaves, and more comfortable to wear on long marches. Even those who did still wear a full panoply tended to do away with the thigh and upper arm plates.

PersonMan
2015-06-17, 11:23 AM
Question: In a modern-day force, what sort of losses can be absorbed by a military? If we're looking at a country that has, say, 160,000 active troops and 80,000 in reserves, with a total population of about 66 million, what would their limits on losses taken be? Is it something mostly limited by the people - i.e., ability of the government to justify losing X troops - or is it more related to their ability to recruit/train replacements?

Additionally: If one country has complete air supremacy, what sort of losses would they be able to inflict upon a large advancing enemy force, assuming they have a large body of fighter/bombers and some dedicated ground attack aircraft? How much would that sort of air power be a deterrent to the enemy? Can one effectively protect a smaller force of one's own from a large enemy force by threatening to inflict massive losses if they move through areas you control with your air force?

Storm Bringer
2015-06-17, 12:01 PM
Question: In a modern-day force, what sort of losses can be absorbed by a military? If we're looking at a country that has, say, 160,000 active troops and 80,000 in reserves, with a total population of about 66 million, what would their limits on losses taken be? Is it something mostly limited by the people - i.e., ability of the government to justify losing X troops - or is it more related to their ability to recruit/train replacements?

In a modern context, training, arming and equipping (to a basic standard) large bodies of troops is possible to a extent that the major limiting factor is political will to take losses. see the German volksturm or the VC for examples.




Additionally: If one country has complete air supremacy, what sort of losses would they be able to inflict upon a large advancing enemy force, assuming they have a large body of fighter/bombers and some dedicated ground attack aircraft? How much would that sort of air power be a deterrent to the enemy? Can one effectively protect a smaller force of one's own from a large enemy force by threatening to inflict massive losses if they move through areas you control with your air force?

effectively total. massed air strikes would inflict sufficient losses to break any concentration of ground troops, given sufficient time and accurate targeting information, especially if its trying to manoeuvre (and thus is not hiding in cover).

they couldn't catch every tank, truck or man, but they can inflict such damage as to break any cohesion the reminder may have.

see the Highway of Death (https://en.wikipedia.org/wiki/Highway_of_Death)for a example of what you are talking about. While significant numbers of troops did manage to escape up that road, they got to the far end with nothing but the clothes on their back and maybe their rifle. Even if they could have stopped the US army before the retreat, they sure as hell were not stopping them afterwards.


with a ground force in play, you then have better targeting info for the air strikes, plus a threat of the ground force will limit the enemies options and force him to either concentrate to fight you (and take more losses form air strikes). stay dispersed to limit the air strikes (which leaves you vunerable to a ground attack*) or fall back.

using air support as a force multiplier is pretty much the US military doctrine since WW2. see any number of Vietnam flims for examples of isolated us troops fighting off waves of VC with liberal help of napalm and HE bombs.




* there is a quote form a Iraqi tank commander from 91. it goes something like "At the start of the air war, I had 36 tanks in my unit. after 30 days of air strikes, I had 30. after 30 minutes of tank fighting, I had none." his tanks survived the air war because they were well dug in, and the strikes were concentrated on the C2 and support systems more than the front line (who were left to "wither on the vine", as they say).

Gnoman
2015-06-17, 04:41 PM
Question: In a modern-day force, what sort of losses can be absorbed by a military? If we're looking at a country that has, say, 160,000 active troops and 80,000 in reserves, with a total population of about 66 million, what would their limits on losses taken be? Is it something mostly limited by the people - i.e., ability of the government to justify losing X troops - or is it more related to their ability to recruit/train replacements?


It's both - general infantry, truck drivers, common seamen and such can be replaced nearly indefinitely as long as the national will survives (unless the country literally starts running out of people, as happened with both Germany and Japan in WWII), but training is a major bottleneck for pilots, tank crew, elite units such as Airborne or mountain troops, most of the naval professions, logistics officers, and other vital tasks of similar skill. This can be clearly shown by the Empire Of Japan, which trained as many pilots in the entire war as the US trained in 1944 alone, greatly contributing to their heavy losses after the Battle of Midway, in which a huge percentage of the IJN's prewar pilot corps was killed - not only were they losing the enormous edge that their superbly trained veteran pilots gave them early in the war as American pilots gained combat experience, but the average IJN pilot after that time was half-trained and considerably worse than a fresh USN or USAAF one.

Hjolnai
2015-06-17, 11:51 PM
Assuming the knowledge existed, would it be possible to build a human-carrying hot air balloon in Medieval Europe?

The Chinese were using sky lanterns/Kongming lanterns for signalling before 300 AD, but I don't know if enough paper was readily available in Europe (one of the Montgolfier experiments apparently used sackcloth with three layers of paper inside). I doubt that silk would be available in sufficient quantities either. So, I guess half the question is: was there a sufficiently light airtight material available?

The other materials question to make it possible is that of fuel. I'm not sure how the energy density of available fuels stacks up to those available later, for the Montgolfier balloon in 1783.


On the other hand, if you could get such balloons working in, say, 1000AD, how would that change society? I'd expect it mostly to have military applications (scouting, possibly firebombing), since it seems unlikely to be cost effective for goods transport in trade. Messages might be easier to send, at least in the direction of the prevailing wind.

Gnoman
2015-06-18, 12:21 AM
Assuming the knowledge existed, would it be possible to build a human-carrying hot air balloon in Medieval Europe?

The Chinese were using sky lanterns/Kongming lanterns for signalling before 300 AD, but I don't know if enough paper was readily available in Europe (one of the Montgolfier experiments apparently used sackcloth with three layers of paper inside). I doubt that silk would be available in sufficient quantities either. So, I guess half the question is: was there a sufficiently light airtight material available?

The other materials question to make it possible is that of fuel. I'm not sure how the energy density of available fuels stacks up to those available later, for the Montgolfier balloon in 1783.


Silk was much more common than you might think. It was a popular luxury among the rich, and was produced in large quantities in Italy starting from somewhere in the early 1000s. Prior to that, it was imported quite a bit starting in the 100s.

As for fuel, whale oil would be more than sufficient, and was already being harvested by the French and others.



On the other hand, if you could get such balloons working in, say, 1000AD, how would that change society? I'd expect it mostly to have military applications (scouting, possibly firebombing), since it seems unlikely to be cost effective for goods transport in trade. Messages might be easier to send, at least in the direction of the prevailing wind.

Virtually none. That era didn't have the technology to build any practical propulsion device, and an uncontrollable balloon is essentially useless. Even stationary observation balloons would be barely worthwhile, as the observer wouldn't have the means to communicate very well with the rest of the army, so his observations wouldn't be much more timely than horse scouts, and the scale of battlefields in that era were small enough that a wide vista wasn't really needed.

There might be some cartographical advantages, and it would potentially be useful for purposes of art, but that's about it.

Brother Oni
2015-06-18, 01:49 AM
Virtually none. That era didn't have the technology to build any practical propulsion device, and an uncontrollable balloon is essentially useless. Even stationary observation balloons would be barely worthwhile, as the observer wouldn't have the means to communicate very well with the rest of the army, so his observations wouldn't be much more timely than horse scouts, and the scale of battlefields in that era were small enough that a wide vista wasn't really needed.

There might be some cartographical advantages, and it would potentially be useful for purposes of art, but that's about it.


I disagree on the usefulessness of static balloons as observation devices. In difficult or impassible terrain, the time spent inflating and coming down could easily beat horse scouts and commands/observations can still be sent while in the air by use of signal flags (or simply shouting if they're not too high up) or message bottles pulled up and down by rope.

I do agree they wouldn't provide revolutionary change, but certainly a significant tactical (and possibly strategic) advantage in certain situations.

Mr. Mask
2015-06-18, 03:28 AM
This reminds me of a plan during the American civil war to have men parachute down. It might've included balloons, but I think they were thinking of doing it from some particular cliffs.

MrZJunior
2015-06-18, 06:28 PM
I could see the balloons being a psychological weapon. Suspend a giant paper mache angel from underneath it and you might be able to convince your enemy that you have the forces of heaven on your side.

MrZJunior
2015-06-18, 07:37 PM
TL:DR - we need more information.

Sorry it took me so long to reply.

The players will be a group of mercenaries and adventures hired to improve the military of a small tropical island nation. The nation is invaded by Country A, an up and coming new power eager to carve themselves out their own sphere of influence. They are opposed by Country B an established empire emerging from a period of instability and looking to reestablish their influence on the international stage. Country A is an island nation and has the finest navy in the world, including their new dreadnought class of ships, the most powerful and technologically advanced warships in the world. Country B is more of a land power, with a large army and an excellent air force, including the most advanced zeppelins in the world able to fly faster, higher, and farther than any other airships in the world. County A will invade and occupy the coastline of the tropical nation and some of the outlying islands but won't be able to easily penetrate the heavily forested interior. Country B, interested in humiliating their rival, will use their air force to airlift supplies to the guerrillas in the jungle. It would be pretty obvious what they are doing, so probably little need to try for plausible deniability.

Technology wise it is slightly steampunk. They have zeppelins, dreadnoughts, and steam powered cars, but no radio, mechs, or other such things. Electricity is somewhat common.

I hope this helps.

Blackstone
2015-06-18, 07:53 PM
Very interesting, thank you.

fusilier
2015-06-18, 08:07 PM
On the other hand, if you could get such balloons working in, say, 1000AD, how would that change society? I'd expect it mostly to have military applications (scouting, possibly firebombing), since it seems unlikely to be cost effective for goods transport in trade. Messages might be easier to send, at least in the direction of the prevailing wind.

For military applications, the traditional use of (static) observation balloons is in static conditions, like sieges and WW1. These should not be confused with "Barrage Balloons" which are designed to protect ground targets, by forcing attacking bombers to fly around them (or above them, which reduces their bombing accuracy).

Fire bombing with free balloons has been tried: the Japanese during WW2 made use of the jet-stream to try to set some fires, and the Austrians attempted to use unmanned balloons armed with bombs during the Siege of Venice in 1849(?). The British also used untethered balloons during WW2, both for fire-bombing, but also to disrupt power lines. Their balloons were so cheap that it actually cost the Germans more in wear and fuel for their aircraft to shoot them down, then it cost the British to make them.

However, in all historical situations I can think of they used gas balloons. I'm not sure how using hot-air might affect the usability of the balloon, although I assume it would be less effective.

Gnoman
2015-06-18, 08:34 PM
Sorry it took me so long to reply.

The players will be a group of mercenaries and adventures hired to improve the military of a small tropical island nation. The nation is invaded by Country A, an up and coming new power eager to carve themselves out their own sphere of influence. They are opposed by Country B an established empire emerging from a period of instability and looking to reestablish their influence on the international stage. Country A is an island nation and has the finest navy in the world, including their new dreadnought class of ships, the most powerful and technologically advanced warships in the world. Country B is more of a land power, with a large army and an excellent air force, including the most advanced zeppelins in the world able to fly faster, higher, and farther than any other airships in the world. County A will invade and occupy the coastline of the tropical nation and some of the outlying islands but won't be able to easily penetrate the heavily forested interior. Country B, interested in humiliating their rival, will use their air force to airlift supplies to the guerrillas in the jungle. It would be pretty obvious what they are doing, so probably little need to try for plausible deniability.

Technology wise it is slightly steampunk. They have zeppelins, dreadnoughts, and steam powered cars, but no radio, mechs, or other such things. Electricity is somewhat common.

I hope this helps.

That helps a lot. The first thing troops will need is machetes. Lots of machetes. In heavy forest or jungle these are vital for getting around, and wear out shockingly quick. They're also a nice weapon. After that, you provide machine guns, shotguns, and rifles, in that order. Bicycles or steamcars probably won't help - that kind of terrain isn't friendly to the sorts of either that would be available, so mobility wouldn't help much. Mines (called torpedoes when they were used extensively in the American Civil War) would be useful, both water (throw a few in a river, they drift down to the bay and possibly blow up a dreadnought or supply ship) and land types. You'll also want to send a lot of uniforms and boots, antimalaria medication, morphine/opium/whiskey, sterile bandages, etc. Oh, don't forget the propaganda posters and leaflets (these could just be airdropped by B's air force). Lots and lots of explosives would be good also - these could be used for booby traps, blowing up barracks or supply warehouses (after they loot whatever's useful), clearing a landing area for supply drops, etc.

If possible, any weapons sent should be chambered for Country A's bullets to make raids on ammo depots a good way to resupply - carrying off a few crates of bullets and blowing up/booby-trapping the armory is much easier than trying to carry off a lot of guns as well.

MrZJunior
2015-06-18, 10:09 PM
The Japanese used bicycles to good effect during the advance on Singapore during WW2. The Malay peninsula was heavily jungled at that time. I believe the Viet Cong also used bicycles for transportation purposes.

Weren't machine guns unusual and expensive during the late 19th century? I know one was brought on the Emin Pasha Relief Expedition and they were used in various other colonial expeditions, but usually just a couple of them. Weren't shotguns not used by most militaries until WW1?

That idea of floating naval mines down the river is brilliant. Do you know of any cases where that tactic was used in real life?

Mr. Mask
2015-06-18, 10:27 PM
Can you fight while lying down? With a gun, you can shoot from a prone position. But what about with a spear?

More specifically, would mermaids on land have any combat capabilities, crawling about? Or would they be helpless? I figure the latter, but Knight of Lodis makes me curious.

Gnoman
2015-06-18, 10:41 PM
Weren't machine guns unusual and expensive during the late 19th century? I know one was brought on the Emin Pasha Relief Expedition and they were used in various other colonial expeditions, but usually just a couple of them.


Nope. Even before the Maxim Gun in 1883, the Gatling Gun and derivatives were pretty common in military circles. The Emin Pasha Relief Expedition only had one because it was a free sample, and the reason they didn't send a lot with the colonial expeditions is because they didn't need all that many - they were mostly used against opponents that had limited access to firearms in general, and relied on massed charges that could overwhelm the low fire rate of contemporary rifles - one or two machine guns turned those into a bad idea. Machine guns of various types were ordered by the hundreds (on the same scale as field artillery, which is how they were seen) in the late 19th century, and a heavy gun built on the Gatling principle (the 37mm Hotchkiss revolving cannon) was crucial in the naval battles of the Spanish American war.

As for shotguns, the simple fact that the main combat shotgun of the US army in WWI and II was the Model 1897 shows them to have been used in the period. They were very popular with the troops fighting the Filipinos at the end of the century because it is an extremely effective weapon for jungle fighting.



That idea of floating naval mines down the river is brilliant. Do you know of any cases where that tactic was used in real life?

The only example I can think of right no is the "Battle Of The Kegs" in 1778, where gunpowder-filled barrels were fitted with contact fuses and placed into the Delaware river. It didn't work due to a combination of the British fleet having moved to avoid ice floating down the river, and also to one of them going off prematurely and alerting the ships. I'm nearly certain that it's been done since, and the Mississippi was mined during the American Civil War, but I can't think of another example right now.

fusilier
2015-06-18, 10:43 PM
Weren't machine guns unusual and expensive during the late 19th century? I know one was brought on the Emin Pasha Relief Expedition and they were used in various other colonial expeditions, but usually just a couple of them. Weren't shotguns not used by most militaries until WW1?

Japanese weren't fighting a guerrilla war though (I don't think), so they were using bicycles to move armies along roads.

As for machine guns: By the very end of the 19th century they were *becoming* more common. It's hard to say precisely, there were also the manual machine guns. Generally speaking they became more common as the 19th century progressed, but nothing like in WW1 (and then many armies felt they lacked enough of them).

Broken Crown
2015-06-18, 11:41 PM
Weren't machine guns unusual and expensive during the late 19th century? I know one was brought on the Emin Pasha Relief Expedition and they were used in various other colonial expeditions, but usually just a couple of them.

Depends on what part of the late 19th century we're talking about. The Maxim gun was first deployed in the mid-1880's and was a big improvement in the reliability and portability of machine guns. (It still wasn't very portable, but it was an improvement.) They took a while to be widely adopted, partly due to conservatism, and partly because it took a while for armies to figure out how to really use them effectively. Once those issues were resolved, they didn't stay rare for long.


Weren't shotguns not used by most militaries until WW1?

Most militaries weren't doing much jungle fighting, house-to-house fighting, or other very close range combat in the late 19th century, so there was little perceived need for such a weapon. Also, the first pump shotgun wasn't invented until 1893. However, if the need had existed, there's no reason a comparable weapon couldn't have been developed.


That idea of floating naval mines down the river is brilliant. Do you know of any cases where that tactic was used in real life?

It doesn't seem to have been done often, as it isn't a very efficient or effective way to deploy mines:

- Unless your mine has reasonably good depth-keeping gear, it will have to float on the surface, where your enemies can easily see it and destroy it.

- Rivers are easily blocked by anti-torpedo nets.

- If you're air-dropping your mines, the river is narrower and shallower upriver, so it's a more difficult target. If you're air-dropping your mines to guerrilla troops, it's worth noting that a naval mine big enough to significantly damage a 19th century battleship is not easily man-portable.

Since we're air-dropping things anyway, it was much more common to drop the mines in harbour entrances and other high traffic areas, and let the enemy's ships come to you.

Brother Oni
2015-06-19, 03:48 AM
Can you fight while lying down? With a gun, you can shoot from a prone position. But what about with a spear?

More specifically, would mermaids on land have any combat capabilities, crawling about? Or would they be helpless? I figure the latter, but Knight of Lodis makes me curious.

You can fight to a degree, but anybody standing has a significant advantage. If they have a spear or similar long range weapon, you're basically at their mercy.
Spears and other long weapons would be barely usable while prone (try it for yourself), while other melee weapons would be best served while fighting on your back..

Sidestepping the question of your mermaids being able to support themselves out of water, they would probably be best able to fight on land while using crossbows (shooting while prone, then flipping over on their backs to span).

Benthesquid
2015-06-19, 10:28 PM
Not exactly a weapon or armor question but; gladiators, before going into battle, were traditionally given a banquet, the coena libera, which doubled as a last party before their possible death and a chance for the public to see the gladiators up close before they placed bets on the next day's battle.

I'm wondering if anyone knows anything regarding the sort of food traditionally served at these 'last meals.'

MrZJunior
2015-06-20, 08:55 AM
They took a while to be widely adopted, partly due to conservatism, and partly because it took a while for armies to figure out how to really use them effectively. Once those issues were resolved, they didn't stay rare for long.

Wasn't this the period that machine guns were wheeled around on little artillery carriages like field guns? Not terribly practical for the jungle. When did the tripod come into general use?


The first pump shotgun wasn't invented until 1893. However, if the need had existed, there's no reason a comparable weapon couldn't have been developed.

According to Wikipedia the British Army in India was still using the Snider Enfield rifle in the early 20th century as a shotgun. It would be used to shoot at tribesmen who tried to sneak into camp at night to steal rifles. https://en.wikipedia.org/wiki/Snider%E2%80%93Enfield

If shotguns are that vital to a war in the jungle you could probably just equip them with civilian hunting weapons.

As to dropping the mines in the river, that is clever and inventive enough that it ought to work in an RPG, regardless of real world practicality. Even if it is not an effective means of damaging the enemy navy it could still be useful. Mining the rivers denies your enemies an extra way for them to move troops, supplies, and heavy equipment into the forested interior. If you just need to damage river boats the mines can be much smaller and more easily portable.

Would mountain artillery or pack artillery be a practical thing to ship? I don't think the gun itself would be that difficult, the difficulty comes in keeping it supplied.

Broken Crown
2015-06-20, 10:10 AM
Wasn't this the period that machine guns were wheeled around on little artillery carriages like field guns? Not terribly practical for the jungle. When did the tripod come into general use?

Wikipedia (https://en.wikipedia.org/wiki/Maxim_gun#/media/File:Maxim_machine_gun_Megapixie.jpg) has a photo of an air-cooled, tripod-mounted Maxim gun dating from 1895, which looks a lot more portable than the carriage-mounted, water-cooled versions. You'd still want two or three people to carry it around (not including ammunition), but it wouldn't be any worse than a modern heavy machine gun.


If shotguns are that vital to a war in the jungle you could probably just equip them with civilian hunting weapons.

Most civilian shotguns from that period were very long, heavy, and fired only two shots – really unsuitable for close range combat.


As to dropping the mines in the river, that is clever and inventive enough that it ought to work in an RPG, regardless of real world practicality. Even if it is not an effective means of damaging the enemy navy it could still be useful. Mining the rivers denies your enemies an extra way for them to move troops, supplies, and heavy equipment into the forested interior. If you just need to damage river boats the mines can be much smaller and more easily portable.

The trouble is, floating mines down a river doesn't really deny the enemy the use of the river, because free-floating mines won't stay in the river; they'll either get stuck somewhere (and possibly explode, depending on the detonator) or float out to sea; either way, the river is clear again after a very short time. And as I said before, once the enemy knows you're doing it, a net spread across the river can block your mines, and be moved out of the way if the enemy needs to go upriver for some reason.


Would mountain artillery or pack artillery be a practical thing to ship? I don't think the gun itself would be that difficult, the difficulty comes in keeping it supplied.

It should be practical. Mountain guns (https://en.wikipedia.org/wiki/RML_7_pounder_Mountain_Gun) were used in the late 19th century. Moving them would be more difficult than machine guns, but not impossible if you have pack animals. Ammunition weighs extra, of course. As Gnoman suggested above, it would be a good idea for a guerrilla force to use guns chambered to take the enemy's ammunition: It makes resupply much easier.

Mike_G
2015-06-20, 06:01 PM
For jungle combat, remember that engagement distance will be very short, visibility will be very short. Long range weapons won't be as useful as they would be in the open fields of the desert or the plains of Africa or No Man's Land. The Maxim gun or Gatling guns won't be as useful as they were in many historical conflicts.

Short, light, quick weapons. Carbines, shotguns, and sub machineguns are good for close range, low visibility fights. Bayonets are a must for the same reason. It's entirely probable to come upon an enemy at melee range before you see him.

And this is coming from a Marine who worships at the altar of long range accurate fire.

The other thing that would be very helpful would be water purification tablets. There's plenty of water in the jungle, but it's teeming with bacteria and parasites. Drinking raw water will have half your army down with dysentery. Tablets are a lot lighter and more portable than bringing in fresh water. I know troops were using tablets in WWII, not sure how long before that. But if you can fill your canteens from jungle streams, drop in a few halizone tabs and shake it, you're good to go. Other than that, you can strain water through cloth and boil it, but that takes time and requires fires which can give away your position.

Stellar_Magic
2015-06-21, 01:00 AM
Hmm... anyone suddenly think that something like the broomhandle mauser would be popular for this jungle setting?

Anyway, I've been reading up a bit on naval combat and trying to apply some of the information for a science fiction setting... What's the equivalent of being 'sunk' in a space fight? It's not like a starship that's been shot full of holes from railgun rounds and laser blasts is going to sink beneath the waves?

So... aside for catastrophic failures (antimatter containment failure, fuel explosion, etc...) could a spacecraft keep fighting so long as it had power, munitions, and crew to man it (or even keep fighting without the crew if it's somewhat autonomous)?

Carl
2015-06-21, 01:58 AM
So... aside for catastrophic failures (antimatter containment failure, fuel explosion, etc...) could a spacecraft keep fighting so long as it had power, munitions, and crew to man it (or even keep fighting without the crew if it's somewhat autonomous)?

Pretty much. It's worth noting however that very few laser designs will be especially effective at extended ranges due to power limits and contact area limits.

Realistically any future space combat without shields will be fought using contact nuke's, and well you only get hit once by those generally speaking.

SowZ
2015-06-21, 02:09 AM
It's certainly possible to do so, I don't think the tolerances of bronze are so wildly different to steel that it would be unsuitable for the job. Though it would be about 10% heavier, which is not inconsiderable for the amount of metal you'd be wearing.

I don't know, though, whether ancient armour-smiths had the knowledge of creating articulating joints, or whether they'd have considered it worthwhile.

They could certainly create articulating joints. Plate armour likely reached its zenith at around the time of King Henry the VIII and the craft was maintained for a while before being lost. Not entirely, mind you, but plate smithing never again reached its peak. This is true even today. Despite all our fancy tools, there isn't an armorer alive who has crafted a suit as sophisticated as the best stuff produced during the 16th century.


Pretty much. It's worth noting however that very few laser designs will be especially effective at extended ranges due to power limits and contact area limits.

Realistically any future space combat without shields will be fought using contact nuke's, and well you only get hit once by those generally speaking.

Of course that has it's own difficulties two spaceships might have distances between them that dwarf any distance which vehicles could engage each other on Earth. This could give the enemy time to detonate the nuke from a safe distance, maybe even dodge it or deflect it with flak of some kind.

Space battles would probably be closer to submarine combat than any sort of aerial equivalent.

Spiryt
2015-06-21, 03:29 AM
They could certainly create articulating joints. Plate armour likely reached its zenith at around the time of King Henry the VIII and the craft was maintained for a while before being lost. Not entirely, mind you, but plate smithing never again reached its peak. This is true even today. Despite all our fancy tools, there isn't an armorer alive who has crafted a suit as sophisticated as the best stuff produced during the 16th century.




I'm not sure what you're saying here, TBH.

If anything your post hints that articulating joints was indeed sophisticated craft that bloomed in some European centers at specific time.

It's very possible that most ancient smith wouldn't really be capable of doing so, completely different technical context, different ideas about armour etc.

Of course, there were plenty of pretty sophisticated bronze machines in antique Mediterranean, including elegantly hinged helmets etc. but it's hard to say if it all could automatically translate to articulated joints seamlessly.

Brother Oni
2015-06-21, 07:05 AM
Anyway, I've been reading up a bit on naval combat and trying to apply some of the information for a science fiction setting... What's the equivalent of being 'sunk' in a space fight? It's not like a starship that's been shot full of holes from railgun rounds and laser blasts is going to sink beneath the waves?

I'd say complete loss of life support and atmosphere containment. It's hard to keep operating your post when you're sucking vacuum (especially since unconsciousness results within 15-25 seconds).

You can argue that all your crew will be wearing space suits, but given the critical nature of life support, if that's gone pretty much everything else, like hull integrity and power routing, would be gone as well.

MrZJunior
2015-06-21, 08:55 AM
Wikipedia (https://en.wikipedia.org/wiki/Maxim_gun#/media/File:Maxim_machine_gun_Megapixie.jpg) has a photo of an air-cooled, tripod-mounted Maxim gun dating from 1895, which looks a lot more portable than the carriage-mounted, water-cooled versions. You'd still want two or three people to carry it around (not including ammunition), but it wouldn't be any worse than a modern heavy machine gun.

Fair enough.


The trouble is, floating mines down a river doesn't really deny the enemy the use of the river, because free-floating mines won't stay in the river; they'll either get stuck somewhere (and possibly explode, depending on the detonator) or float out to sea; either way, the river is clear again after a very short time. And as I said before, once the enemy knows you're doing it, a net spread across the river can block your mines, and be moved out of the way if the enemy needs to go upriver for some reason.

I meant tether the mines in the river like you would with normal naval mines. My apologies, I should have been more clear.


For jungle combat, remember that engagement distance will be very short, visibility will be very short. Long range weapons won't be as useful as they would be in the open fields of the desert or the plains of Africa or No Man's Land. The Maxim gun or Gatling guns won't be as useful as they were in many historical conflicts.

I think the point of the machine gun is to use it as a defensive weapon or when attacking enemy positions outside the jungle.

The Maxim Gun will have some advantages in the jungle, like the ability to shoot through the foliage.


Short, light, quick weapons. Carbines, shotguns, and sub machineguns are good for close range, low visibility fights. Bayonets are a must for the same reason. It's entirely probable to come upon an enemy at melee range before you see him.

Bayonets are a good idea, but they were pretty standard in the 19th century. As for sub machineguns and the type of carbines you are probably thinking of, those are about 20 to 30 years off in tech level. Hypothetically you could build such a thing with the technology available in universe, but there hasn't been a need for such a thing yet.

I have no idea what people did to purify water back in the 19th century. Wikipedia has some references to various filtration systems and chlorination going back that far, but I don't know how portable it would be.


Hmm... anyone suddenly think that something like the broomhandle mauser would be popular for this jungle setting?

That would actually work really well. Some of those old Mauser pistols had a full auto setting, but that might have been a later innovation.

Mike_G
2015-06-21, 09:17 AM
Fair enough.



I think the point of the machine gun is to use it as a defensive weapon or when attacking enemy positions outside the jungle.

The Maxim Gun will have some advantages in the jungle, like the ability to shoot through the foliage.



Still big and hard to move, and you won't see the enemy until you are right on top of them. A few to defend river crossings or trails, but machine guns work best with prepared fields of fire.

Maybe something like a Lewis gun. It's half the weight of a Vickers, took a 97 round drum magazine and was reliable by all accounts. The BAR was developed in the very early 1900s, an that would be a good choice.


Fair enough.

Bayonets are a good idea, but they were pretty standard in the 19th century. As for sub machineguns and the type of carbines you are probably thinking of, those are about 20 to 30 years off in tech level. Hypothetically you could build such a thing with the technology available in universe, but there hasn't been a need for such a thing yet.


Plenty of repeating carbines in the latter half of the 19th century. A Henry rifle or a repeating Spencer would be a good weapon. Any of the short repeaters favored by the cavalry would be good. An M1 carbine would be great, but that's a bit far off.

The lower powered rounds aren't a big disadvantage, since you won' be shooting at long range, and the larger magazine and lower weight are good.

The Thompson was developed for WWI, but too late to see service. It's a terrific jungle gun.


Fair enough.

I have no idea what people did to purify water back in the 19th century. Wikipedia has some references to various filtration systems and chlorination going back that far, but I don't know how portable it would be.


You can always boil water, but that's a process. Worth setting up in base camps so the troops can set out with canteens full of safe water.

Broken Crown
2015-06-21, 11:29 AM
Anyway, I've been reading up a bit on naval combat and trying to apply some of the information for a science fiction setting... What's the equivalent of being 'sunk' in a space fight? It's not like a starship that's been shot full of holes from railgun rounds and laser blasts is going to sink beneath the waves?

So... aside for catastrophic failures (antimatter containment failure, fuel explosion, etc...) could a spacecraft keep fighting so long as it had power, munitions, and crew to man it (or even keep fighting without the crew if it's somewhat autonomous)?

One issue that spaceships have to deal with is heat dissipation, since vacuum is essentially a perfect insulator. Anything that uses energy on your ship will create some amount of waste heat (as will being hit by directed energy weapons like lasers). If your ship builds up heat faster than it can get rid of it, the ship will cook. This is especially likely to happen in combat, since your ship's radiators are likely to be large and vulnerable to enemy fire, and will probably be the first things to be wrecked unless you fold them up and put them away before combat starts. So it's quite possible for a spaceship to be rendered incapable of fighting even before it has suffered significant amounts of damage to its structure or systems.

I know of one science fiction setting where a spaceship spreading its radiators was equivalent to a sailing ship striking its colours: a universally recognized signal of surrender, indicating that you couldn't fight anymore.

Storm Bringer
2015-06-21, 12:07 PM
Sorry it took me so long to reply.

The players will be a group of mercenaries and adventures hired to improve the military of a small tropical island nation. The nation is invaded by Country A, an up and coming new power eager to carve themselves out their own sphere of influence. They are opposed by Country B an established empire emerging from a period of instability and looking to reestablish their influence on the international stage. Country A is an island nation and has the finest navy in the world, including their new dreadnought class of ships, the most powerful and technologically advanced warships in the world. Country B is more of a land power, with a large army and an excellent air force, including the most advanced zeppelins in the world able to fly faster, higher, and farther than any other airships in the world. County A will invade and occupy the coastline of the tropical nation and some of the outlying islands but won't be able to easily penetrate the heavily forested interior. Country B, interested in humiliating their rival, will use their air force to airlift supplies to the guerrillas in the jungle. It would be pretty obvious what they are doing, so probably little need to try for plausible deniability.

Technology wise it is slightly steampunk. They have zeppelins, dreadnoughts, and steam powered cars, but no radio, mechs, or other such things. Electricity is somewhat common.

I hope this helps.

if they have electricity, then theirs no reason they could not have telegraph, which would both provide a good target for the players (attack the telegraph station where the cable that comes in form Country A, and thus either tap thier comms or destroy their best means of controlling their troops), and a possible form of aid. A ground laid or tree strung telegraph to link several guerrillacamps together, giving them a faster comms loop (for example, a main base in a river valley could be linked to a OP down the river to warn of approaching attackers, or to a firebase set on a overlooking hill to bring down fire support on a attacker).

also, don't overlook the recon value the airships could provide, even in just a passive role ("while i was doing my normal trade run out over the Windhorn Reach, i saw a convoy of six troopships, three frigates and one of those new cost bombardment monitors they have, heading south. i expect they will reach Port Windhorn in about two or three days...")


country A will try to push roads into the jungle, establishing patrol bases to create a buffer zone where the guerrillas have trouble operating. being a naval power, they will likey try to use the rivers to push inland as well. the key to guerrilla combat is "hit and RUN". its not about standup fights. if you can take them in a standup fight, you don't need to use guerrilla tactics. ambush, cause damage and casulties, get the hell out of dodge and move to the next ambush point. if they chase you, great, lead them into another ambush.

JustSomeGuy
2015-06-21, 12:31 PM
Wouldn't a spaceship unable to propel itself just kind of spin-float-orbit off in some random direction depending on what last hit it?

Carl
2015-06-21, 01:55 PM
@Broken Crown: Not true, any form of Laser resistant armour that relies on a combination of Active cooling and emission to dissipate incoming energy is automatically also a good radiator. And that's assuming you aren't willing to stretch things a little vis a vis realism. Stretch a little so you can get a material, (Gas solid, or liquid), that undergoes lasing at a controllable input IR range and you can radiate the thermal energy as a directed laser. Purely theoretical and i haven;t heard of any IRL material that would do the job, but it's not explicitly forbidden by science so a suitable material would make even fairly high radiant energies possible.

@JustSomeGuy: Depends on the prior vector, depends on what hits it. Generally though anything capable of significantly altering a ships vestor will just shatter it into a billion pieces.

Broken Crown
2015-06-21, 03:18 PM
@Broken Crown: Not true, any form of Laser resistant armour that relies on a combination of Active cooling and emission to dissipate incoming energy is automatically also a good radiator.

Which part is not true? That a spaceship's systems will generate heat? That a ship, on being exposed to external sources of energy (such as lasers), will absorb heat? That heat will need to be radiated away or the temperature of the ship will increase?

You're correct if you're saying that radiators don't strictly need to be fragile, but to be efficient, they need a large area:mass ratio, which means they work best if they're big and flimsy. You could also use vaporization to create an efficient yet compact radiator, but that would only work until you ran out of coolant.

By "active" cooling, do you mean refrigeration? Because that isn't 100% efficient, and therefore will cause a net gain in the amount of heat produced, and therefore will require even larger radiators. I'm not sure what you mean by "emission"; everything emits thermal energy, so everything works as a radiator to some degree if it's warmer than its surroundings; it's just that some designs are more efficient than others.


And that's assuming you aren't willing to stretch things a little vis a vis realism. Stretch a little so you can get a material, (Gas solid, or liquid), that undergoes lasing at a controllable input IR range and you can radiate the thermal energy as a directed laser. Purely theoretical and i haven;t heard of any IRL material that would do the job, but it's not explicitly forbidden by science so a suitable material would make even fairly high radiant energies possible.

I don't know enough about lasers to comment on this, other than to say that I've heard similar schemes discussed elsewhere, and the consensus among the people who actually explain the thermodynamics involved is that it violates the 2nd Law. (Lasers can get rid of the energy, but not the entropy.)

Gnoman
2015-06-21, 03:48 PM
Wouldn't a spaceship unable to propel itself just kind of spin-float-orbit off in some random direction depending on what last hit it?

It's complicated. If the drive on an intermittent-boost ship simply failed to ignite, it would merely continue on the same vector and velocity it was already on. However, once you get powerful weaponry (nukes (which would NOT be a guaranteed one-shot kill, vacuum removes most of their power), high-output lasers vaporizing part of the hull, conventional cannon, etc.), the effects of a failing constant-boost drive, internal explosions of fuel or munitions, or the other effects of combat, the ship will be subjected to a very large number of thrust vectors, so predicting the course and speed becomes much more difficult.

Telok
2015-06-21, 04:52 PM
I don't know enough about lasers to comment on this, other than to say that I've heard similar schemes discussed elsewhere, and the consensus among the people who actually explain the thermodynamics involved is that it violates the 2nd Law. (Lasers can get rid of the energy, but not the entropy.)

Oh man is that a violation of thermodynamics. Look, it's like this:
1) Inefficency turns some of your energy to heat.
2) Nothing is prefectly efficent.

What you want to do is to take a thermocouple or turbine or something and turn heat into energy. Take that energy and feed it into a laser system. Then have the laser shoot out into space. Which is a nice idea but takes lots of mass and turns out to be less efficent than a radiator.

With the magic material that converts IR radiation straight to IR laser radiation you're just doing what a radiator does anyways, with an extra step of absorbing and re-emitting. Which won't be perfectly efficent. Which will convert some of it into heat. Heating the laser and it's mount, contributing to the heat load that you need to get rid of.

There may also be some confusion between heat and IR radiation here. When I say heat I mean the thermal energy of matter which can be emitted as IR radiation. On a spaceship it's the heat that's an issue. Your lasers get hot, your hull heats up, your reactors that power the ship get hotter, all of which is bad. The emission of IR radiation is governed by the properties of a material and the surface area. That magic material would be better used as or in a radiator than to make lasers because you'd get more surface area from the radiator.

So compare the magic laser spaceship with a normal radiator ship. The magic laser ship has a feedback loop in it's heat dissipation scheme that takes the IR radiation going out and puts some of it back into the ship as heat that the magic laser can't dissipate. The radiator doesn't have that.

Carl
2015-06-21, 05:04 PM
@Broken Crown: By active cooling i mean active cooling. The difference between a radiator and actively cooled laser armour is that one has hot coolant that needs to get rid of heat and the other has cold coolant waiting to absorb heat, with rapid pump rates and some creative piping you can switch any hull area piece from one to the other in a second or less, (really high flow systems flow the coolant at insane rates of speed, so as long as the valves are close enough to the hull the switching will be super rapid).


@Telok:


Heating the laser and it's mount, contributing to the heat load that you need to get rid of.

This violates conservation of energy. If your sole input is waste heat energy you cannot get more heat out than your waste heat and with even a modest laser efficiency will get quite a bit less than this. the problem is that AFAIK there is no known, material that will lase at the frequencies of IR radiation emission spectra that a good radiator will work at, at least not at any efficiency level. hat's the slightly unrealistic part as we don't know of a suitable material, even if science doesn't currently say one cannot exist.


It's complicated. If the drive on an intermittent-boost ship simply failed to ignite, it would merely continue on the same vector and velocity it was already on. However, once you get powerful weaponry (nukes (which would NOT be a guaranteed one-shot kill, vacuum removes most of their power), high-output lasers vaporizing part of the hull, conventional cannon, etc.), the effects of a failing constant-boost drive, internal explosions of fuel or munitions, or the other effects of combat, the ship will be subjected to a very large number of thrust vectors, so predicting the course and speed becomes much more difficult.

Again our violating conservation of energy there Gnoman. A nuke in contact with the hull for anything less that the most insanely over sized designs is a one shot kill. Vacuum doesn't make the energy go away, it just means it doesn't radiate over distance very well. But a 1 megaton nuke still produces enough thermal energy over a 50% hemesphere coverage to vaporise over a 100,000 tons of steel.


it's just that at a range of say 10KM a 1KM long by 1KM wide target will only get hit by 0.08% of the energy.

SowZ
2015-06-21, 05:28 PM
I'm not sure what you're saying here, TBH.

If anything your post hints that articulating joints was indeed sophisticated craft that bloomed in some European centers at specific time.

It's very possible that most ancient smith wouldn't really be capable of doing so, completely different technical context, different ideas about armour etc.

Of course, there were plenty of pretty sophisticated bronze machines in antique Mediterranean, including elegantly hinged helmets etc. but it's hard to say if it all could automatically translate to articulated joints seamlessly.

I might have misunderstood what you were saying. My point was just that articulated joints was something armorers did eventually accomplish, (as well or better than current blacksmiths,) not that bronze age smiths could do it.

Kiero
2015-06-21, 07:04 PM
They could certainly create articulating joints. Plate armour likely reached its zenith at around the time of King Henry the VIII and the craft was maintained for a while before being lost. Not entirely, mind you, but plate smithing never again reached its peak. This is true even today. Despite all our fancy tools, there isn't an armorer alive who has crafted a suit as sophisticated as the best stuff produced during the 16th century.

Ancient is not medieval. We were talking about whether armourers working bronze in 4th-1st century BC, or thereabouts, could make articulated joints. Not in the 16th century AD.

SowZ
2015-06-21, 08:02 PM
Ancient is not medieval. We were talking about whether armourers working bronze in 4th-1st century BC, or thereabouts, could make articulated joints. Not in the 16th century AD.

Sure, I thought the statement was more general and talking about if armour ever reached that point pre-modern era. I don't know enough about the pre-medieval era to say.

MrZJunior
2015-06-21, 08:53 PM
Maybe something like a Lewis gun. It's half the weight of a Vickers, took a 97 round drum magazine and was reliable by all accounts. The BAR was developed in the very early 1900s, an that would be a good choice.

Well, when it's the only rapid fire weapon you have, sometimes you have to take what you can get. Though I suppose it would make sense to ask Broken Crown why they feel machine guns are so vital.


Plenty of repeating carbines in the latter half of the 19th century. A Henry rifle or a repeating Spencer would be a good weapon. Any of the short repeaters favored by the cavalry would be good.

That is an excellent point, I hadn't even considered what civilian weapons would be available. I tend to get caught up in military equipment that I forget that there are weapons mainly designed for the civilian market.


if they have electricity, then theirs no reason they could not have telegraph, which would both provide a good target for the players (attack the telegraph station where the cable that comes in form Country A, and thus either tap thier comms or destroy their best means of controlling their troops), and a possible form of aid. A ground laid or tree strung telegraph to link several guerrillacamps together, giving them a faster comms loop (for example, a main base in a river valley could be linked to a OP down the river to warn of approaching attackers, or to a firebase set on a overlooking hill to bring down fire support on a attacker).


Makes sense, they could also do the same thing with telephones if they can get the parts.


also, don't overlook the recon value the airships could provide, even in just a passive role ("while i was doing my normal trade run out over the Windhorn Reach, i saw a convoy of six troopships, three frigates and one of those new cost bombardment monitors they have, heading south. i expect they will reach Port Windhorn in about two or three days...")

I like the lateral thinking. Country B wouldn't like to be brought directly into the war, but passive recon is a great idea.

Gnoman
2015-06-21, 09:26 PM
Again our violating conservation of energy there Gnoman. A nuke in contact with the hull for anything less that the most insanely over sized designs is a one shot kill. Vacuum doesn't make the energy go away, it just means it doesn't radiate over distance very well. But a 1 megaton nuke still produces enough thermal energy over a 50% hemesphere coverage to vaporise over a 100,000 tons of steel.


it's just that at a range of say 10KM a 1KM long by 1KM wide target will only get hit by 0.08% of the energy.

The energy produced by a nuke initially goes out in electromagnetic form - this is why a high-altitude burst is a devastating attack against communications and electronics, and also why the very first indicator of a nuclear detonation is an absolutely pure white light. In an atmosphere, this energy gets dumped into the air, which can't absorb it - resulting in a massive thermal pulse, which results in a fireball and a deadly shock wave. In space, there is no atmosphere, and all the energy remains electromagnetic except the tiny portion that goes into whatever it's hitting. Metal can handle much more EM energy than air can, so a large portion of what goes into the hull will not be turned into heat. Much of what IS turned into heat will go into shattering a few individual section of armor or hull plate, welding bits of hull together, etc. The remainder will "merely" be extra thermal energy for the ship to deal with, and any ship will have to be able to dump heat in massive quantities just to function, a little more won't be catastrophic. (note: very large nuclear reactions with the gravity to maintain their shape and enough fuel to maintain their reaction for extended periods operate by slightly different rules, thus the existence of stars does not contradict this).

Even in atmosphere, where a nuke is thousands of times more powerful, we have structures of metal and stone that have survived direct hits from nuclear bombs,

fusilier
2015-06-21, 10:32 PM
Maybe something like a Lewis gun. It's half the weight of a Vickers, took a 97 round drum magazine and was reliable by all accounts. The BAR was developed in the very early 1900s, an that would be a good choice.

The earliest, successful, light machine gun I can think of is the 1902 Madsen. An oft overlooked gun, it was made a bewildering array of calibers and used for a surprisingly long time.

Given the general description of the technology level, probably a "repeater" carbine of some sort is the best bet. Maybe some semi-auto pistols too?

Broken Crown
2015-06-21, 10:36 PM
@Broken Crown: By active cooling i mean active cooling.

Thank you for that clarification. That was very helpful.

Let's see what Wikipedia (https://en.wikipedia.org/wiki/Active_cooling) has to say on the subject:


"An active cooling system is one that involves the use of energy to cool something, as opposed to passive cooling that uses no energy.

Such systems circulate a coolant to transfer heat from one place to another. The coolant is either a gas, such as in air cooling of computers, or a liquid such as in a car engine. In the latter case, liquid is pumped to transfer heat from the engine to the radiator, which in turn is cooled by passing air over it.

Other active cooling systems make use of a refrigeration cycle."

Okay, that sounds similar to what you've written here:


The difference between a radiator and actively cooled laser armour is that one has hot coolant that needs to get rid of heat and the other has cold coolant waiting to absorb heat, with rapid pump rates and some creative piping you can switch any hull area piece from one to the other in a second or less, (really high flow systems flow the coolant at insane rates of speed, so as long as the valves are close enough to the hull the switching will be super rapid).

So, you propose using a system that will consume large amounts of energy (meaning more waste heat will be generated) and will have a large mass (meaning the ship will need to consume more energy in order to maneuver) and a high degree of mechanical complexity, and, because it will need to cover the ship's entire surface in order to function effectively, will be incredibly susceptible to battle damage. And it will need radiators anyway, because it doesn't matter how "insane" the flow rate is, the coolant still needs to get rid of the heat at some stage in the cycle.

So, what's the potential advantage to doing it this way?

(I know, but I'm curious whether you know. The numbers would vary a lot from case to case, but I suspect that most of the time it wouldn't be worth it.)


Heating the laser and it's mount, contributing to the heat load that you need to get rid of.

This violates conservation of energy. If your sole input is waste heat energy you cannot get more heat out than your waste heat and with even a modest laser efficiency will get quite a bit less than this. the problem is that AFAIK there is no known, material that will lase at the frequencies of IR radiation emission spectra that a good radiator will work at, at least not at any efficiency level. hat's the slightly unrealistic part as we don't know of a suitable material, even if science doesn't currently say one cannot exist.

Is that the problem that you see? No problem with violations of the Second Law of Thermodynamics come to mind? No problem with the fact that Telok was talking about your own proposal, and explaining why it wouldn't be as effective as a regular radiator, assuming the laws of thermodynamics allowed it to work at all?

(And, assuming you could make a heat-powered laser, it doesn't actually violate Conservation of Energy, as long as the amount of energy you're trying to get rid of is smaller than the amount of energy available. As I said, it's a 2nd Law problem, not 1st Law.)

Also, since this is the Real World Weapon, Armour or Tactics thread, shall we restrict our discussion to technology that is known to be physically possible, rather than not known to be physically impossible?

Broken Crown
2015-06-21, 10:48 PM
Well, when it's the only rapid fire weapon you have, sometimes you have to take what you can get. Though I suppose it would make sense to ask Broken Crown why they feel machine guns are so vital.

Sorry; I missed this.

Wasn't it Gnoman who suggested machine guns as important equipment for this mission (top of page 3)? I wouldn't have given them a high priority myself, until lighter ones were developed; I was just describing what was available in the late 19th century.

Stellar_Magic
2015-06-21, 11:35 PM
The energy produced by a nuke initially goes out in electromagnetic form - this is why a high-altitude burst is a devastating attack against communications and electronics, and also why the very first indicator of a nuclear detonation is an absolutely pure white light. In an atmosphere, this energy gets dumped into the air, which can't absorb it - resulting in a massive thermal pulse, which results in a fireball and a deadly shock wave. In space, there is no atmosphere, and all the energy remains electromagnetic except the tiny portion that goes into whatever it's hitting. Metal can handle much more EM energy than air can, so a large portion of what goes into the hull will not be turned into heat. Much of what IS turned into heat will go into shattering a few individual section of armor or hull plate, welding bits of hull together, etc. The remainder will "merely" be extra thermal energy for the ship to deal with, and any ship will have to be able to dump heat in massive quantities just to function, a little more won't be catastrophic. (note: very large nuclear reactions with the gravity to maintain their shape and enough fuel to maintain their reaction for extended periods operate by slightly different rules, thus the existence of stars does not contradict this).

Even in atmosphere, where a nuke is thousands of times more powerful, we have structures of metal and stone that have survived direct hits from nuclear bombs,

This is kind of similar to my thoughts on nuclear weapons in space. They are effective, but not as effective as other forms of weaponry would be. A direct hit from a nuke could destroy a ship... but the moment you start moving the detonation from the hull the lower the weapon's effectiveness as the weapon's energy delivered to the target would fall off by the inverse of the square (inverse square law... it's important).

So a 1 mT bomb detonating at 10 meters delivers the same effective energy on target as a 10 kT bomb on impact, and the 1 mT bomb detonating at 100 meters delivers the same effective amount of energy on target as a mere 100 tons of TnT detonating on the ship.

At 1,000 meters? The effective damage would be around the same as hitting the ship with 1 ton of explosives.

A flak shell or beehive round from a railgun would be lethal at a larger detonation distance, as each individual bee-hive flechette or piece of shrapnel isn't loosing velocity or impact energy across the distance from detonation point to impact. Still, their individual destructive potential would be lower then a nuclear device.

Moreover, any ship designed for reentry will be very resistance to the thermal effects of a nuclear device... and most spaceships would be designed to offer as much radiation protection as possible since that's a concern just from being in space.

The thermal load is a real issue, but there are ways to stretch that. One idea I had was that a ship that used say... water as a propellant or fuel component (AM/M reaction), might be able to pump it into the coolant system... after venting their normal coolant in an emergency or combat situation.

When I imagine a combat starship I tend to give them large 'wing' radiators that line up with the ship's axis... That way they're harder to hit if say most of the weapons are mounted axially (which seems to be the way to do it for a realistic space craft), and you point your ship at the enemy.

Telok
2015-06-22, 01:05 AM
@Telok:
This violates conservation of energy. If your sole input is waste heat energy you cannot get more heat out than your waste heat and with even a modest laser efficiency will get quite a bit less than this. the problem is that AFAIK there is no known, material that will lase at the frequencies of IR radiation emission spectra that a good radiator will work at, at least not at any efficiency level. hat's the slightly unrealistic part as we don't know of a suitable material, even if science doesn't currently say one cannot exist.

Ok, I think I misunderstood the magic laser you were proposing. You want something that absorbs thermal energy from adjacent mass of higher temp and radiates IR. The problem is that every material does that and the way physics works is that the radiating is porportional to surface area. Your magic material emitting the energy as coherent IR instead of radiant IR doesn't change anything. The material will still emit IR radiation in porportion to it's material properties and surface area.

As far as human knowledge extends the laws of thremodynamics are an iron clad bitch.

Gnoman
2015-06-22, 01:46 AM
Well, when it's the only rapid fire weapon you have, sometimes you have to take what you can get. Though I suppose it would make sense to ask Broken Crown why they feel machine guns are so vital.

One machine gun has the firepower of a thousand riflemen. Are they cumbersome to move around? Of course. Is it worth it? Without question. Even the heavy Gatling and Maxim guns (roughly 60 pounds each) were used to great effect in jungle fighting, and lighter weapons such as the 1895 Colt-Browning (27 pounds with tripod only) were even better at it.

The key to remember that the sort of fighting you're describing will be asymmetric. They cannot fight a stand-up battle almost by definition - if they had the ability to do that, they wouldn't be fighting as guerrillas. This limits their ability to inflict decisive damage to an enemy (put simply, they can't drive the enemy off their land, they just have to keep the country a running sore in the hope that either the cost gets high enough for the civilian leadership or that the occupier gets involved in a big war somewhere else that will weaken the garrison enough to be destroyed), but also simplifies their operational realities enormously. Insurgents will either attack from ambush with the intention of completely exterminating whatever forces they're in contact with, or mass slowly while moving under cover of darkness in order to attack a given base or outpost with overwhelming force. It doesn't matter if it takes a week to slowly move a gun through the jungle to ambush a regular supply shipment, or that you need a month's notice before you can wipe an outlying fort from the face of the earth. The convoy gets hit or the fort wiped out in the end, and that is what matters.

Meanwhile, as long as YOU have some machine guns at your camp, either you'll have a great deal of warning before they move up the necessary weapons to deal with them (allowing you to evacuate, set up an ambush, or preferably both), or you can slaughter whatever gets sent at you.

Brother Oni
2015-06-22, 02:08 AM
One machine gun has the firepower of a thousand riflemen. Are they cumbersome to move around? Of course. Is it worth it? Without question. Even the heavy Gatling and Maxim guns (roughly 60 pounds each) were used to great effect in jungle fighting, and lighter weapons such as the 1895 Colt-Browning (27 pounds with tripod only) were even better at it.


Even without machine guns, you can put so much lead down range that any infiltrators would be flushed out.

German infantry during WW1 came under what they thought was machine gun fire, was in fact squads of British riflemen on a mad minute (https://en.wikipedia.org/wiki/Mad_minute). That said, if you can get squads of well trained riflemen to converge on a target, it's not really asymmetric warfare anymore, just standard warfare.

Carl
2015-06-22, 02:34 AM
@Gnoman: I know all that except you seem to have forgotten that a Nuke still outputs a significant percentage, (about 40% off the top of my head), as pure IR even in vacuum. My figure was based on that value. It's by no means 100% accurate but it demonstrates just how much overkill a nuke would be unless you built the ship really, really, really big.

@Broken Crown: How about actually looking how active cooling systems actually work before spouting nonsense. the only heat generated would be by the pumps. It's like saying your car radiator's cooling generate more heat than the engine it's cooling. It doesn't, in fact aside from the negligible heat from the pump it generate's nothing. Your confusing forced cooling with thermocouples and the like with what i'm talking about.

@Telok: Except such a substance would almost certainly absorb IR from the laser walls faster than space will. And because it's internal, (thus 3d and any shape you want), and the output is coherent you can get considerably more output energy per square meter of hull area used as the emission points. It might be easier to think of it as an internal liquid heatsink that is able to radiate a percentage of it's energy per second as coherent light.

Gnoman
2015-06-22, 02:39 AM
Even without machine guns, you can put so much lead down range that any infiltrators would be flushed out.
That said, if you can get squads of well trained riflemen to converge on a target, it's not really asymmetric warfare anymore, just standard warfare.

Precisely. That sort of firepower from bolt-action rifles or lever-action carbines requires you to drill your riflemen to an extremely high quality - something that isn't exactly trivial for a secure nation-state, and all but impossible for one that's been heavily invaded - see the USSR early in WWII, which often had to send vehicles into service straight from the factory (sometimes without even painting them in a less eye-catching color than bare metal!) and at times considered any training at all to be a luxury they simply could not afford.

Telok
2015-06-22, 03:54 AM
It might be easier to think of it as an internal liquid heatsink that is able to radiate a percentage of it's energy per second as coherent light.

Except for the coherent part that's exactly what any other heatsink and radiator system does. It doesn't change the math or the physics. The thermal energy converted to IR light and radiated away obeys the same rules. The emission will be porportional to the exposed surface area doing the emitting.

What you want is something very very good at glowing from thermal energy. You're talking about a substance that can take incoming thermal energy and radiate it as IR light. That's what a heatsink (in space) does. That may be the issue. Current weapons grade lasers are essentially blast furnaces that produce coherent light as a side effect and generally use electricity. Our transformation of heat into electricity then into light is quite inefficent now. Skipping the electricity transformations would increase the efficency but it won't change the rules. For solving the heat dissipation issue of spaceships the coherency of the radiated light isn't important. You have to radiate lots of energy. Lots and lots of energy. That's where the surface area comes in.

Storm_Of_Snow
2015-06-22, 04:42 AM
I'd say complete loss of life support and atmosphere containment. It's hard to keep operating your post when you're sucking vacuum (especially since unconsciousness results within 15-25 seconds).

You can argue that all your crew will be wearing space suits, but given the critical nature of life support, if that's gone pretty much everything else, like hull integrity and power routing, would be gone as well.
Agreed to an extent, but a warship will probably have decompressed most of it's hull spaces to reduce the effects of explosive decompression (medical might be about the one area that's still pressurised, and that's likely deep within the core of the vessel), and most of the crew would probably be wearing a minimum of pressure containment undergarments (on the basis that they may not have time to put space suits on if they're ambushed, but they probably had time to put the undergarments on when they got dressed at the start of their shift) and a breathing mask that covers their full face and ears, connected to the ships life support but also with a short duration personal supply.

So it's whether they can re-establish life support and enough of a sealed hull to maintain the environment before their emergency supplies run out.

Anyway, don't you have something like a minute or so before hypoxia starts to kill you?

Certain fictional universes may have their own issues - for instance, in 40k, combat in the warp is rare, but does happen, and if a ship loses the fields that keep the warp out and can't perform an emergency translation back to real space, wackiness of the Event Horizon/ Hellraiser varieties ensues.

Brother Oni
2015-06-22, 06:52 AM
Agreed to an extent, but a warship will probably have decompressed most of it's hull spaces to reduce the effects of explosive decompression (medical might be about the one area that's still pressurised, and that's likely deep within the core of the vessel), and most of the crew would probably be wearing a minimum of pressure containment undergarments (on the basis that they may not have time to put space suits on if they're ambushed, but they probably had time to put the undergarments on when they got dressed at the start of their shift) and a breathing mask that covers their full face and ears, connected to the ships life support but also with a short duration personal supply.

So it's whether they can re-establish life support and enough of a sealed hull to maintain the environment before their emergency supplies run out.

Anyway, don't you have something like a minute or so before hypoxia starts to kill you?


Indeed but it's the lack of consciousness after about 10-15 seconds that results in your eventual death.

There's a video of an astronaut that developed a leak in his space suit while in a vaccum chamber on Earth and he lost consciousness after ~15 seconds (link (http://www.spacesafetymagazine.com/aerospace-engineering/space-suit-design/early-spacesuit-vacuum-test-wrong/)).

I would agree that it doesn't need very much - there's pictures of a minimal space suit required to work during an EVA and it leaves very little to the imagination (not linked as I think it breaks board rules on suggestive content). Whether the garment would be wearable during regular duties would be debatable though; not for practicality, but for morale reasons (particularly with mixed gender crews) and I would think it would be too uncomfortable to wear that under a uniform or other external clothing for modesty.

Nevertheless, the original question would be 'what is the space ship version of a wet navy ship sinking' and I think loss of main life support would be equivalent to sinking ship with both requiring emergency repairs soon or the ship is lost. Other catastrophic failures have analogues though (engines, magazine explosions, etc), and in my opinion, life support doesn't have a direct analogue to a wet navy, just as a space ship doesn't have an extreme loss of Z axis (AC4 Jackdaw glitches (https://www.youtube.com/watch?v=9pQ_ZozZIio) aside).

Storm_Of_Snow
2015-06-22, 07:18 AM
I would agree that it doesn't need very much - there's pictures of a minimal space suit required to work during an EVA and it leaves very little to the imagination (not linked as I think it breaks board rules on suggestive content). Whether the garment would be wearable during regular duties would be debatable though; not for practicality, but for morale reasons (particularly with mixed gender crews) and I would think it would be too uncomfortable to wear that under a uniform or other external clothing for modesty.

Most of the bulk of a space suit is to absorb micro-meteor hits and radiation - given the ship's hull will be doing that for you, I was really thinking what would effectively be rubber underwear to keep everything in your digestive tract and bladder if the pressure suddenly drops, plus the mask to stop you vomiting, supply oxygen and protect the eyes and ears. Skin capillaries bursting and other effects might be unpleasant, but they could be dealt with later.

Damage control crews on the other hand might need more heavily armoured suits than a modern space suit, as they're more likely at risk of their suits being punctured when working in damaged areas of the ship. They might even need to work during the event that's caused the damage.



Nevertheless, the original question would be 'what is the space ship version of a wet navy ship sinking' and I think loss of main life support would be equivalent to sinking ship with both requiring emergency repairs soon or the ship is lost. Other catastrophic failures have analogues though (engines, magazine explosions, etc), and in my opinion, life support doesn't have a direct analogue to a wet navy, just as a space ship doesn't have an extreme loss of Z axis (AC4 Jackdaw glitches (https://www.youtube.com/watch?v=9pQ_ZozZIio) aside).
Well, even without considering the interior spaces of naval vessels possibly being buttoned up and running on recycled air in case of NBC conditions, I guess you could say suddenly having the air you'd normally breath being replaced by water is a critical loss of life support. :smallwink:

Mike_G
2015-06-22, 09:13 AM
Even without machine guns, you can put so much lead down range that any infiltrators would be flushed out.

German infantry during WW1 came under what they thought was machine gun fire, was in fact squads of British riflemen on a mad minute (https://en.wikipedia.org/wiki/Mad_minute). That said, if you can get squads of well trained riflemen to converge on a target, it's not really asymmetric warfare anymore, just standard warfare.

Machine guns are the king of battle in open spaces. If your field of vision only extends a few yards because of heavy jungle, and an enemy can approach very close, then the squad with repeating carbines can approach and flank your big, slow, heavy gun and overwhelm you. Try carrying an early 20th century machine gun and the huge amount of ammo it needs through the undergrowth. That's why I suggested a Lewis gun. One man can carry it over his shoulder. Distribute magazines among the rest of the squad who all have nice light carbines, and you're golden. You can move, attack, retreat and so on.

If you have a hundred yards of clear field of fire in front of you, prepare machine gun positions. If you plan to have sudden shootouts at a range of ten yards, not so much.

Jungle fighting is a lot like house to house fighting. Look at what the troops carried in Iraq. Medium or heavy machine guns were to defend static positions or on vehicles, not lugged by infantry patrols. Even the M16A2, which is very light rifle, was replaced by the shorter M4, because nobody cares about having less range and accuracy when you are having a shootout in an 8x10 room.

If I had to take a machine gun position over 500 yards of No Man's Land, I would worry if I only had 100 riflemen. In the jungle, I'd feel ok with a four man fire team.

Been to Parris Island, went to infantry school, studied this kind of thing.

Mr. Mask
2015-06-22, 09:42 AM
With the coming Warhammer Total War, it made me wonder. In what circumstances are you likely to use astronaut infantry?

If your boarding a merchant vessel to inspect its goods, certainly. If your taking over a surrendering vessel in general. Similarly, if you want to take a satellite or asteroid bunker rather than blow it to smithereens.

Other than that, I'm running short of ideas at the present.

Carl
2015-06-22, 10:27 AM
@Telok: I'm curious what you know about lasers because again what your claiming violates basic laser operating principles, (at least of the type i'm talking about). For the laser to be as bad as empty space would require that you would be unable to build a multi-KW laser with a lens diameter of less than x square meters, (i forget what the exact watt's per square meter of a radiator in space is but it's a lot lower than the lens energies of a laser). In a laser the entire lasing medium is an emissions system, meaning it''s the volume of the lasing medium not the surface area that matters, (i'm not 100% sure on why that is, i just know thats what the operational physics works out as, presumably the lasing medium is nearly transparent to it's emissions spectra). Surface area mater's primarily in the sense of lensing.

@Brother_Oni: And where might i ask did you read about said immodest space suits? Unless it;s a new development in materials science out of NASA or another fully operational space agency, just disregard it. A space suit has most of the same requirements as cold water wet suit, (heating and tear/cut resistant), combined with the need to be rigid enough to take internal pressure without inflating like a balloon. The worst such a suit should show off unless there's a new material on the market i haven't heard of, (not totally impossible but i'm definitely sceptical right now), is the kind of curves a tight ground side outfit would show off.

Obviously i respect your not linking them if their as bad as you claim but i question weather your source is in any way in touch with reality.

Storm_Of_Snow
2015-06-22, 11:25 AM
With the coming Warhammer Total War, it made me wonder. In what circumstances are you likely to use astronaut infantry?

If your boarding a merchant vessel to inspect its goods, certainly. If your taking over a surrendering vessel in general. Similarly, if you want to take a satellite or asteroid bunker rather than blow it to smithereens.

Other than that, I'm running short of ideas at the present.
Some of these are probably specific instances of things you've already mentioned in general, but:

Attacking a base on an airless moon, or a moon/planet that's got a toxic atmosphere, or deploying your own troops outside of said base to defend it (Mars or Titan for example).

Rescue missions for downed craft on such bodies, or escorting scientists who're investigating strange items on/under the surface.

Taking over orbital shipyards so you can use them to repair/build your own vessels, or steal/destroy a new vessel the enemy is building. Or orbital defences to both secure orbit for your ships and turn their guns on the planet's surface.

And depending on what they're equipped with, a stealth boat might be able to drop them close to the planet, then they could potentially paradrop to the surface - a stealth boat would still have a fairly significant re-entry heat signature, while the trooper's small size would make them virtually impossible to pick up (I remember once seeing a concept for a single person re-entry device for evacuating from a space station, which was basically a life raft filled with an ablative foam that would be deploy from the back of the space suit, and a parachute for when they'd been slowed down and had enough atmosphere around them to make it worthwhile).

Brother Oni
2015-06-22, 11:40 AM
@Brother_Oni: And where might i ask did you read about said immodest space suits? Unless it;s a new development in materials science out of NASA or another fully operational space agency, just disregard it. A space suit has most of the same requirements as cold water wet suit, (heating and tear/cut resistant), combined with the need to be rigid enough to take internal pressure without inflating like a balloon. The worst such a suit should show off unless there's a new material on the market i haven't heard of, (not totally impossible but i'm definitely sceptical right now), is the kind of curves a tight ground side outfit would show off.

Obviously i respect your not linking them if their as bad as you claim but i question weather your source is in any way in touch with reality.

They were proposed designs back in the 60s and 70s which weren't ultimately used: Space activity suit (https://en.wikipedia.org/wiki/Space_activity_suit). Apparently development has been ticking over since then and they appear to have solved the issues: MIT biosuit (http://newsoffice.mit.edu/2014/second-skin-spacesuits-0918).

I've seen a higher res version of the 1971 version and it doesn't leave much to the imagination regarding the astronaut's anatomy - not bad enough to see whether he was Jewish, but equally not conducive to a formal environment.
Surprisingly, it's thinner than a cold water wet suit as heat loss isn't that much of a concern in space as it is under the sea (if anything, shedding excess heat is the issue).

PersonMan
2015-06-22, 12:38 PM
In films and such you often see a military force keep firing away at something that isn't even slowed down by their assault - I've read that this isn't something any well-trained force would do, as it's just a waste of ammunition, but what would be done? If you have something approaching you, whether it's some kind of mech or monster or single person, that seems to be unaffected by everything your forces can bring to bear, what do you do? Just retreat before it inflicts major losses? Try to trap it somewhere? Something else?

Khatoblepas
2015-06-22, 12:46 PM
Me and my DM are currently working on rules for construction of siege engines based off the old table of Siege Weapons from AD&D's Combat & Tactics and though it has gold cost, it lacks any information on how long it would take for an engineer to build them and then for them to be set up. Information on construction times is not forthcoming in our google searches, unfortunately, so I thought it might be good to ask GITP's resident experts!

How long does it take to build (and set up on the battlefield) various siege engines? Ballista, Catapults, Cannons, Rams, Towers, Trebuchets - the whole shebang. How long would it take for them to be constructed, how many people would need to work on them in that time and how long would it take them to be set up on the day? I'm not sure what time period this would be in, but we could possibly assume european middle ages, as that's AD&D's rough period it emulates.

Thank you in advance!

Mr. Mask
2015-06-22, 01:03 PM
Snow: Reminds me of the drop suits from Starship Trooper. Though I guess conventional infantry won't really be used, for the same reasons underwater infantry aren't.


Person: Some of that happened in WW1 and 2. In WW1, the early tanks may've seemed like terrible monsters (in actuality, they were closer to deathtraps). In WW2, the Americans came under attack by Tiger tanks a few times without proper support for dealing with them, and it's pretty depressing when you bazooka does zilch. The Germans likely had similar problems against the T34s, as many of their anti-tank guns and Panzers were underpowered against their armour.

That's not to say these were truly unstoppable monsters like in the movies, but if you lack the artillery and tanks and other munitions to take out a tank, then a tank can get to seem pretty invincible to a body of troops. I can't remember the accounts of these cases in much detail, sadly, but the general procedure was to call for help and try to hold position, as far as I can recall (not to say holding position against a superior foe is a wise idea).


Khat: There was some RPG or other that had some detailed information on siege weapons, I think, or possibly a few of them, but I can't remember their names I'm afraid. I hope someone does remember so they can help you out.

Gnoman
2015-06-22, 01:28 PM
In WW1, the early tanks may've seemed like terrible monsters (in actuality, they were closer to deathtraps). In WW2, the Americans came under attack by Tiger tanks a few times without proper support for dealing with them, and it's pretty depressing when you bazooka does zilch. The Germans likely had similar problems against the T34s, as many of their anti-tank guns and Panzers were underpowered against their armour.

That's not to say these were truly unstoppable monsters like in the movies, but if you lack the artillery and tanks and other munitions to take out a tank, then a tank can get to seem pretty invincible to a body of troops. I can't remember the accounts of these cases in much detail, sadly, but the general procedure was to call for help and try to hold position, as far as I can recall (not to say holding position against a superior foe is a wise idea).


This is accurate for WWI - whenever tanks appeared, everything in the area concentrated fire on them both out of fear and because of the sheer threat they represented - WWI tanks were designed to maximize enfilade fire and absolutely devastate troops in trenches.

For WWII, both your examples are fatally flawed. US forces rarely encountered Tigers of any type, and on the occasions they did, the Tiger was destroyed with ease - every antitank weapon in the US arsenal except the 35mm AT gun they started the war with could penetrate a Tiger quite easily (Because the Tiger was slab-sided, the frontal armor was only about 20% better than the M4 Sherman's), and even that could reliable penetrate the rear or sides, making it merely a question of holding against it while you moved a bazooka team or towed AT gun around to the side. Meanwhile, the excellence T-34 tank was greatly offset by the relatively poor doctrine and crew experience levels that were endemic to the Soviet army in the first several months of BARBAROSSA - luring the tanks into an ambush was very easy, and by the time the Soviets got that problem sorted out the Germans had upgraded enough that the T-34 and KV tanks were no longer invincible.

Stellar_Magic
2015-06-22, 01:37 PM
They were proposed designs back in the 60s and 70s which weren't ultimately used: Space activity suit (https://en.wikipedia.org/wiki/Space_activity_suit). Apparently development has been ticking over since then and they appear to have solved the issues: MIT biosuit (http://newsoffice.mit.edu/2014/second-skin-spacesuits-0918).

I've seen a higher res version of the 1971 version and it doesn't leave much to the imagination regarding the astronaut's anatomy - not bad enough to see whether he was Jewish, but equally not conducive to a formal environment.
Surprisingly, it's thinner than a cold water wet suit as heat loss isn't that much of a concern in space as it is under the sea (if anything, shedding excess heat is the issue).

Considering we're talking about a warship... I'd expect something akin to the biosuit to be standard issue... plus some sort of body armor worn over the top of it when in combat to protect the crew from spalling and so forth from the ship taking fire (wait, a body smock with armor on top... that's a stormtrooper!) A loose breathable uniform could be worn over the top of the biosuit design, sort of like BDUs. Then in true formal situations, they could take them off and break out the equivalent of Class As or whatever.

Mr. Mask
2015-06-22, 01:59 PM
Gnoman: Tigers, from what I recall of the last lengthy discussion you had with G on this subject, it was not so plain as you insist.

As for the T34s, are you saying the Germans never had a skirmish where the Russians had the drop on them?

Gnoman
2015-06-22, 02:31 PM
Gnoman: Tigers, from what I recall of the last lengthy discussion you had with G on this subject, it was not so plain as you insist.

As for the T34s, are you saying the Germans never had a skirmish where the Russians had the drop on them?

1: Why don't you prove it. You can't - Nobody ever had significant trouble dealing with the Tiger. Not the British, not the Americans, not the Russians. It was a piece of garbage that hastened the fall of Germany by wasting resources. Every single account of Tigers inflicting serious harm has been disproven - either by Allied records showing there not having been a battle on that day at all, or by German records showing that there wasn't a Tiger within a thousand miles of where Allied troops claim to have fought one. Sure, it had a monstrous gun and could do some real damage if it managed an ambush, but it was never more useful than a towed 75, let alone a vehicle like the StuG that was actually good.

As for the T-34, in the early stages of the war successes were few, and the veteran German troops were excellent at simply falling back a little and leading the Soviets into a trap.

Telok
2015-06-22, 05:51 PM
@Carl. We are certainly misunderstanding each other somewhere. I get that the output of this substance or machine is an IR laser, and it's purpose is to cool a spaceship without jettisoning mass. So you need to convert the thermal energy of the ship's components into radiant IR energy. This is usually done with heatsinks and radiators. So what are the properties of this thing, it's input, and how is it supposed to be better than a radiator?

MrZJunior
2015-06-22, 06:58 PM
Machine guns are the king of battle in open spaces. If your field of vision only extends a few yards because of heavy jungle, and an enemy can approach very close, then the squad with repeating carbines can approach and flank your big, slow, heavy gun and overwhelm you. Try carrying an early 20th century machine gun and the huge amount of ammo it needs through the undergrowth. That's why I suggested a Lewis gun. One man can carry it over his shoulder. Distribute magazines among the rest of the squad who all have nice light carbines, and you're golden. You can move, attack, retreat and so on.

If you have a hundred yards of clear field of fire in front of you, prepare machine gun positions. If you plan to have sudden shootouts at a range of ten yards, not so much.

Jungle fighting is a lot like house to house fighting. Look at what the troops carried in Iraq. Medium or heavy machine guns were to defend static positions or on vehicles, not lugged by infantry patrols. Even the M16A2, which is very light rifle, was replaced by the shorter M4, because nobody cares about having less range and accuracy when you are having a shootout in an 8x10 room.

If I had to take a machine gun position over 500 yards of No Man's Land, I would worry if I only had 100 riflemen. In the jungle, I'd feel ok with a four man fire team.

Been to Parris Island, went to infantry school, studied this kind of thing.

I apologize if anything I said was taken as questioning your qualifications, I did not intend any disrespect. I wanted to try and clear up the disagreement between the various pieces of advice I have been given.

Broken Crown
2015-06-22, 07:59 PM
@Broken Crown: How about actually looking how active cooling systems actually work before spouting nonsense.

That's pretty rich. How about addressing the issues I raised? Such as, first of all, explaining why you think active cooling eliminates the need for large radiators? You keep dodging this question, in spite of it being your original point of contention with my post, and I'm frankly out of patience.

I know how active cooling works. If you read my earlier post, you'll note that I was asking you what you meant by active cooling in the context of "laser resistant armour", and pointing out that it produces extra waste heat.


the only heat generated would be by the pumps.

And how much heat would that be? It depends on the power and efficiency of the pumps, doesn't it? If you're moving large masses of fluid (which you would need to move large amounts of heat, as heat capacity is proportional to mass) at "insane" flow rates (and what flow rate to you consider "insane"?) then you need powerful pumps.

Also, as I pointed out, adding this cooling system will also require the ship to generate more heat, because, to reiterate, it's heavy, so the ship will require more thrust in order to maneuver.


It's like saying your car radiator's cooling generate more heat than the engine it's cooling. It doesn't, in fact aside from the negligible heat from the pump it generate's nothing.

Except that we're not talking about a car, are we? Compared to a spaceship, a car doesn't have any trouble getting rid of heat. Look at the International Space Station (https://en.wikipedia.org/wiki/International_Space_Station#/media/File:ISS_and_Endeavour_seen_from_the_Soyuz_TMA-20_spacecraft_14.jpg). Compare the size of the radiators to the size of the station as a whole. The ISS uses about 90 kW of power (or about 120 hp; a lot less than most cars). It also uses active cooling. In space, 90 kJ is not a "negligible" amount of heat to have to get rid of.


Your confusing forced cooling with thermocouples and the like with what i'm talking about.

Given that I haven't even mentioned thermocouples in this thread, I don't think I'm the one who's confused, here.

Anyway, I'm afraid that I must take my leave of this conversation for the time being, as my computer is dying and I need to take it to the shop. I don't think the debate was going anywhere productive in any case.

Mr. Mask
2015-06-22, 09:31 PM
Gnoman: How would I prove the Tiger isn't useless when you only trust evidence which favours your standing, as when you argued with G?

Germany had the upper hand consistently and consistently outdid them. Saying that is conclusive proof that a group of Germans never faced situations where they lacked the necessary firepower to deal with a T34 effectively is ludicrous for any large scale war.



Speaking of the world wars, does anyone know of cases of the Henry Rifle or other repeaters being used by infantry in World War 1?

Mike_G
2015-06-22, 09:52 PM
I apologize if anything I said was taken as questioning your qualifications, I did not intend any disrespect. I wanted to try and clear up the disagreement between the various pieces of advice I have been given.

No, no. I'm not insulted, and I wasn't trying to imply that you ha d slighted me.

I just had an issue with whoever said a machine gun has the firepower of "a thousand riflemen," because that's very misleading.

It has the theoretical firepower of many riflemen. It has a greater effective range and with a tripod or bipod or carriage mount has a greater effective accuracy, and can put more rounds downrange.

But most of that means diddly squat in close terrain, while its weakness, maneuverability, means a whole lot.

It's like saying a sniper rifle has much more firepower than a snub nosed revolver. That's true, but if you are having a shootout in a crowded elevator, you want the revolver.

Jungle means close range, short sight lines, lots of cover, all of which hurt the HMG/MMG and help the dispersed rifleman trying to flank it. And it's hard to walk through even if you aren't carrying a sixty pound gun and a twenty pound tripod and ten pounds of ammo. And if you are supplying by airdrop, weight of the load matters. For a mobile, light infantry jungle force I'd take the Vickers' weight in shotguns or carbines any day.

rs2excelsior
2015-06-22, 10:06 PM
Regarding the space combat radiator discussion, there's a site that's absolutely essential for this kind of thing. Although be warned: it shoots holes in a lot of classic space opera tropes.

Relevant to the discussion:
http://www.projectrho.com/public_html/rocket/basicdesign.php#id--Heat_Radiators
http://www.projectrho.com/public_html/rocket/spacegunconvent.php#id--Laser_Cannon--Efficiency
http://www.projectrho.com/public_html/rocket/thermodynamics.php

It's a wonderful site, check it out.

Gnoman
2015-06-22, 10:44 PM
Gnoman: How would I prove the Tiger isn't useless when you only trust evidence which favours your standing, as when you argued with G?


All you need to do is cite a single battle backed up by both German and Allied records where the Tiger proved a decisive advantage compared to a StuG or Panzer IV. THe problem is that you cannot do this. The Americans fought Tiger Is less than a dozen times in the entire war - reports of "Tigers" were almost invariably proven after the war to be Panzer IV tanks, which look quite similar. In every case, the Tigers were destroyed with very little trouble.

The British first encountered the Tiger early - killing their first Tigers with 6-pounder and 2-pounder antitank guns at range in North Africa on February 1, 1943. Tests showed that the only anti-tank weapon that could not reliably defeat a Tiger was the obsolete 2-pounder - the 6 pounder (with the early, less capable ammunition) required careful aim, and anything bigger did not.

The USSR was so contemptuous of the Tiger that they made anti-tank capability dead last on their design priority when they upgunned the T-34 and designed the IS series of tank - the guns already in service were more than adequate, so they went for the highest HE charge instead.

The ONLY sources that support the notion that a Tiger was a superweapon, or even an adequateweapon, are post-war histories that, on the opening of the German archives, did not vet what they were looking at properly for accuracy and thus took boasting claims that were impossible (One Tiger commander claimed to have killed 30 Shermans on a day when none of the Allied Powers HAD any Shermans within 300 miles) at face value because they hadn't yet found the "Ignore this, but use it for propaganda" files. All of these have corrected the error in subsequent books, but the older material is still out there, and forms the base of most pop-history.

Mr. Mask
2015-06-22, 11:08 PM
A battle where the Tiger proved a decisive advantage? They're slightly less rare than unicorns, and making contingency plans for facing them would make slightly more sense than contingencies for unicorns.

I recently read an account of Tigers being impressive from a soldier in an article about that movie, Fury. http://www.theguardian.com/film/filmblog/2014/oct/24/fury-movie-tank-veteran-sherman-verdict-realistic

Is he just saying the Tiger is impressive for the film's portrayal's sake?

Gnoman
2015-06-22, 11:41 PM
A battle where the Tiger proved a decisive advantage? They're slightly less rare than unicorns, and making contingency plans for facing them would make slightly more sense than contingencies for unicorns.

I recently read an account of Tigers being impressive from a soldier in an article about that movie, Fury. http://www.theguardian.com/film/filmblog/2014/oct/24/fury-movie-tank-veteran-sherman-verdict-realistic

Is he just saying the Tiger is impressive for the film's portrayal's sake?

Have you ever played the "telephone game"? If you haven't, it is a child's game (often played in schools to demonstrate communication problems) in which each member of a group of children whispers a message into another's ear with instructions to "Pass it on". By the time it reaches the end of the line, it's entirely different from the original. Memory works like that - your memories are constantly rewritten based on what you hear, see, are told, and what you tell about the events, particularly with the passage of decades on a subject that is constantly the subject of documentaries, popular films, history books, novels, etc.

Not only this, but the sheer stresses of combat make for very poor memories in the first place. Most soldiers remember battles in snapshots and short clips - the overcooked pork smell of a burned-out tank crew or the explosion of a tanks ammunition might remain indelibly printed forever when firing the shot that killed it was forgotten immediately.

Finally, soldiers on the ground are in an incredibly poor position to correctly identify enemy equipment in the first place. The British Army, for example, reported taking fire from 88mm cannon hundreds of times more often than the Germans actually used 88s. 50mm and 75mm guns were used in the vast majority of cases, but the 88 is what the British feared, and they were all too ready to imagine those fears given life. The same was true for the Tiger. Visually, there is very little difference between a Tiger I and mid-model Panzer IV - the gun's longer, the hull's wider in proportion to the height, turret's a slightly different shape, and several other differences - but these are not obvious ones. The main identifying characteristics are the same, and you're not likely to get a good look at the thing until you've already killed it, at which point you don't care what it was - only thing that matters is that it is dead and you are not. Thus rumors of the rarely encountered German supertank carrying the bogeyman 88 and invulnerable to anything short of a battleship spread from unit to unit, aided by fragmentary intelligence briefings that contained little more than images (sometimes pictures of Panzer IVs were used by mistake!) and assurances that the weapons available would do the job - which most soldiers ignored, because they *knew* the tank was invincible.

On the other side, there are many German diaries from the African period that attribute the same bogeyman status to the British's new M4 - describing it as an almost invulnerable supertank that carried a gun capable of smashing through anything they could throw at it.


Incidentally, I suspect that Fury was written to use a Tiger II for that climactic battle - Tiger Is had mostly gone the way of the dodo by that time, and Fury (a 76mm Sherman) scored several square hits that would have penetrated a Tiger I's frontal armor but not a II's- but the opportunity of using a real Tiger came up and they simply substituted.

Mr. Mask
2015-06-23, 12:05 AM
Which brings it back to my original point. Because soldiers' accounts are capable of error, you won't accept accounts from soldiers', except when they attest the Tiger to be useless. I don't doubt that there was plenty of confusion going on around troops, but that doesn't disprove all accounts of the Tiger being an effective tank.

How close was the Fury to the Tiger when it scored its hits (can't remember)?

Gnoman
2015-06-23, 12:26 AM
Which brings it back to my original point. Because soldiers' accounts are capable of error, you won't accept accounts from soldiers', except when they attest the Tiger to be useless. I don't doubt that there was plenty of confusion going on around troops, but that doesn't disprove all accounts of the Tiger being an effective tank.


I don't accept soldier's reports at all unless they are backed up by loss records, salvage records, photographic proof, reports from both sides, or at least some evidence that the tanks in question actually existed. This (http://tankarchives.blogspot.ca/2015/04/lend-lease-hero.html) is a good example of what I consider a reliable source - the blog author cross-checked the Soviet claims with german reports and found that they matched, while here (http://tankarchives.blogspot.ca/2014/10/cheating-at-statistics-9-art-imitates.html) is a good example of why reports from only one side are useless - the German reports claim that their attack completely destroyed the enemy (while reporting losses only in that the Tigers Available number goes from 18 to 3), while the Soviets claim 13 tanks destroyed with minimal losses.

EDIT: As for the Fury, it was very close in the movie - I don't recall an exact figure given, but it can't have been more than 500 yards given how quickly they got behind the Tiger.

Brother Oni
2015-06-23, 02:17 AM
All you need to do is cite a single battle backed up by both German and Allied records where the Tiger proved a decisive advantage compared to a StuG or Panzer IV. THe problem is that you cannot do this. The Americans fought Tiger Is less than a dozen times in the entire war - reports of "Tigers" were almost invariably proven after the war to be Panzer IV tanks, which look quite similar. In every case, the Tigers were destroyed with very little trouble.

Are you saying that you believe that the Battle of Villers Bocage (https://en.wikipedia.org/wiki/Battle_of_Villers-Bocage) and Micheal Wittmann's record (https://en.wikipedia.org/wiki/Michael_Wittmann) are just overblown propoganda?

While I agree that often the Allied soldiers over-reacted to every German tank, assuming they were a Panzer or a Tiger when they weren't, there's a sufficient reports where a Sherman scored a direct hit on a German tank only for the shell to bounce off, to lend some credence to their reputation.
On the German side, I know they quickly learned to distinguish between a regular M4 and the M4 with the longer cannon that could punch a hole through anything (the Sherman Firefly).

I'll have to do some research on what sort of tanks, but I know that tanks were crucial to finally defeating the British forces entrenched in at Arnhem bridge during Operation Market Garden.

Edit: I just remembered I found an ARR of Shermans vs a pair of Tigers for a previous version of this thread, where a Sherman hid behind a building and nailed a Tiger in the fuel tank as it passed by, so while I agree with Gnoman that the mythical reputation of Tigers is somewhat overblown, it's still an intimidating tank from the actions of the Sherman commanders during that engagement. I'll see if I can find it again.

Edit 2: Still looking for the ARR, but I found an earlier post by Galloglaich (http://www.giantitp.com/forums/showsinglepost.php?p=18256490&postcount=351) that suggests a small number of Tigers among the other AFVs, were instrumental to the Allied rout during the Battle of Kasserine Pass (https://en.wikipedia.org/wiki/Battle_of_Kasserine_Pass).

Gnoman
2015-06-23, 04:02 AM
Are you saying that you believe that the Battle of Villers Bocage (https://en.wikipedia.org/wiki/Battle_of_Villers-Bocage) and Micheal Wittmann's record (https://en.wikipedia.org/wiki/Michael_Wittmann) are just overblown propoganda?

The extent of Whittmann's role in the battle is unclear - the losses are known, but it is not certain precisely what happened - Whitmann's account, the British account, and the official German account differ quite a bit, including precisely where Whitmann's tank was disabled. Still, it is true that this is one engagement where the Tiger's gun and armor made a major contribution.


While I agree that often the Allied soldiers over-reacted to every German tank, assuming they were a Panzer or a Tiger when they weren't, there's a sufficient reports where a Sherman scored a direct hit on a German tank only for the shell to bounce off, to lend some credence to their reputation.
On the German side, I know they quickly learned to distinguish between a regular M4 and the M4 with the longer cannon that could punch a hole through anything (the Sherman Firefly).

This is oft cited, but nobody seems to be able to prove it - British tank units show no significant loss differences between Firefly and non-Firefly tanks. If the Germans cared what model of M4 they were fighting, the more dangerous ones would be target preferentially, which no evidence supports.



Edit 2: Still looking for the ARR, but I found an earlier post by Galloglaich (http://www.giantitp.com/forums/showsinglepost.php?p=18256490&postcount=351) that suggests a small number of Tigers among the other AFVs, were instrumental to the Allied rout during the Battle of Kasserine Pass (https://en.wikipedia.org/wiki/Battle_of_Kasserine_Pass).

Kasserine Pass took place in February of 1943. At that point, the Tiger had been in production since August of the previous year, producing 25 tanks per month for for ~7 months. That translates to a maximum total of 175 Tigers on all fronts - this includes full production for the beginning and end months, which is unlikely. A large percentage of this were sent to the Eastern Front - we'll say half the total. That leaves no more than 90 Tigers for the entire North African theatre. The Tiger was unreliable - it was rare for a Tiger unit to have half their vehicles running, particulalry in the desert as the first batches of Tigers had air filters that didn't handle sand too well. That leaves 45 combat-effective vehicles outside the Eastern Front. Let's assume every last one of them was at Kasserine Pass. The Germans deployed over 200 tanks to that series of battles. The Panzer III and IV made up the bulk of them by far - and there's no evidence that the Tiger performed better than those two models in the battle - it is unlikely that that would have mattered as the US Army drove straight into a trap and was engaged by flanking tanks and heavy anti-tank guns. Any tank would have done well in that sort of battle, and the Wehrmacht were masters of setting it up.

Carl
2015-06-23, 04:31 AM
@Brother_Oni: Ahh cheers, yeah i'm pretty sure that would be useless in practise because the material's too thin to resist wear and tear of bump and knocks and catching on stuff. That's why i mentioned cold water diving suits, apart from space suits also needing to cool if the astronaut is in sunlight instead of shade as you mentioned, and the pressure differential they have to deal with they have many of the same requirements. Short of a new material coming in any realistic space suit that's sufficiently hard to damage accidentally, (let alone a military grade one that has to stand up to shrapnel and the like), is going to require thick enough material that there shouldn't be any issues, (you could take extra measures if needed but i doubt it would be).

@Broken crown: Instead of parroting the wiki do research and actually apply your brain to the problem. If you had done that you would allready know that electric motor driven pumping systems are remarkably efficient. Given all the other source of heat on a spacecraft, especially on a military ship under attack the pumps are not going to be a significant amount of extra heat.

Also i answered how you get rid of the heat in the initial premise. The armour of the ship IS the radiator. You just switch that part under attack to cooling mode when it gets hit. You certainly would need some dedicated structures for certain types of waste heat where you want to radiate at maximum thermal temperature and the like. But for many general purpose circumstances the same system that can carry away the heat of a laser attack will be able to dump heat to space. It all depends on certain parameters. It's not going to be as god as a dedicated radiatir because the surface area won;t be optomised. But it's still bloody useful because again, it lets you turn what would otherwise be non-radiating surface area into radiating surface area.

@Telok: No worries. I'll try and lead you through the basics,.

The concept is inspired by the earliest models of laser. These used a material, (i think neodymium was the first then CO2, but i'd need to double check my sources for the progression and names to be sure), in a tube and then using what amounts to a super powered camera flash to dump light into it. The material absorbs the light and then re-emits it as coherent light, a percentage is lost and become heat.

The concept is to take the same general principle but do it with infrared radiated from the walls of the lasing tube, (which unlike a ship exterior can be any shape you desire to maximize emissions area), in normal radiator fashion and with a material that then outputs that. Thermodynamics says it would have to be a frequency of IR output, you can''t turn IR into UV AFAIK. But you can turn it into a different form of IR. Not all of the IR you dump in will go away, a lot will remain trapped in the system so you have to carefully control how much heat you dump in and without a much better efficiency than those early lasers you won't get very good output. But it allows you to turn internal volume into radiator area. That's bloody useful so long as the efficiency isn't too bad.

Does that help?

Brother Oni
2015-06-23, 12:38 PM
This is oft cited, but nobody seems to be able to prove it - British tank units show no significant loss differences between Firefly and non-Firefly tanks. If the Germans cared what model of M4 they were fighting, the more dangerous ones would be target preferentially, which no evidence supports.

I'll see if I can find a reference, but in the mean time, here's an account of the engagement I mentioned earlier: link (http://www.thefewgoodmen.com/thefgmforum/threads/sherman-75mm-v-tiger-action.3696/).

Of note is the fact that a direct hit on the front lower hull of a Tiger at a distance of 75-100 yds from a 75mm Sherman did minimal damage to the Tiger, however a 20-30yd shot later on penetrated the Tiger's thinner side armour.



The Panzer III and IV made up the bulk of them by far - and there's no evidence that the Tiger performed better than those two models in the battle - it is unlikely that that would have mattered as the US Army drove straight into a trap and was engaged by flanking tanks and heavy anti-tank guns. Any tank would have done well in that sort of battle, and the Wehrmacht were masters of setting it up.

I'm not as familiar with the North Africa campaign, so if you're right in that the evidence is minimal or non-existent, I'll defer to you.


@Brother_Oni: Ahh cheers, yeah i'm pretty sure that would be useless in practise because the material's too thin to resist wear and tear of bump and knocks and catching on stuff.

I'm not so sure that it's completely useless given that NASA and MIT are still working on the concept nearly 40 years later.

Gnoman
2015-06-23, 12:47 PM
I'll see if I can find a reference, but in the mean time, here's an account of the engagement I mentioned earlier: link (http://www.thefewgoodmen.com/thefgmforum/threads/sherman-75mm-v-tiger-action.3696/).

Of note is the fact that a direct hit on the front lower hull of a Tiger at a distance of 75-100 yds from a 75mm Sherman did minimal damage to the Tiger, however a 20-30yd shot later on penetrated the Tiger's thinner side armour.


This is exactly as expected - the 75mm was never intended to remain the primary gun for as long as it did (production had shifted entirely to the vastly superior 76mm before the US had ever heard of the Tiger), but it was decided there was no need to replace the ones already in theatre because they were proving to be more than good enough for everything they were fighting at the time (including Panthers and the occasional Tiger, as in both the African and Italian campaigns the German armor was easily outflanked, and it wasn't until the battles in France after OVERLORD that the German tanks were able to use their oversized guns and excessive frontal armor to good effect). While a 75mm could theoretically have penetrated at that range with a square hit, the round fired was a snapshot that's almost guaranteed to hit at an angle horizontally, and firing at the lower plate at that range would have given it a vertical angle as well.

PersonMan
2015-06-23, 01:16 PM
Somewhat related to a question I asked a few days ago, on sustainable losses for modern militaries:

1. Roughly how much time is necessary to turn a batch of recruits/conscripts/whatever into combat-ready soldiers? How many corners can you cut before your troops are at a noticeable disadvantage to properly-trained ones?
2. How large is the difference between green and veteran soldiers on the battlefield? How much does this change if the green troops have only a shortened training period behind them?

Only somewhat related:
3. What sort of effect can reputation have on morale? If we're dealing with soldiers who believe themselves to be going up against indestructible tanks, or an unbeatable force, can this belief be used to break them and ensure an easy victory?

SowZ
2015-06-23, 01:39 PM
Somewhat related to a question I asked a few days ago, on sustainable losses for modern militaries:

1. Roughly how much time is necessary to turn a batch of recruits/conscripts/whatever into combat-ready soldiers? How many corners can you cut before your troops are at a noticeable disadvantage to properly-trained ones?
2. How large is the difference between green and veteran soldiers on the battlefield? How much does this change if the green troops have only a shortened training period behind them?

Only somewhat related:
3. What sort of effect can reputation have on morale? If we're dealing with soldiers who believe themselves to be going up against indestructible tanks, or an unbeatable force, can this belief be used to break them and ensure an easy victory?

Let me amend the third point with, "How much of a difference does having a famous and beloved general make, even if that general's tactics are no better or worse than the next guys?" How much of a morale difference does fighting beside a well known, Srgnt. York style war hero make?

Gnoman
2015-06-23, 02:19 PM
Somewhat related to a question I asked a few days ago, on sustainable losses for modern militaries:

1. Roughly how much time is necessary to turn a batch of recruits/conscripts/whatever into combat-ready soldiers? How many corners can you cut before your troops are at a noticeable disadvantage to properly-trained ones?
2. How large is the difference between green and veteran soldiers on the battlefield? How much does this change if the green troops have only a shortened training period behind them?


1. This depends a lot on what you're starting with. Educated civilians turn into soldiers more readily than ignorant ones because they're used to learning things, and if a particular military skill is particularly common in your recruit pool you can often skimp on it. As a general reference, the USMC currently uses a 13 week program, in WWII the standard was 8 weeks, and the need for troops caused it to be halved to 4.

2. Massive - just read a few posts up about what happened when a green American unit ran into German veterans in Africa, for example. Lower-quality training makes things much worse, but the most important battlefield skills simply can only be taught to a man by trying very hard to kill him.

Mr. Mask
2015-06-23, 02:40 PM
Well, it depends how combat ready the conscripts were to start with. If they're frontiersmen from the tough part of the country, they probably only need a few weeks of drill so they're organized into a cohesive unit. If they're people with no background of violence whatsoever, you're going to get less robust results and it'll take longer.

The difference between untrained semi-combatants and warriors can get pretty tremendous. Who was the American fellow who caused a whole platoon of Germans to surrender? He wasn't even interested in the war, until his own unit came under attack and he decided to defend it. There was also the fellow who held off a similarly large contingent of Germans with a machine gun, which was on a tank, which was on fire. Some of that is dumb luck, but you need a capable soldier in position to take advantage of that luck, or else it'd not make much difference (if they panic fire and don't hit anything with the MG, they won't hold off many enemies). And as Gnoman says, you get cases like the Italians against the British, the first American battles against the Germans.


Morale is an interesting subject, and it really depends on a lot of political and social factors. Is it socially acceptable to join the other side? That has been common enough in history, even when it wasn't socially acceptable. Will the enemy burn your homes and genocide your people? That has also been common in history, and often inspires men to fight to the death regardless of odds (assuming there isn't a comfortable means of escape for them and their families). Will the enemy offer them decent conditions if they surrender and not perform atrocities against civilians? That is a big incentive to surrender rather than fight, particularly if there isn't a lot of loyalty to the current government.

As for the effects of reputation, it certainly has an effect. The British and US troops seemed to like Rommel, being more interested in what he was doing than their own generals. If men were undecided as to whether to surrender or fight, their enemy being renowned and undefeated would certainly sway them towards not fighting. Some castles have surrendered by that very merit, that other castles did not stand for long against a given conqueror.

Your own general's reputation certainly can have a great effect. Soldiers will follow a leader they respect into death. At the Battle of Gettysburg, the Confederates charged some very well defended positions that'd make most men baulk, and they reportedly offered to charge again after they were forced back with terrible casualties. That was the kind of trust and respect they had for General Lee and their other officers (before, they used to call him Granny Lee).

Storm Bringer
2015-06-23, 02:42 PM
Somewhat related to a question I asked a few days ago, on sustainable losses for modern militaries:

1. Roughly how much time is necessary to turn a batch of recruits/conscripts/whatever into combat-ready soldiers? How many corners can you cut before your troops are at a noticeable disadvantage to properly-trained ones?
2. How large is the difference between green and veteran soldiers on the battlefield? How much does this change if the green troops have only a shortened training period behind them?

Only somewhat related:
3. What sort of effect can reputation have on morale? If we're dealing with soldiers who believe themselves to be going up against indestructible tanks, or an unbeatable force, can this belief be used to break them and ensure an easy victory?

1. modern british army basic training takes about 3 months, and gets you blokes who can stay alive in the field, know what to do when shot at and can hold a rifle without shooting themselves. Infanty get another 6 months on top of that, which covers things like weapons other than a rifle, advanced patrols, assaults, FIBUA, etc. So, for a lorry diver, for example, 3 months, and a infantry tom, about 9 months. not sure how much you can cut form that before things get really ropey. I was hardly chuck norris after three months of basic. some things just take time to sink in.

2. In short, big. no matter how good and realistic the training, some thin gs can only be learnt the hard way. some lessons are quite simply fatal, and a lot of things are "like riding a bicycle": you can't really explain it, you just gotta learn how to do it. however, the relevance of the experenice matters. years of experience patrolling northen ireland is only of so much use in southern Iraq.

3. yes, it can, and has been used in history. Napoleon famously said "the moral is to the physical as the three is to one". empires like Rome often relied on thier reputation to deter potential attackers: why attack a foe that you know is not going to rest until you are dead? the Mongols made a point of being over the top brutal and savage when they attacked a enemy, with the aim of convincing their next target to lay down and surrender (and thus saving everybody a lot of time, money and lives). People faced with a enemy they can't beat, or even hurt, will tend not to stand and fight if any other option is available.


Let me amend the third point with, "How much of a difference does having a famous and beloved general make, even if that general's tactics are no better or worse than the next guys?" How much of a morale difference does fighting beside a well known, Srgnt. York style war hero make?

Again, lots. The is another old saying to the tune of "donkeys led by lions are more dangerous than lions led by donkeys". people will follow a leader who they trust, and will take risks for a leader they trust, because they know he isn't going to squander thier efforts. In 1917, the french summer offensive was a massive failure that cost many casulties for minimal gains. Many troops mutinied, and refused to attack. They were still willing to fight, though, and defended thier lines with suffcient vigor to prevent the germans form realising the scale of the problem. they just wanted a leader they could trust in, plus a few improvments to working conditions (in line with what the brits and germans provided their troops). A new french supreme commander was appointed, one with a reputation for looking after his men, and who the troops liked (cant remember his name for certain. Petian?). This, plus a few well placed crackdowns, solved a threat that could have caused the french front to collapse.

fusilier
2015-06-23, 04:33 PM
Again, lots. The is another old saying to the tune of "donkeys led by lions are more dangerous than lions led by donkeys". people will follow a leader who they trust, and will take risks for a leader they trust, because they know he isn't going to squander thier efforts. In 1917, the french summer offensive was a massive failure that cost many casulties for minimal gains. Many troops mutinied, and refused to attack. They were still willing to fight, though, and defended thier lines with suffcient vigor to prevent the germans form realising the scale of the problem. they just wanted a leader they could trust in, plus a few improvments to working conditions (in line with what the brits and germans provided their troops). A new french supreme commander was appointed, one with a reputation for looking after his men, and who the troops liked (cant remember his name for certain. Petian?). This, plus a few well placed crackdowns, solved a threat that could have caused the french front to collapse.

Yes it was Petain. Who sadly would be so discredited by his actions during WW2, but during WW1 was a sensible general, who believed that attacks that failed should be called off -- many of the other French commanders were willing to lie to their troops and order many useless assaults. Which undercut the trust the soldiers had in their superiors.

I would point out that "motivation" is sometimes a key factor -- there are historical examples of highly motivated soldiers defeating much more disciplined and experience soldiers. There are so many factors in combat that it's hard to reduce failure/success to just one, although often people attempt to.

There was a comment about Italian soldiers versus British up thread that I wanted to address briefly: during the North Africa campaign (especially the early phases), Italians surrendered in large numbers to the British forces there. This is usually interpreted as poor morale, or a poor fighting spirit -- but that wasn't the whole story. The fact of the matter was the Italian army lacked the mechanized transport to move their soldiers fast enough to respond to threats, and, perhaps more importantly, lacked the means to supply water to their soldiers. When Commonwealth and German soldiers found themselves in the same position (cutoff with no water or transport), they surrendered just as readily -- it's just that the Italians found themselves in those positions a lot more often.

The point is that there are many factors that should go into the analysis that don't often make it. Instead there's a tendency to reduce it to one or two. The British made this mistake after the opening North Africa battles, assuming that the Italians simply lacked fighting spirit. While poor morale was a factor, there were other organizational and supply issues that should have been considered. It's not uncommon in the description of later battles to read something like "the Italian divisions put up unexpected resistance." A clear indication that they had misread the earlier battles.

Gnoman
2015-06-23, 04:38 PM
There was also the fellow who held off a similarly large contingent of Germans with a machine gun, which was on a tank, which was on fire.

Audie Murphy.

Carl
2015-06-23, 05:24 PM
@Brother_Oni: From what i can tell the proposed suit has some significant constructional differences that should allow for a much more rugged outer layer. It has some basis in the old suits but it's not just an upgraded version thereof. Because the force is applied mechanically rather than via the material itself a much wider array of materials can be used with corresponding durability benefits. Weather it can go far enough is an open question, but i suspect if someone gets their brain in gear and stops relying on "what everyone knows" it will be easy, (i can think of 2 or 3 concepts right now for adding such damage resistance on with a minimal mobility cost).

Telok
2015-06-23, 06:08 PM
The concept is inspired by the earliest models of laser. These used a material, (i think neodymium was the first then CO2, but i'd need to double check my sources for the progression and names to be sure), in a tube and then using what amounts to a super powered camera flash to dump light into it. The material absorbs the light and then re-emits it as coherent light, a percentage is lost and become heat.

The concept is to take the same general principle but do it with infrared radiated from the walls of the lasing tube, (which unlike a ship exterior can be any shape you desire to maximize emissions area), in normal radiator fashion and with a material that then outputs that. Thermodynamics says it would have to be a frequency of IR output, you can''t turn IR into UV AFAIK. But you can turn it into a different form of IR. Not all of the IR you dump in will go away, a lot will remain trapped in the system so you have to carefully control how much heat you dump in and without a much better efficiency than those early lasers you won't get very good output. But it allows you to turn internal volume into radiator area. That's bloody useful so long as the efficiency isn't too bad.

Does that help?

Right. The old gas lasers. I got to play with one as part of a student project back in the early 90s. Three inches by three inches by three feet, it plugged into the wall socket and had a noisy fan blowing across a heatsink in it. Probably about as powerful as a handful of the cat toy laser pointers today. Horrible efficiency.

So you are putting your radiators on the inside of a box inside the ship, using normal thermal->IR methods to power a laser, and hopefully the increased mass and loss of efficency isn't too bad.

Try the wikipedia on lasers (https://en.wikipedia.org/?title=Laser). It will help in understanding the issues here. What you're advocating is adding another step to the heat dumping process, more inefficency, more mass, more complexity... Your laser output isn't limited to IR wavelengths, it's based on what you build the laser out of. The laser being a tube is pretty common because it simplifies all sorts of things, a non-linear optical resonating cavity would be... interesting to design.

I'm still trying to understand why increasing the size and mass of a spaceship to put the radiators on the inside, in order to power a laser, is a good thing.

Carl
2015-06-24, 08:03 AM
Telok: I don't need to go read the wikipedia, i have reference books that make wikipedia's info largely useless as they're more complete unless it's something fairly recent.

A great example of this is that old laser you saw. That's a great example of the very first generation teribble output efficiency examples. They've built much more efficient and/or powerful, (depending on the exact type and example under discussion), lasers than those, they'd done it by the nineties but well that one you know was probably either a very old piece of kit or the equivalent of the various low grade particle accelerators some universities have. Namely not remotely comparable to the state of the art.


Your laser output isn't limited to IR wavelengths

It is if the input energy is IR, anything else would violate thermodynamics as i understand it. You can't turn heat into another EM frequency. Entropy and all that.


I'm still trying to understand why increasing the size and mass of a spaceship to put the radiators on the inside, in order to power a laser, is a good thing.

What part of you've got limited surface area externally is hard to understand?

In addition any lasing medium should have vastly high heat transfer rates than open space. Meaning that unit area for unit area you get a significant multiplier over open space radiators. In practise system efficiency will bleed off all that advantage, but even if your efficiency is such that you need 10 square meters of lasing medium surface area for every square meter of hull area you can get literally hundreds of square meters of surface area per cubic meter if you can build the lasing and coolant channels thin enough. But even if you've got only a couple of square meters extra cooling per cubic meter you can have as many cubic meter's dedicated to cooling as you have square meter's of surface area and still thanks to square cube law have huge amounts of internal space.

To throw up an example lets say each cubic meter of lasing volume is worth 3 square meters of surface area. Now lets assume a ship that is a cube 20m a side. Providing extra cooling via lasing medium equal to your surface area only requires 10% of your internal volume. (volume of 8,000M^3 surface area of 2400m^2, 800m^3 to match 2400^2 at a 3 m^2 per 1m^3).

Yaktan
2015-06-24, 08:11 AM
Yeah, the big problem with trying to use lasers to cool stuff off is that you need a power source. Because of how light absorption/emission works, you cannot get a laser working by just inputing the light you want amplified. (also, no perpetual motion lasers) While I think cooling lasers would be cool, they seem quite impractical.

Hjolnai
2015-06-24, 09:42 AM
I'm not entirely sure that spaceship cooling is such a major problem. Your standard heat-removal method is black body radiation, which is proportional to absolute temperature^4 (but only linearly proportional to surface area). If your hull has a high enough melting point (steels tend to be around 1700-1800K, I think, while tungsten is more than twice that - although tungsten might not be suitable in other ways), and you have good enough insulators, then heat removal is very rapid when it needs to be.

If you also thermally isolate sections of hull, then you can use a heat pump to push internal heat to the cooler hull sections while the hotter ones disperse the heat from weapons. Heat pumps become less efficient when trying to push through too much temperature differential, so you can't push your internal heat out to the already hot sections.

Of course, I'm not an engineer and I haven't run the numbers, so you can take this with a grain of salt. Also, if the temperature is instantly spiked high enough to melt your hull, it's too late for anything else to help except ablative armour.

Carl
2015-06-24, 10:05 AM
I'm not entirely sure that spaceship cooling is such a major problem. Your standard heat-removal method is black body radiation, which is proportional to absolute temperature^4 (but only linearly proportional to surface area). If your hull has a high enough melting point (steels tend to be around 1700-1800K, I think, while tungsten is more than twice that - although tungsten might not be suitable in other ways), and you have good enough insulators, then heat removal is very rapid when it needs to be.

If you also thermally isolate sections of hull, then you can use a heat pump to push internal heat to the cooler hull sections while the hotter ones disperse the heat from weapons. Heat pumps become less efficient when trying to push through too much temperature differential, so you can't push your internal heat out to the already hot sections.

Of course, I'm not an engineer and I haven't run the numbers, so you can take this with a grain of salt. Also, if the temperature is instantly spiked high enough to melt your hull, it's too late for anything else to help except ablative armour.

One of the links earlier in the thread covered this. A) you can only reject heat at the temperature the coolant is at which can't be higher than system operating temperatures. In short electronics and life support are all going to be fairly low, (270-320 kelvins or so off the top of my head), temperatures which makes rejection rates very low, (a few hundred watts per square meter). B) trying to use thermocouples to force he heat into a high temperature output gets thermally very expensive very fast.


Yeah, the big problem with trying to use lasers to cool stuff off is that you need a power source. Because of how light absorption/emission works, you cannot get a laser working by just inputting the light you want amplified. (also, no perpetual motion lasers) While I think cooling lasers would be cool, they seem quite impractical.

Well no one's talking about using same frequency. Old school lasers as noted used broad spectrum visible light input and got single frequency visible light output. The idea is just the same concept applied to IR except you can also potentially use single spectrum IR input at a sufficiently different frequency to your output. The power source basically is your waste heat here which is the beauty of the system, your not doing anything that's going to generate excess waste heat, at worst at 0% efficency you just get back everything you put in. At anything above 0% you get some of it back and the rest leaves as coherent IR. though to b fair it wouldn't even have to be entirely coherent so long as your optical cavity can handle a non-coherent output, (tricky as i understand it).

Telok
2015-06-24, 04:31 PM
What part of you've got limited surface area externally is hard to understand?
Why is surface area limited? If it's a spaceship you can make huge radiators that fold down to very small size. Surface area is only limited in you want smooth, unbroken, spaceship hulls.


To throw up an example lets say each cubic meter of lasing volume is worth 3 square meters of surface area. Now lets assume a ship that is a cube 20m a side. Providing extra cooling via lasing medium equal to your surface area only requires 10% of your internal volume. (volume of 8,000M^3 surface area of 2400m^2, 800m^3 to match 2400^2 at a 3 m^2 per 1m^3).
That's a small ship, it's smaller than the ISS. It looks like your numbers are probably right but I can't tell what you're describing with them. What is an 8000 cubic meter surface area? Where is the 800 cubic meters coming from?

The volume of the ship is 8000 cubic meters, surface area is 2400 square meters. The magic heat-eating laser is 2 cubic meters and assumed to be equal to the 2400 meters of hull. Sounds wonderful. I'm sure you could build a laser that big today that would use up energy and you could still bolt folding radiator panels to the outside of the ship that would increase the radiant surface area by more than that.

As for the laser, you're expecting to take heat from ship components, turn that into radiant IR, and use that to excite material that gives you the beam. You are probably right that you can't higher energy radiation out of that, I'm not that up on the physics of that part. There's probably some variation on the visible light lasers works in the IR part of the spectrum, which means that this isn't advanced tech. But why do you want to turn the heat into IR inside the ship? At that point you can just assume that you can turn the heat back into work and use that to power the laser or to make your power plant smaller and create less waste heat in the first place.

Really this does sound like just keeping the heat inside the ship to power a laser to get rid of heat. Why does it need to be a laser then? You don't realy care about the form of energy you're expelling, just that you take the heat from your ship's components and radiate it away. It looks like that in order for this laser to work your internal heat->IR converters have to be more efficent than external radiators and then you destroy that efficency by using it to power a laser. At that point you could just put those converters on the hull and get better results.

I'm not seeing any savings in mass or the efficency of expelling waste heat over just using standard radiators. What's the benefit here? Nicer looking spaceships? Fancier technology?

Mr. Mask
2015-06-24, 05:13 PM
I made a thread about the idea of the classic dragon scours the land story, but in the modern day. I thought I'd bring part of the question here.

The scenario, in detail, is thus (short summary below): Inspired by the the few threads about modern day military monsters, I want to discuss in some detail the idea of the classic of a dragon terrorizing the land, but in modern day.


Premise:

A group stumbles upon the long forgotten horde of a great dragon (archaeologists? Militia? Corporations? Oil drillers?). Taking care of a dragon didn't seem too hard. Get a few missiles, some artillery, maybe a helicopter, and it should die. It really, really should have...

After having its horde thieved, and its long slumber ended, the dragon goes berserk, turning the surrounding area into a literal firestorm. A large military force is assembled to combat the dragon, and it's figured it should be easy this time, other than the issues of the firestorm. It did not work out as planned. Being magical in nature, modern equipment fails to kill the dragon, and it scatters the army which assembled to fight it. Its wrath kindled more than ever, the dragon attacks several cities, putting the event among some of the greatest massacres in history.

Once the dragon's wrath has cooled somewhat, chooses a place to return to its rest--a nuclear weapons facility. Dumbfounded as to why the dragon would choose such a place to rest, and that it knew of the place's existence, the nations of the world felt paralysed to act. With its strange powers, could the dragon launch a nuke? And is that why it took the facilities?

With the existence of the dragon, and the Armageddon it unleashed, the whole nation and in particular the surrounding area was left in a vacuum of chaos. Deserting soldiers, people turned bandit, monsters(?), desperate survivors, and looters, mercenaries, and corporations flooding in to profit. An adventurer's haven.


Questions

Several questions arise from this premise, both for how to improve it, and for details as to how this could or would work.

High Fantasy?: Would the setting work better as high fantasy with monsters, or as a mostly real world with the exception of a dragon?

Dragon Capabilities: Seemingly the dragon would have to be highly magical to pull this off. They can start a pretty good firestorm before others can reaction, I'd figure, which would complicate things. But to prevent being targeted by satellites and long range missiles, or even an ICBM if it really became a threat, it would need something like the ability to disguise its presence, perhaps by summoning a hurricane. There's the question of its personal combat abilities. Is it armoured like a battleship, with flight rivalling modern bombers, with a small volcano-like flamethrower (like, when it breathes fire, there is a shockwave that'll deathen those close by)?

Nuclear: The dragon could instead take refuge in a natural or designed nuclear bunker, without the nuclear weapons, and then the nations of the world may not wish to blast it for fear of inspiring another attack while suffering from fallout. The idea of taking a place with nuclear weapons just to confuse governments is interesting, but it may result in them having an itchy trigger finger instead.

Chaos: Would the adventurer friendly scenario likely arise from the chaos? It's true that the collapsing of great powers has lead to large amounts of banditry and mercenary work, but the dragon's actions may be more akin to a very terrible natural disaster. If other elements cause the local government to lose control of the region, it ought to still have the effect.


What are your thoughts on a campaign/setting like this? Anything you would add, or discuss what has been presented.

In short, an ancient and powerful dragon is rudely awoken, it starts a huge firestorm. An army is sent to kill the dragon, they fail. The dragon scatters them, and goes on to massacre several cities, as well as capturing a major nuclear weapons silo as its home (the fact it knew where it was, and its powerful magic, has given credence to the theory it understands nukes, making the idea of nuking the dragon seem tenuous).

I was wondering how powerful and magical the dragon would need to be to accomplish these things.


First thought was long range missiles and artillery being a problem. Supposedly the dragon has physics defying flight for its massive size, making it a small target relative to its swiftness. More importantly, it may be able to summon a small hurricane to act as cover. The hurricanes alone could be disasters of their own, but acting as cover for the dragon and slowing down nearby army manoeuvres, it becomes a catastrophe.

There is the question of personal combat ability. I was thinking the dragon might be armoured like a small warship, shrugging off weapons that would pulverize any tank or aircraft. It's speed would likely be impressive (perhaps akin to modern bombers?), making it a hard target to hit, and since its speed is magically assisted, it may well be strangely agile in its flight (similar to fighter aircraft, perhaps?). For its weapons aside from summoning storms, I was wondering if it may have firebreath reminiscent of a small volcanic explosion, something where the shock wave alone is a threat at several hundred yards, with long range fire that burns like napalm or magma.

What I describe above sounds quite absurd and godlike. Which is the credit I am giving modern military, that any dragon capable of overpowering them unscathed must be an insanely powerful thing. On the other hand, the fact it may've taken the military totally by surprise and general incompetence may've made the difference between a national disaster and a close call.


Any thoughts on what a dragon would need to bring a modern army and nation to its knees?

Carl
2015-06-24, 05:18 PM
Why is surface area limited? If it's a spaceship you can make huge radiators that fold down to very small size. Surface area is only limited in you want smooth, unbroken, spaceship hulls.

Because where talking about a warship that can't do that if it dosen't want to have them instantly destroyed the moment an enemy turns even a low powered laser on you. Like i said good alser resistant armour with piping that allows for really rapid changes from cooling to radiating and back again can perform the radiator role as well as functioning as armour giving you armoured radiators that only have to use their armour function when under attack. Your a bit limited in rejection temperature if you want this to work but it;s the lower rejection temperature stuff that causes the real radiator area issues anyway. The higher rejection temperature stuff probably could be armoured after fashion but it would be trickier. You'd have to use multiple dispersed radiators with a fair amount of ablative material for laser hits and fast snap armoured doors to cover them as they come under attack.

Also again thermodynamics says you can never turn 100% of your heat output into work. Look up carnot efficiency.At the end no matter what you do you will have a lage bulk of waste heat to get rid of.


As for the numbers your struggling to follow. Let me try in more detail.


OK our 20 m cube has an internal volume of 8000 cubic meters, (8000m^3). It also has a surface area of 2400 square meters, (2400m^2). Now then our theoretical laser radiator system can using various methods achieve a very high internal radiator area to volume used, (simplest would be lots of small diameter lasing tubes, non-linear lasing cavities would be better but more complicated), i won;t throw an example up here but at least a couple of hundred square meter's should be doable. Now efficiency, (or lack of perfect efficiency rather), in turning heat input into laser output means that in practise the surface area radiating into the lasing medium won't match the equivalent space radiator area. Exact ratio is a bit beyond my means to figure out i think, but in short each cubic meter may well be worth only a few percent of it's internal surface area. However since the internal surface area is so huge each cubic meter of volume dedicated to the laser radiators may be worth several square meters of surface radiator area.

So lets say each cubic meter of laser radiator volume gives thermal dissipation equivalent to three square meters of hull surface area. Just an example number pulled out of thin air, i'd expect higher numbers at a guess TBH.

Anyway if one cubic meter of laser radiation is equivalent to three square meters of surface area then an extra 2400 square meters of surface area cooling equivalents laser radiators requires 2400/3 = 800 cubic meter's of laser radiation.


Bearing in mind also how square cube law would otherwise limit conventional surface radiator area more than internal volume would the laser radiator concept becomes exponentially more powerful as size increases as it lets you overcome normal square cube law limits to a degree. The smaller ship will still have a better power density but every little will help obviously.


Does that all make sense now?



@Mr_Mask: Give me time to read and write and i'll get back to you, may be morning though when i do, getting late here.

Stellar_Magic
2015-06-24, 05:30 PM
Honestly the energy efficiency problem with lasers is one reason I doubt they'd be really effective in space. Most of the stuff I've read indicate that a lasers are generally only around 30% efficiency. This means that in order to destroy a ship with a laser blast, the ship that's firing has to deal with over twice the amount of energy used to destroy the target as heat buildup inside their ship.

In which case, that sounds like a terrible weapon.

Hence why I think kinetic kill weapons would stick around... as they don't discharge nearly as much of their energy as excess heat. The problem with those weapons is that their effective combat range would be absurdly short compared to a laser. Plus kinetic kill weapons are probably also something that can be countered through point defense, and without mounted engines on the projectiles... mere evasive action would be a very effective counter at range.

Mr. Mask
2015-06-24, 05:57 PM
Carl: Thanks Carl. Looking forward to it.


Stellar: I read an interesting article on that while researching lasers. Some lasers have pretty low efficiency, but it depends how you create the laser and how they go about it. I think I heard of a laser that got as high as 80%? It was pretty hot, but not the hottest laser by far. It does seem to indicate that for a scifi setting, or just the future in general, lasers may improve enough to be applicable weapons for spaceships.

Carl
2015-06-24, 06:11 PM
@Stellar: You're not far wrong there, though heatsinking some of it, Chemical Laser's, (or other open cycle cooling methods), and simply having the bigger ship would all help here.

They're likely to be more useful for long range missile intercepts and as a way to ensure the enemy can't build ships with flimsy radiators.


Actually my one attempt at a mostly serious ship design, the engine core would probably be unworkable thermally, but that's the price of building a ship with thick armour and a usable acceleration, (D/V was bad though, only about 80KPS), used primarily kinetics. In order the weapons systems where:

IR free electron lasers. These provided long range missile defence and help kill smaller ships.

Missiles with fusion drives and nuclear warheads form the long range arm, it's primarily an harassing weapon unless you have major numbers superiority, but handy for finishing off cripples and they can distract a lot of PD fire in the final engagement phases.

The close in PD is provided by railguns, there's a possibility, (remote but possible), that you might be able to build the rounds as inertia on impact activated gun type fission device. it would require some kind of sabot to hold the two pieces apart under launch stress that then falls away leaving the two lumps of sub critical material and in a set of guides with only their matched inertia's holding them apart. On impact newton takes over and blam. Despite their power they won't usually kill a warship, (did i mention these ships are big with hundreds of meters to a full km of armour on them, hence the drive core issue), but they're hell against anything else.

The big close in gun is a spinal mounted kinetic weapon firing a very large mass of material, (non-nuclear, you don't want to know what a neutron bomb would do to such a large mass of nuclear material).

Interception of such items as well as the weapons used if the big guns miss and the two sides pass broadside to broadside is big nasty missiles with armoured noses and shaped nuclear charges. Given how both sides throw out the PD fire, decoy's, jammers, and all the other crap and how they're the main means of defence against the big kinetic projectiles as well all added onto how they push the bleeding edge of acceleration, (which limits their pointing accuracy at distance), they're fused only for contact, or very close proximity. Anything more than the jet will probably miss due to all of the above. And even then you need massive amounts of fire to get shots through, which is where the standard missiles help. Almost certainly chemically propelled to provide the required acceleration so limited range.

The really nasty stuff is the stuff only found on really big ships, maybe even limited to bases. Giant Nuclear mortars. They're effectively one shot shaped nuclear charges contained in a liner that allows for a disposable "barrel" allowing for a repeated use high pointing accuracy, (but slow loading), shaped nuclear weapon. Obviously would have to be entirely external so somewhat vulnerable to damage.


Basic design is two cones with a center section. PD, Lasers, and the big nasty close range missiles are on a central cylindrical section, (most of the length is the cones though), the cones have much thinner armour than the central section but consist of multipule "bands" that rotate, (with counter-rotation too), to smeaer streams of kinetic PD fire and laser fire across their surface, (the angle also add's armour depth and increases laser spot area)

Brother Oni
2015-06-24, 06:58 PM
What I describe above sounds quite absurd and godlike. Which is the credit I am giving modern military, that any dragon capable of overpowering them unscathed must be an insanely powerful thing.

The dragon had better stay away from taking France then as I believe standing policy is that the nukes start flying if French sovereign soil is taken.

Of course given that your dragon is immune to 120mm APFSDS shells and radiation, fly at >600mph with only leather wings, can magically make a hurricane mask itself against radar and IR, plus defeat the safety mechanisms of a modern IBCM and reprogram the navigation system despite never encountering modern cryptography or computers before, you might as well make it immune to being nuked as well since we've left reality far, far behind.

As you've said, it's borderline ridiculously absurd and you're under estimating the willingness of certain countries to nuke such a massive threat, especially since civilian damage is minimal because the dragon's already killed them all.


Any thoughts on what a dragon would need to bring a modern army and nation to its knees?

Taking a leaf out of Shadowrun's book; its hoard, wisely invested into key corporations (defence and biotechnology for example), merging and purchasing other companies until it controls the single most powerful multinational corporation on the planet, then apply for citizenship and run for President/Prime Minister of any country it likes (it could probably claim naturalisation of any country since it pre-dates time immemorial (https://en.wikipedia.org/?title=Time_immemorial)).

Mr. Mask
2015-06-24, 07:31 PM
Not so much leather wings, as metallic armoured ones. It's questionable whether such a deific dragon would use its wings for flying. You could go with a more spirit like one and forego wings entirely.

The M829 isn't really qualified for dealing with naval vessels, its penetration being something like three feet of armour. I'm more concerned with missiles that are qualified for taking down warships, as either the dragon needs to evade or avoid their firing, or simply be invincible.

I'm pretty uncertain of a nation's willingness to nuke their own armies and cities, or to blanket their countryside in nukes. It's possible they may do so, though the political backlash would be a pretty interesting scenario of itself.

How good is radar in storms? I know IR is pretty short ranged in such cases.

Why bother decrypting the computers, when you have metaphysical characteristics? A more efficient way may simply be to say, "go," and the ICBM goes. And when it reaches it's target, the materials and mechanisms work by your very will. If you can command winds to come like some legends of dragons, it becomes very questionable as to how much you can and can't know and how much you can or can't do. In the example I gave, it may've simply been the dragon was very lucky in finding its way into one of the largest nuclear facilities in the area, and just happened to go that way and decide to stay there after scorching the earth to the point that the nation may well start blanketing it countryside or even civilian areas in nukes. Was it luck, or does the dragon know everything about modern human society, to the extent of the politics of thermal nuclear war? And if it can be seen as possible for it to know that much, perhaps mere days after awaking, and that it can bring the very winds of the world under its command, why is it so astounding that it may also be a skilled hacker?

Incanur
2015-06-24, 09:11 PM
I approve of "godlike" dragons, Mr. Mask. :smallsmile: The easiest way for a dragon to wreck circa-2015 Earth would be via trickery rather than brute force. 3.x D&D dragons can do this, for example: polymorph, dominate/bluff world leaders, etc. However, such subterfuge likely defeats the purpose of your scenario. The dragon you describe sounds up to the task - potentially even against nuclear weapons because of its speed and armor.

Other potential options:

Lightning breath - This could have a range of many miles; even actual lightning bolts can be as long as 118 miles. Lightning breath could additionally function as point defense against missiles and whatnot. Such lightning breath would make the dragon effectively immune to even nuclear weapons if it could shoot the nuclear missiles or nuclear bombers down multiple miles away.

Teleportion - Perhaps the dragon can disappear in a cloud of flame or transform itself into a massive bolt of lightning. This ability likewise protects the dragon from nuclear weapons.

Fear aura - Even the bravest [mundane] soldiers simply cower as soon as they see the dragon. Thus, normal folks cannot effectively engage it.

rs2excelsior
2015-06-24, 09:47 PM
Honestly the energy efficiency problem with lasers is one reason I doubt they'd be really effective in space. Most of the stuff I've read indicate that a lasers are generally only around 30% efficiency. This means that in order to destroy a ship with a laser blast, the ship that's firing has to deal with over twice the amount of energy used to destroy the target as heat buildup inside their ship.

In which case, that sounds like a terrible weapon.

Hence why I think kinetic kill weapons would stick around... as they don't discharge nearly as much of their energy as excess heat. The problem with those weapons is that their effective combat range would be absurdly short compared to a laser. Plus kinetic kill weapons are probably also something that can be countered through point defense, and without mounted engines on the projectiles... mere evasive action would be a very effective counter at range.

I won't get too far into the Purple vs. Green debate here--just suffice to say that I think both lasers and kinetics will probably have a place in space combat--but I do have one thing to say regarding your analysis of lasers.

Yes, a laser that's 30% efficient puts twice as much energy into your ship as the enemy's... but the type of energy is important. The waste heat generated by a laser can be channeled and radiated away (or even dumped via coolant) fairly easily... the energy from a laser, not so much. The fact that it is coherent and focused on a small area makes a huge difference.

A powerful flashlight can dazzle you briefly, while a laser pointer can permanently blind you if it's strong enough. It's the same kind of thing.

Brother Oni
2015-06-25, 02:28 AM
I'm more concerned with missiles that are qualified for taking down warships, as either the dragon needs to evade or avoid their firing, or simply be invincible.

Just have it be invincible and be done with it, no need to go into the specifics of whether its skin is tougher than 460mm of RHA (for reference, the Iowa class battleships had armour at most effectively 370mm thick and your listed APFSDS shell will penetrate that).

Your dragon is a plot device, not something to be fought - if you stat it out or quantify its abilities, your PCs will try and figure out a way to kill it and if they're anything like some players I've gamed with, probably succeed, thus you'll need DM fiat for it to survive.



And if it can be seen as possible for it to know that much, perhaps mere days after awaking, and that it can bring the very winds of the world under its command, why is it so astounding that it may also be a skilled hacker?

There's a logical fallacy that presumes because something is good at X, it is also good at unrelated field y (I forget the name of it). As mentioned, just make your dragon a plot device and give it whatever powers you need.

I have no objection to a god like dragon ruling the Earth and defeating all comers as a plot device. I simply see no point in trying to defining that further - you've mentioned that it's apparently weak against antiship missiles - the Harpoon has a mere 221kg HE warhead. You don't think something like a MOAB (https://en.wikipedia.org/wiki/MOAB) with a blast yield of 11 tons of TNT, detonated anywhere near it would kill it with the shockwave alone?

Please don't get hung up on the examples like you usually do (eg. the APFSDS point), but consider the main point I'm trying to make - statting it out will only make PCs try and kill it. Make it a character and people will only scream 'Mary Sue' at it as an epithet.

Edit: Some additional information to answer your earlier questions as this is the real world weapons, armour and tactics thread: the reason why they don't normally use kinetic penetrators against battleships, is that punching a neat hole in a ship isn't going to do much damage to it as most of it is empty space, which can be sealed off by bulkheads to contain fires or incoming water. Presumably this isn't the case with an ostensibly biological organism (which incidentally also opens the door to concussive blasts and shockwaves causing damage, which is very difficult for armour to defend against).

As far as I can tell, the frequencies that would be used in radar guided munitions and air defence radars are not significantly affected by adverse weather and would still find and hit your dragon, particularly if it's flying around in the eye of its hurricane (unless it's also immune to the winds it causes) and its comparatively dense metallic form would give a massive radar bounce even if the weather did affect radar resolution (unless that somehow gives it stealth capabilities).
Given the speed of the newer antiship missiles (>5,000 mph) the window of detection and reaction is very small - assuming it's on the ground and its eyes are 20m off the ground, distance to the horizon is ~16km, thus from the time the missile is even visible, it has 6.7 seconds to evade accordingly (don't forget the missile will adjust trajectory to hit an evading target).

Edit 2: Also forgot about immunity to other NBC class weapons - even if a country is unwilling to go nuclear or biological, chemical weapons won't have the same reluctance and with the death toll potentially in the millions, resistance to the former two will be low enough as it is.

Carl
2015-06-25, 04:05 AM
Oni hit the high points Mask. And yes faced with such a massive threat to their continue existence out flying around destroying their cities they would go nuclear. More to the point they'd be willing to go to excess if they really, absolutely had to. Whilst i've never heard of a laydown plan anything like this and i doubt one's ever been planed it's totally within our capabilities to drop large numbers of nukes with variable placements and detonation heights such that an area ranging a 100 miles in diameter from ground level to the upper limits of the outer atmosphere is encased entirely in the overlapping fireballs of the nukes with an out to in detonation pattern so closely coordinated it forms a focused blast front at it's center turning the dragon into an oversized chunk of fusible/fissile material in the center of the biggest fusion/fission bomb ever built.

And then there's nuclear shaped charges.

The whole concept actually has some similarities with the dragons from my white wizard setting i was asking questions about artillery for a little while ago. The Dragoons there are virtually as tough, (i needed the artillery figures to figure out an equivalent in energy usage for the white wizards at the top ed so i could get a handle on what else might or might not bother a dragon), for you as mine, were talking high fractional gigaton range directed bursts just to kill them. Outright obliteration is in gigaton range numbers.

Mr. Mask
2015-06-25, 07:01 AM
Incanur: Glad you also find the idea interest, Incanur. It's true that trickery and economics are more conceivable ways for the dragon to prevail, but as you say it wouldn't fit the dragon scouring the land theme.

A thought I had on the dragon's speed, if it can control storms and wind, it might be able to ride on a hurricane, more or less. Since it controls the wind, turbulence probably wouldn't be an issue. If the dragon has some means of advanced detection (perhaps the miles of surrounding air itself works like a kind of spiderweb or nervous system?), it could also fold up its wings in preparation for a hit, making them less vulnerable (of course, at this point it's questionable if it need wings, or if it can't just regenerate them). The wind can also help to deflect or slow incoming attacks (it may have other utilities I've failed to consider).

For armour, for now I'm thinking the dragon is simply very, very large, its scales and flesh are some magnificent, divinely empowered material. It may well have properties that seem impossible, its response to kinetic energy being unlike any mundane material's. There are a lot of fun tricks you could try. Someone gave me the idea of what if the material was super dense, like the core of a star, and I am very curious as to how explosives and conventional weapons would react to a material of that density.

Your ideas related to lightning are excellent points. I was thinking of something like a volcanic explosion of a weapon, but lightning would be fitting into the theme of storms and would give it excellent capabilities as you note. I had a similar idea of a fire elemental, who could heal injuries by converting their mass into fire and back again. That'd be the simplest way to make the dragon seem immortal, that its body can reform out of the very storm even if it were scattered to bits. While this seems the optimal solution, I'll still consider the idea of a more mundane body, just because I'm interested in exploring the subject.


Oni: The players would be mercs and adventurers, without access to high grade weaponry and nukes, so even if they worked out a plan that would work, it would be a campaign of itself to get the tools to try it out and put it into action. In itself, I'm interested in how invincible the dragon would need to be.


There's a logical fallacy that presumes because something is good at X, it is also good at unrelated field y (I forget the name of it). Well, that's not my argument. My statement is simply that the assumption that a deity is old, and so can't be good at skills associated with modern times, seems like an unfound assumption based off false correlations. Indeed, the dragon may have no powers over modern electronics, but I don't think it would be correct to assume upon ignorance it had failed to show thus far (if it knows where your military bunkers are, it's questionable what it doesn't know). If you met someone who could say, "come," and the winds come at their command, what could we say they do or don't know?


I have no objection to a god like dragon ruling the Earth and defeating all comers as a plot device. I simply see no point in trying to defining that further - you've mentioned that it's apparently weak against antiship missiles - the Harpoon has a mere 221kg HE warhead. You don't think something like a MOAB with a blast yield of 11 tons of TNT, detonated anywhere near it would kill it with the shockwave alone? Well, defining it further is something I consider to be an interesting though exercise. If a deity like being came down and killed several thousand people, it'd make sense to be curious as to how.

With things like the MOAB, I was figuring the dragon could obscure it's position to prevent a good lock for the missile, using electrical interference and high winds to make sure the missile never gets close enough to try its luck. Incanur's mention of a lightning weapon would also give it a form of point defence it could use.


Please don't get hung up on the examples like you usually do I'm sorry if I've failed to behave appropriately, but I don't know how to modulate my behaviour based off of this.


Some additional information to answer your earlier questions as this is the real world weapons, armour and tactics thread: the reason why they don't normally use kinetic penetrators against battleships, is that punching a neat hole in a ship isn't going to do much damage to it as most of it is empty space, which can be sealed off by bulkheads to contain fires or incoming water. Presumably this isn't the case with an ostensibly biological organism (which incidentally also opens the door to concussive blasts and shockwaves causing damage, which is very difficult for armour to defend against). Mm, this is what I meant with the APFSDS not being capable of use against ships. While a normal animal has little ways of treating wounds, and so they often become fatal quickly or in the long term, I'm figuring a creature of truly immense size and magical power will shrug off what seem mortal wounds (similar to Sagas and other legends). While a suitably large shockwave might be fatal to the dragon, I figure it's missile defence would prevent it being that close to such a shockwave.


As far as I can tell, the frequencies that would be used in radar guided munitions and air defence radars are not significantly affected by adverse weather and would still find and hit your dragon, particularly if it's flying around in the eye of its hurricane (unless it's also immune to the winds it causes) and its comparatively dense metallic form would give a massive radar bounce even if the weather did affect radar resolution (unless that somehow gives it stealth capabilities).
Given the speed of the newer antiship missiles (>5,000 mph) the window of detection and reaction is very small - assuming it's on the ground and its eyes are 20m off the ground, distance to the horizon is ~16km, thus from the time the missile is even visible, it has 6.7 seconds to evade accordingly (don't forget the missile will adjust trajectory to hit an evading target). I figure the dragon is immune to its own wind, in the fact it can make the wind go with it and not against it. I was thinking that the dragon may have capabilities similar to a stealth plane, or as much as it can have with its shape.

Mm, I agree that normal eyesight would not allow the dragon to react accordingly. I was thinking the wind and storms would act as a nervous system for the dragon, so it can detect any fast moving object for a hundred miles in all directions. It may also be able to detect exit and re-entry in the Earth's magnetosphere by the same mechanism (if its nervous system is really connected to the storms). Of course, something to consider with normal eyesight, is the missile isn't likely to be on course en-route to the dragon. As far as I'm aware, no missile can go through a hurricane or tornado and stay on course.


Thanks for discussing this with me, Oni. I don't think I've shown it properly in the past, but I do really appreciate the help you've often given me, and I'm sorry for my tendency to argue or become vindictive.


Carl: Perhaps I am underestimating the willingness to go nuclear. I was thinking that obscuring its location, and sending missiles off course, and rapidly locating itself near to populated areas and armies, it would make a nuclear response seem questionable enough to prevent it. The dragon would probably destroy something short of ten major cities in total, so it may be necessary for the story that one or more nukes were launched, but the dragons defences were capable of preventing the dragon's annihilation.

If a plan to destroy a 100 mile area was formed against the dragon, I expect that even an ICBM will not remain on course against a suitably typhoon-like force directed against it.

Speaking of outright obliteration, I was curious about how super dense matter like that at the core of a star would react to explosives. At the point of obliteration, will any matter within the area of effect be obliterated to the point where it makes no difference what the matter was, whether cotton or the core of a star?

Incanur
2015-06-25, 07:01 AM
Because of the inverse-square law, theoretically an extremely durable entity might require a rather close hit to defeat with a nuclear weapon. At that point, blanketing an area with nuclear detonations would be unlikely to succeed against a mobile target.

I guess a nuclear weapon on a guided hypersonic missile would have better odds, perhaps like the designs China has apparently been testing recent. But lighting-breath point defense would still work. Also, assuming the dragon would be on the ground and limited to visual perception seems unlikely in this scenario.

Personally I think I'd go with storm dragon capable of creating its own hurricane filled with lightning that fries any incoming missiles. That right there should be sufficient, really. High-end particle-beam point defense beats missiles. :smallwink:

Brother Oni
2015-06-25, 09:01 AM
If you met someone who could say, "come," and the winds come at their command, what could we say they do or don't know?

It's like assuming Oedipus, who was innately good at riddle and word play, would be able to easily crack the Enigma code (https://en.wikipedia.org/wiki/Enigma_code) or even a one-time pad (https://en.wikipedia.org/wiki/One-time_pad), except that your argument is apparently 'it can summon the wind, therefore he has automatic mastery over something that wasn't invented until centuries after it went to sleep'.

If you want to fiat it, please go ahead, but don't try and find a logical reason for it.


Well, defining it further is something I consider to be an interesting though exercise. If a deity like being came down and killed several thousand people, it'd make sense to be curious as to how.

Make a list of what weaponry a modern military has (particularly AAA defences and other fast, highly destructive weapons) and calculate accordingly.

You've mentioned that it's wandered around destroying several cities in the original scenario - given that a settlement isn't typically classed as a city until it hits a population of about 150,000, several thousand people is a very low mortality rate. For increasing its threat level and providing a greater sense of scale, I'd up the death toll by a least an order or two of magnitude - if it hit a major metropolis like London, a mere 1% death toll is over 83 thousand people.


I'm sorry if I've failed to behave appropriately, but I don't know how to modulate my behaviour based off of this.

Due to my training, I tend to offer examples as a way of illustrating my point. Some times these examples are not the best ones and we end up arguing over the examples and not the original point I was trying to make.



Mm, this is what I meant with the APFSDS not being capable of use against ships. While a normal animal has little ways of treating wounds, and so they often become fatal quickly or in the long term, I'm figuring a creature of truly immense size and magical power will shrug off what seem mortal wounds (similar to Sagas and other legends). While a suitably large shockwave might be fatal to the dragon, I figure it's missile defence would prevent it being that close to such a shockwave. I figure the dragon is immune to its own wind, in the fact it can make the wind go with it and not against it. I was thinking that the dragon may have capabilities similar to a stealth plane, or as much as it can have with its shape.

The problem is, you're asking for specifics grounded in real life and we're pointing out that the scenario isn't even possible (let alone the specifics) if you use the constraints of real life.

You've said that the dragon can create hurricanes, which means upwards of 75+ mph winds. While I agree that if it has that sort of innate control over the wind, it would be immune to the effects of the storm, it's still not going to even slow a modern missile. I'm finding it hard to locate references for this though, simply because warfare is typically not conducted during hurricane force winds, but bear in mind that such missiles have in built course direction.

You've said that the winds can act as a detection system - how is that going to help against a hypersonic missile travelling at several times above the speed of sound? Air simply can't propogate the disturbance back to the dragon faster than the missile (much like space combat getting a bit weird when distances are measurable in light minutes).

While disturbances in an electrical field could possibly be detected, this implies highly charged air, which means lightning and massive amounts of rain. This also means there is an upper limit to how far it can extend its effects, thus something like a MIRV or other re-entry vehicle could bypass it. Given that the storms are unable to connect to the magnetosphere that high up, it's likely that it won't receive any warning until it penetrates the cloud layer, by which point it may be too late for evasion.

Given that you haven't stated what your dragon looks like yet, I'll reserve judgement on any possible stealth capabilities, but assuming it doesn't look like a stealth bomber, it's going to have a measurable radar signature as both shape and material are important to achieving stealth. Again as a biological organism, it's going to be remarkable dense, especially fortified with metals.



Thanks for discussing this with me, Oni. I don't think I've shown it properly in the past, but I do really appreciate the help you've often given me, and I'm sorry for my tendency to argue or become vindictive.

I apologise if I come across as overly argumentative. I personally enjoy fiction very much grounded in reality and scenarios which basically use a Hollywood level understanding of physics (no offense intended) tend to annoy me.
I'd like to point put again that I have no issue with an all powerful, all conquering dragon - just don't try and quantify it.

Even Incanur's suggestion of a constant lighting storm acting as a particle beam defense can be defeated - fire a railgun at it or other high speed inert projectile, or daisy chained nukes (the first detonated at the edge of its the storm to clear it, the subsequent ones to hit the dragon). I seriously doubt a hurricane can generate more force than a nuclear detonation.

Incanur
2015-06-25, 09:53 AM
I'd like to point put again that I have no issue with an all powerful, all conquering dragon - just don't try and quantify it.

I used to frequent sf/fantasy versus boards, so I'm a fan of trying to quantify fictional entities. As a writer, I find this useful for understanding the fictional universe. So much speculative fiction doesn't take the implications of magic/technology seriously.


Even Incanur's suggestion of a constant lighting storm acting as a particle beam defense can be defeated - fire a railgun at it or other high speed inert projectile, or daisy chained nukes (the first detonated at the edge of its the storm to clear it, the subsequent ones to hit the dragon). I seriously doubt a hurricane can generate more force than a nuclear detonation.

I don't know exactly how much energy it would take, but clearing a hurricane via nuclear weapons would be far from easy. Actual hurricane are typically *huge* in a way that nuclear detonations aren't: we're talking about hurricane-force-wind radius of 100+ miles, depending on the storm. Your average hurricane (https://en.wikipedia.org/wiki/Orders_of_magnitude_(energy)) releases 6e+14 J (143 kilotons) per second because of its sheer scale. I haven't done the math, but I'm pretty sure any attempt to clear a hurricane with nuclear denotations would require so many nukes that it'd basically doom civilization for decades. I'm not even sure enough nuclear weapons current exist. Of course, the dragon may well not have a full-size hurricane. In any case, clearing the hurricane wouldn't work for lightning that comes directly from the dragon. If it can zap nukes at 5+ miles and has decent durability, nuking it ain't going to work.

Mr. Mask
2015-06-25, 10:15 AM
It's like assuming Oedipus, who was innately good at riddle and word play, would be able to easily crack the Enigma code or even a one-time pad, except that your argument is apparently 'it can summon the wind, therefore he has automatic mastery over something that wasn't invented until centuries after it went to sleep'. Was Oedipus divinely good at world play and riddles? I recall him answering the Sphinx's riddle, but I didn't recall anything of him being supernaturally or divinely good at them. If someone's had a divinely inspired ability to answer riddles, I wouldn't presume what they couldn't crack. Perhaps such a person can't crack it, but such an assumption seems to have a loose foundation related to false correlations (elders tend to be slow to adopt new technology, etc.).


You've mentioned that it's wandered around destroying several cities in the original scenario - given that a settlement isn't typically classed as a city until it hits a population of about 150,000, several thousand people is a very low mortality rate. For increasing its threat level and providing a greater sense of scale, I'd up the death toll by a least an order or two of magnitude - if it hit a major metropolis like London, a mere 1% death toll is over 83 thousand people. Oh, sorry, the several thousand people was a reference to cases from mythology and religion. In the dragon's case, I figure it hits something short of ten major cities, and a death toll somewhere around thirty million people, by the end of it.


Due to my training, I tend to offer examples as a way of illustrating my point. Some times these examples are not the best ones and we end up arguing over the examples and not the original point I was trying to make. I'm sorry if I've had a habit of taking the example too literally.


The problem is, you're asking for specifics grounded in real life and we're pointing out that the scenario isn't even possible (let alone the specifics) if you use the constraints of real life.

You've said that the dragon can create hurricanes, which means upwards of 75+ mph winds. While I agree that if it has that sort of innate control over the wind, it would be immune to the effects of the storm, it's still not going to even slow a modern missile. I'm finding it hard to locate references for this though, simply because warfare is typically not conducted during hurricane force winds, but bear in mind that such missiles have in built course direction.

You've said that the winds can act as a detection system - how is that going to help against a hypersonic missile travelling at several times above the speed of sound? Air simply can't propogate the disturbance back to the dragon faster than the missile (much like space combat getting a bit weird when distances are measurable in light minutes).

While disturbances in an electrical field could possibly be detected, this implies highly charged air, which means lightning and massive amounts of rain. This also means there is an upper limit to how far it can extend its effects, thus something like a MIRV or other re-entry vehicle could bypass it. Given that the storms are unable to connect to the magnetosphere that high up, it's likely that it won't receive any warning until it penetrates the cloud layer, by which point it may be too late for evasion.

Given that you haven't stated what your dragon looks like yet, I'll reserve judgement on any possible stealth capabilities, but assuming it doesn't look like a stealth bomber, it's going to have a measurable radar signature as both shape and material are important to achieving stealth. Again as a biological organism, it's going to be remarkable dense, especially fortified with metals.

Well, I've heard some experts speak confidently that strong wind will effect a missile's course. Someone stated specifically that no missile guidance system was good enough for navigating a hurricane accurately.

For detection, I was meaning wind and storm. That is, electrical signals too. So, if it can process the information quickly, it could be alerted to changes in the atmosphere at the speed of natural lightning. This does make me wonder how fast an air disturbance travels when caused by a hypersonic missile, though.

The dragon may need to use other methods to prevent radar detection. It may be able to unleash a storm of chaff material amongst its wind, making radar useless.


I apologise if I come across as overly argumentative. I personally enjoy fiction very much grounded in reality and scenarios which basically use a Hollywood level understanding of physics (no offense intended) tend to annoy me.
I'd like to point put again that I have no issue with an all powerful, all conquering dragon - just don't try and quantify it. There's nothing to worry about, I'm glad you're interested in discussing it with me. Though, I do find it interesting trying to quantify these things.


Even Incanur's suggestion of a constant lighting storm acting as a particle beam defense can be defeated - fire a railgun at it or other high speed inert projectile, or daisy chained nukes (the first detonated at the edge of its the storm to clear it, the subsequent ones to hit the dragon). I seriously doubt a hurricane can generate more force than a nuclear detonation. Well, this may explain the misunderstanding of the dragon's abilities. Hurricanes generate a lot more force than nuclear detonations. There was one storm in particular that was calculated to have been worth 11,000 Hiroshima bombs.

Incanur
2015-06-25, 11:55 AM
Well, this may explain the misunderstanding of the dragon's abilities. Hurricanes generate a lot more force than nuclear detonations. There was one storm in particular that was calculated to have been worth 11,000 Hiroshima bombs.

Yes, but note that this is because of the hurricane's scale and duration. The energy density of a nuclear detonation is of course far, far higher.

Mr. Mask
2015-06-25, 12:22 PM
Oh sure, hurricanes have their energy dispersed over a wide area. But if a being has the ability to control this energy so as to make it denser, to create tornadoes and the like, the amount of energy they can muster becomes deific in nature.

Telok
2015-06-25, 04:55 PM
Because where talking about a warship
Ok, a warship. You're sacrificing efficency and effectiveness for durability. That makes more sense as long as we assume that the ship takes hits that damage external radiators without major loss of combat ability.


So lets say each cubic meter of laser radiator volume gives thermal dissipation equivalent to three square meters of hull surface area. Just an example number pulled out of thin air, i'd expect higher numbers at a guess TBH.Let's do a quick check here and assume that you get an incredible efficency of 30 square meters of radiator surface equivalent per cubic meter. So you are replacing one retractable 3x5 meter external radiator with one cubic meter of internal laser stuff. Sounds good.


Bearing in mind also how square cube law would otherwise limit conventional surface radiator area more than internal volume would the laser radiator concept becomes exponentially more powerful as size increases as it lets you overcome normal square cube law limits to a degree.
I'm not sure what you're saying here. How does the square cube law limit normal radiators but not the laser thing?

Actually the only issues I see with this now is the space combat assumptions you're making ("broadsides", contact nukes, lasers that damage external components but not the hull, unguided kinetic kill munitions (http://www.projectrho.com/public_html/rocket/spacegunintro.php)) and the special material of the laser. Essentially you're turning heat into laser light, which is already how things are done (oil/coal/nuclear powerplants->electricity->lasers), in a single step at extremely high efficency. So all you're doing there is positing a wonderfully useful material that is completely unknown to current science.

Carl
2015-06-25, 04:58 PM
@Mask: the problem comes in that the dragon represents a clear danger on par with the use of Nuclear weapons. One of the scenarios that is guaranteed to make a country use it's nukes is one in which it feels it's continued existence is under threat. get something like a Dragon that's displaying a propensity for treating the whole of the human race liek that and everyone would pitch in on nereal self interest principles.

MIRV's are incredibly high momentum projectiles, especially until they hit the lower parts of the atmosphere and they're small. Wind could deflect them, but i doubt a hurricane could do more than a slight knock of course easily accounted for by tightening the tolerances on the overlap. Now Tornado strength winds could get interesting, but i wouldn't put it past them to punch through given how little time they spend in thick atmosphere. And of course shaped nuclear charges largely render that irellevent. They care about atmosphere, but not too much about winds. And they care about atmosphere in the sense that LEO to ground reduces them from nuclear airburst to merely massively destructive. I also suggest you look up the results of the castle bravo test, it gives an idea of how total the destruction within a certain radius of a nuke is. A combination of lagoon, (and going from air to seawater is a great shock wave absorber never mind waters excellent thermal properties), and silt/rock seafloor failed to stop it boring a multi kilometer crater into mother earth. Obviously sub grade device is going to have a much smaller radius but get fireball to fireball lay down and that's the kind of firepower your dragon would have to survive being hit with.

Of course if i got really desperate and nuclear shaped charges and mass laydowns didn't work i'd probably try luring it to yellowstone and set of a nuke there to knock the top off the magma chamber there. Very risky as it would kill most of mankind, but well, that a ot of magma, anything that survives that could probably survive anything less than being dumped into the sun.

@Telok, you posted while i was typing, give me a few minutes.

Carl
2015-06-25, 05:19 PM
I'm not sure what you're saying here. How does the square cube law limit normal radiators but not the laser thing?

If you double the size of a given shape the surface area goes up 4 times but the volume goes up 8 times. Meaning that a larger design has far less gain in surface area than it does volume.

This if we scale the 20m cube upto 40 m and keep the laser radiators working at 10% of volume your actual surface area only increases to 9600m^2 but your laser radiator area increases to an equivalent of 19200m^2 meaning your still getting a cooling area to volume ration of 0.75 to 1 instead of the 2 to 1 you'd get otherwise.


Actually the only issues I see with this now is the space combat assumptions you're making ("broadsides", contact nukes, lasers that damage external components but not the hull, unguided kinetic kill munitions) and the special material of the laser. Essentially you're turning heat into laser light, which is already how things are done (oil/coal/nuclear powerplants->electricity->lasers), in a single step at extremely high efficency. So all you're doing there is positing a wonderfully useful material that is completely unknown to current science.


no, no, no, no.

Not light. IR. There's a difference and it's really bloody important.

Let me repeat for you. You cannot turn waste heat into work, (in a conventional sense, large amounts of waste heat are destructive to most things but that's about the only thing it can do). You can only get rid of it. But as long as you're getting rid of it how you do it, (i.e. give off hot gas, radiate it as broad spectrum radiation, or radiate it as coherent radiation from a laser radiation), doesn't really matter.

Now true such a substance would be useful, but not as much as you think. Again we come back to the point made in the links. When you have the option of open cycle cooling like a power station cooling tower or conduction into another material, (like in a car radiator), you can radiate much more heat per unit area of radiator.

Also i'm not assuming lasers that will damage radiators but not armour. I'm, assuming armour with heat dissipation at a minimum approaching the highest figure i ever found sourced, which was for the SSME's at NASA which is depending on the part 10-15KJ per square centimeter. Bearing in mind possible rotation of ship and/opr armour and minimun laser spot sizes for most common lasers you start needing unfeasibly large and powerful lasers to rapidly overcome the system. you either have to max out the targets cooling capabilities of settle for wear and tea similar to what an SSME suffers and burn through modest thicknesses over hours. That's the real issue with lasers. The target never absorbs 100% of the energy directed at it, and between ablative material's, active cooling loops the energy requirements quickly spiral to uncontrollable levels. As Steller points out the biggest issues is that the firing ship is always generating more hat than it's putting into the target which hard caps the power, so even if it can overwhelm, if the armour is thick enough the shooter may have to stop firing before he burns through due to heat problems.

Straybow
2015-06-25, 09:26 PM
Nobody has mentioned bomb-pumped lasers yet. The nuclear warhead can be destroyed long before it gets close enough to do damage, but a bomb-pumped laser warhead need only close within ten of kilometers. Stealthy versions could be fired from a railgun and use cold gas thrusters for aiming, thus having a heat signature too low to be detected in time to react. In the Honorverse they aren't stealthy because ships and missiles maneuver at 100s or 1000s of gee, but in a more conventional physics melieu, unpowered bomb-laser missiles could be very effective.

Carl:
The problem with IR laser heat dissipation is you still don't have an effective way of moving the waste heat into the lasing medium. If you absorb it into a fluid and pump it away, then you've lowered the temperature and made it even less useful for powering your laser. You would essentially have to build your engines and weapons, or at least their high-temperature radiators, inside IR lasers to get the waste heat into them with enough thermal differential to power the laser.

Lvl 2 Expert
2015-06-26, 02:47 AM
Honestly the energy efficiency problem with lasers is one reason I doubt they'd be really effective in space. Most of the stuff I've read indicate that a lasers are generally only around 30% efficiency. This means that in order to destroy a ship with a laser blast, the ship that's firing has to deal with over twice the amount of energy used to destroy the target as heat buildup inside their ship.

In which case, that sounds like a terrible weapon.

Hence why I think kinetic kill weapons would stick around... as they don't discharge nearly as much of their energy as excess heat. The problem with those weapons is that their effective combat range would be absurdly short compared to a laser. Plus kinetic kill weapons are probably also something that can be countered through point defense, and without mounted engines on the projectiles... mere evasive action would be a very effective counter at range.

Bullets of any kind deposit just as much kinetic energy into the firing ship as they do into the target, and yet the shipbuilders dealt with this. Excess heat in space is tricky, much trickier than recoil, but there are ways to handle it if the weapon makes it worth it (which may be sort of doubtful, while a laser is very easy to aim, it would also be pretty easy to deflect). For instance, maybe the ship has stocked up on a substance with a high heat capacity, pre-cooled to near absolute zero (even if cooling is suboptimal as outlined one post above this one).

Any spaceship produces excess heat, and while I don't know how they do it I don't think the ISS burned up yet.

Brother Oni
2015-06-26, 07:59 AM
Perhaps such a person can't crack it, but such an assumption seems to have a loose foundation related to false correlations (elders tend to be slow to adopt new technology, etc.).

My issue is that most skills and knowledge are hierarchal (you have to have an understanding of the basics before moving on to more advanced concepts), not that elders are slow to adopt new technologies (my father in-law used to work in Sharp R&D and knows more about technology than I do).

A false correlation would assuming somebody like Leonardo da Vinci wouldn't be able to understand the internal combustion engine; given his intimate understanding of mechanics, he would be able to pick it up quite quickly. However he would however be much harder pressed to understand something like an ion thruster (https://en.wikipedia.org/wiki/Ion_thruster) without significant help since his knowledge base is non-existant in that field (that field of knowledge didn't exist in his day!).

Oedipus (or some other divinely influenced hero who was good at word play) would be capable of understanding the concept of cryptography (even if I think he wouldn't be able to break modern cryptographic methods without significant help) and I can just about swallow his divine influence letting him crack codes as there's still a logical basis for his powers to work, as the basic principle of a message being hidden behind other words/symbols is the same, be it by riddles, cryptic crossword clues or a 2048 bit RSA key.

As far as I can tell, your justification for your dragon to be able to defeat the accidental launch security measures of a IBCM is "it can summon the wind at will, who knows what else it can do?".



For detection, I was meaning wind and storm. That is, electrical signals too. So, if it can process the information quickly, it could be alerted to changes in the atmosphere at the speed of natural lightning. This does make me wonder how fast an air disturbance travels when caused by a hypersonic missile, though.

I was under the impression that the air disturbance would be capped at the speed of sound - from the wiki page on sonic booms, as an aircraft travels through air, it creates a pressure wave in the air in front and behind it. As it gets faster, the air simply can't get out of the way fast enough, merging these pressure waves together and creating the characteristic boom.

Lightning strikes propogate at something like 30% of the speed of light, so that would be near instantaneous. I'm not so sure on charged air through - that might be capped to how fast the air can move.


The dragon may need to use other methods to prevent radar detection. It may be able to unleash a storm of chaff material amongst its wind, making radar useless.

While I can understand physical objects getting picked up in the hurricane winds, causing radar fuzzing unintentionally, radar chaff is specifically masses of thin metallic strips and in a hurricane, they're likely to be scattered about very quickly, reducing or possibly negating their radar defeating effect.

It also begs the question, how does the dragon know about radar in the first place to be able to develop ECM?

Lvl 2 Expert
2015-06-26, 12:50 PM
It also begs the question, how does the dragon know about radar in the first place to be able to develop ECM?

It went to school with the dinosaur in the new JP movie who knows about infrared camera's. :smallcool:

Telok
2015-06-26, 03:19 PM
This if we scale the 20m cube upto 40 m and keep the laser radiators working at 10% of volume your actual surface area only increases to 9600m^2 but your laser radiator area increases to an equivalent of 19200m^2 meaning your still getting a cooling area to volume ration of 0.75 to 1 instead of the 2 to 1 you'd get otherwise.
Ok, what's the bolded part referring to? The rest is right but that doesn't seem connected to anything. Also your numbers are correct for solids like cubes but you might want to check them for thin radiator fins.


Not light. IR. There's a difference and it's really bloody important.
Please explain to me how infrared light is not part of the electromagnetic spectrum. This is going to require sources.


But as long as you're getting rid of it how you do it, (i.e. give off hot gas, radiate it as broad spectrum radiation, or radiate it as coherent radiation from a laser radiation), doesn't really matter.
Well, yeah, that's what I've been saying. The benefit to your system is that it's interior to the ship and under your armor. The drawback is that a) it's less efficent that normal methods, and b) your fancy armor is already twice as efficent as the ISS radiators (those radiate from both sides and the armor only radiates from one side) and resistant to kinetic, nuclear, and laser weapons. If you need more radiant area you can add fins or change the shape of the ship or just increase the size and volume without increasing the mass by putting empty spaces in the ship.

snowblizz
2015-06-27, 05:02 AM
It went to school with the dinosaur in the new JP movie who knows about infrared camera's. :smallcool:

Dude! SPOILERS! Some of us can't go to the cinema until it summer-opens on July 2.

Carl
2015-06-27, 05:41 AM
1. I think i may have reversed a part there. What i'm saying is that if you take the volume to surface area of a 20m cube as the baseline then by keeping the interiour ratio volume of the laser radiators constant when you scale the dimensions up to 40m your ratio of radiator area to volume drops such that your ratio is only 75% of it's prior value. if the old ratio is a baseline of 1 that means a ratio of 0.75 to 1. Without the laser radiators it drops to 0.5 to 1, screwed my numbers up there when stating it sorry.

2. Erm no, when you said light i assumed visible light which is a specific part of the EM spectrum. And yes it's not different in principle from doing it the other ways, except all of those involve using work output, not waste output to produce the laser. The goal here isn't to produce a laser because we want a laser. It's to turn the waste heat into a laser because that's something we can do internally rather than need exterior hull area.

3. Who said anything about efficiency compared to the ISS or anything about nukes or kinetics. Sure if you start using my design i put together you've got major kinetic and nuclear resistance. Whilst various factors, (namely explosive vaporisation of sufficient amount of material will create kinetic forces due to rapid expansion in line with newtons third law), mean you need less than the energy required to vaporise to the depth of the armour, if you have sufficient depths the energy requirements get huge. To the point low kiloton yields are survivable in massive quantities. A large megaton range shaped nuclear charge will bust through most things and the heavy kinetic described will do it any day of the weak, but only on direct hits.

And again we come back to the point that it's a warship and thus you can't mess around with frills to make better radiators, they're fragile. Think how you'd design a radiator for a tank but assuming you have to sue radiant heat only, no airflow.

@Straybow: If you have an actual lasing medium in there most of your energy input will be via conduction, (i'd assumed you'd all figured that out despite my tall of IR input, didn't think i had to state it, i'm not aware of any law that says such energy input can't pump a lasing medium), not radiation so whilst the low rejection temperature hurts it's not as bad as the spaced based radiators and you can ft a lot of surface area per cubic meter. The real limiter will be a combination of efficiency and maximum permissible energy contained in the medium at that efficiency. You can't put more energy in if the medium is allready at it's limit. But tha's getting into really complicaited teritory ajd i didn't want to confuse things anymore than i have to.

Brother Oni
2015-06-27, 08:41 AM
Dude! SPOILERS! Some of us can't go to the cinema until it summer-opens on July 2.

It's a Hollywood film. Bad science and characters knowing about modern technology they have absolutely no foundation or reason for to fill in plot holes, is par for the course. :smalltongue:

Straybow
2015-06-27, 01:39 PM
@Straybow: If you have an actual lasing medium in there most of your energy input will be via conduction, (i'd assumed you'd all figured that out despite my tall of IR input, didn't think i had to state it, i'm not aware of any law that says such energy input can't pump a lasing medium), not radiation so whilst the low rejection temperature hurts it's not as bad as the spaced based radiators and you can ft a lot of surface area per cubic meter. The real limiter will be a combination of efficiency and maximum permissible energy contained in the medium at that efficiency. You can't put more energy in if the medium is allready at it's limit. But tha's getting into really complicaited teritory ajd i didn't want to confuse things anymore than i have to.

You'll never reach the energy limit of the lasing medium. Radiating or conducting heat into the lasing medium is the same problem as radiating or conducting heat anywhere else. The temperature differential between the waste heat source and the sink (in this case, lasing medium) is the limiting factor. High temp waste heat can be used for some sort of co-generation, drawing off the useful energy. Whether you use the useful energy to help drive the laser, or for some other shipboard purpose, you still have the useless waste heat to throw off.

If you use high speed pumps to move cooling fluid, you transfer high temp waste heat from engines and weapons into lower temp fluid. The cooler the fluid, the more heat you remove from the sources. Using peltiers to increase the differential helps, but you still end up with the working fluid temperature limited to the average between the (augmented) source and the initial coolant temp. Using peltiers to again increase the differential between the coolant and the laser would again help, so go ahead and assume something of the sort is at work whenever transferring heat. You've still introduced more losses at each step.

No matter what method, you are still limited to passive radiation and surface conduction rates to dump heat, which depend on the limited temperature differential.

BTW, using a laser to dump heat isn't a new idea. Various discussions about Star Wars often invoke it to explain the glowing blue "exhaust" from ships, and dealing with waste heat of the enormous (read: total stellar output level) amounts of energy required to power the death star's main weapon. In somewhat harder scifi, David Brin used the idea in Sundiver, about a ship designed to dive into a star's "atmosphere."

Carl
2015-06-27, 02:56 PM
@Straybrow: I know all that but you seem to be making the mistake of forgetting what the alternatives are. Same as telok. Your talking a warship who's combat radiators at least have some very harsh limits on how much area they cbna have if you don;t want to make them needlessly fragile and remove heat via radiation alone and which have to work at the same temperature gradient. Given my point about how huge the surface area to volume ratio inside the laser can get and given the advantage of conduction in addition to radiation as a transfer method you'd need a very inefficient medium indeed for it to not easily provide quite good cooling per cubic meter relative to your available volume and surface area ratio's. All the limits in the world don't stop it still being almost certainly an incredible improvement over and above the alternative. The sole limitation right now is a lack of known medium for such a purpose.

Lvl 2 Expert
2015-06-27, 05:25 PM
Dude! SPOILERS! Some of us can't go to the cinema until it summer-opens on July 2.

Oops, sorry. Spoilerized it, could you do the same in your quote for what it's worth?

Charlaton
2015-06-27, 10:04 PM
Hey everyone, hopefully this hasn't been addressed in this thread, or at least recently. I was wondering if anyone has any sources on what it takes to defeat different armors of medieval.

For instance:
a) Would a thrust from a spear or longsword be able to defeat maille and its accompanying padded armor? What about against textile armors? If those could, would a thrust from an arming sword be able to generate sufficient force?
b) Would a strike to the back or front of a coif-protected neck have sufficient power to collapse the wind pipe or break the neck, or would it only incapacitate the individual struck?
c) Would a strike from a longsword be able to break the foot or ankle of someone protected by a plate sabaton? What about the maille equivalent (there was such a thing, right)?
d) Could a morte-strike penetrate a plate helm? Or similarly, if an opponent were laying on the ground, would a morte-strike defeat plate (penetration or create a sufficient dent to break ribs) assuming an overhead swing?

Those are the only ones that have struck me so far, and while I would appreciate answers to them, I'd also love to read any research concerning such things that anyone has come across.

Also, I believe I read someone stating earlier that plate likely wouldn't have become so widespread without the advent of firearms. What do others think of that? For instance, if in some fantasy world, civilization never adopted handheld or even field artillery (say magic or nobility putting sufficient pressure to prevent it from happening), would plate still be developed and used to anywhere near the extent it is?

Thank you!

goto124
2015-06-28, 12:02 AM
Plate might work well against magic missle...

fusilier
2015-06-28, 03:11 AM
Also, I believe I read someone stating earlier that plate likely wouldn't have become so widespread without the advent of firearms. What do others think of that? For instance, if in some fantasy world, civilization never adopted handheld or even field artillery (say magic or nobility putting sufficient pressure to prevent it from happening), would plate still be developed and used to anywhere near the extent it is?

Hmmm. The introduction of firearms and the development of plate armor do occur at the same time -- but that doesn't necessarily mean that one was a response to the other. While crossbows had been a around for a long time they were also becoming more common during the same period (and early firearms and "late" crossbows seem to have had similar capabilities). During the 15th century, horse armor became more common as a response to the increased number of missile weapons in use.

Certainly there was some interplay between the development of armor and gunpowder weapons, however, it wasn't until the 16th century that gunpowder weapons became the predominant missile weapon and plate armor peaked in the late 15th/early 16th century. I think the major point is that plate armor developed at a time when firearms were present (refuting the idea that the appearance of firearms immediately rendered armor obsolete). So I guess the question is - is plate armor a response to firearms? A response to an increased use of missile weapons generally? A response to other weapons (wikipedia mentions polearms)? Or, was it a natural development of armor? (Or, perhaps, some sort of combination of factors?)

SowZ
2015-06-28, 03:18 AM
Hmmm. The introduction of firearms and the development of plate armor do occur at the same time -- but that doesn't necessarily mean that one was a response to the other. While crossbows had been a around for a long time they were also becoming more common during the same period (and early firearms and "late" crossbows seem to have had similar capabilities). During the 15th century, horse armor became more common as a response to the increased number of missile weapons in use.

Certainly there was some interplay between the development of armor and gunpowder weapons, however, it wasn't until the 16th century that gunpowder weapons became the predominant missile weapon and plate armor peaked in the late 15th/early 16th century. I think the major point is that plate armor developed at a time when firearms were present (refuting the idea that the appearance of firearms immediately rendered armor obsolete). So I guess the question is - is plate armor a response to firearms? A response to an increased use of missile weapons generally? A response to other weapons (wikipedia mentions polearms)? Or, was it a natural development of armor? (Or, perhaps, some sort of combination of factors?)

Depending on several factors, range, the specific weapon/armor, the right circumstances. plate could effectively deflect a musketball.


Hey everyone, hopefully this hasn't been addressed in this thread, or at least recently. I was wondering if anyone has any sources on what it takes to defeat different armors of medieval.

For instance:
a) Would a thrust from a spear or longsword be able to defeat maille and its accompanying padded armor? What about against textile armors? If those could, would a thrust from an arming sword be able to generate sufficient force?
b) Would a strike to the back or front of a coif-protected neck have sufficient power to collapse the wind pipe or break the neck, or would it only incapacitate the individual struck?
c) Would a strike from a longsword be able to break the foot or ankle of someone protected by a plate sabaton? What about the maille equivalent (there was such a thing, right)?
d) Could a morte-strike penetrate a plate helm? Or similarly, if an opponent were laying on the ground, would a morte-strike defeat plate (penetration or create a sufficient dent to break ribs) assuming an overhead swing?

Those are the only ones that have struck me so far, and while I would appreciate answers to them, I'd also love to read any research concerning such things that anyone has come across.

Also, I believe I read someone stating earlier that plate likely wouldn't have become so widespread without the advent of firearms. What do others think of that? For instance, if in some fantasy world, civilization never adopted handheld or even field artillery (say magic or nobility putting sufficient pressure to prevent it from happening), would plate still be developed and used to anywhere near the extent it is?

Thank you!

A.) A really solid stab with a good sword? Maybe, it depends on the sword and the armor, really. Some swords were really good at it. A spear? Sure.
B.) What, with a sword? Absolutely not. If so I should be dead right now. I wouldn't even say incapacitate. It might hurt like hell but I could keep fighting.
C.) A solid strike from a long sword that didn't penetrate would not have enough impact to break the bones of someone wearing a skiing jacket.
D.) An opponent on the ground takes significantly more blunt force, since their body cannot transfer momentum by moving with the strike. So, maybe, but I still wouldn't bet on it. The technique was used to fight armor, so clearly it gave you a fighting chance, but I still wouldn't expect it to work every time.

To give some general info. Plate made you all but immune to slashes and cuts. The blunt force trauma of a sword is minimal. A thick gambeson or arming coat is much harder to cut through with a slash than you would think. Shoot, cutting through clothing is harder than you might think. If you swung at me and missed and you stopped the sword for an instant, rather than keeping it in motion, I'm liable to just grab the blade and damn the consequences. A shallow cut to my palm is unlikely to kill me.

Archpaladin Zousha
2015-06-28, 04:41 PM
I was interested in creating a character similar to the Cohors Sagittariorvm Levantinorvm (Imperial Eastern Archer Auxilia) unit from Rome: Total War's famous Europa Barborum mod:

http://europabarbarorum.com/EB1/i/units/romani/roman_archer_auxilia.gif

Specialised archer units for the imperial army can be enrolled in all provinces in which skilled bowyers and a population familiar with the traditions of archery can be found. These are mainly in Asia and the fringes of the steppes. Archers now have become an integral part of any well composed roman army. Normally they are placed behind a wall of protective heavy infantry in the battle formation. From there they can provide a supporting barrage fire above the infantry's heads to weaken the enemy and break his charge before the main lines engage. The imperial army's archers are more heavily armoured than their counterparts, with shirts of lorica squamata or hamata (scale or chain mail), conical iron or bronze helmets and small shields protecting their left arm. They use long ranged recurved composite bows with bone ends and bracers to protect their forearms from the sinew, together with multiple types of arrows: three bladed heads to inflict heavy wounds at un armoured targets, thin needle like, pyramidal shaped armour piercing heads and flaming arrows, carrying an ignition load in a kind of small metal cage incorporated into the arrowhead. Additionally the archers are armed with a gladius for self defence, but despite this and their armour it should not be expected that they fight well at close quarters for any longer time.
It's clear from the description that these are composite bows, but I'm not certain if they're longbows or shortbows as per D&D and Pathfinder gear conventions. Generally longbows seem to be associated with wood and more popular in Northern Europe, as indicated by the Celtic and Germanic factions longbow units. By contrast, shortbows were more associated with the horse archers of the east, like the Parthians and Huns.

So, do I go short or long for this foot archer?

Incanur
2015-06-28, 04:44 PM
Hey everyone, hopefully this hasn't been addressed in this thread, or at least recently. I was wondering if anyone has any sources on what it takes to defeat different armors of medieval.

I recommend Alan Williams's The Knight and the Blast Furnace and Michael Edelson's informal test (http://www.myarmoury.com/talk/viewtopic.php?t=11131).


For instance:
a) Would a thrust from a spear or longsword be able to defeat maille and its accompanying padded armor? What about against textile armors? If those could, would a thrust from an arming sword be able to generate sufficient force?

Like most weapon vs. armor weapons, this depends on the person behind the weapon, the details of the weapons construction, and the details of the armor's construction. Williams's test of both reconstruction and period mail plus a rather serious quilted jack against a simulated arrowhead concluded that 120 J was required to defeat this combination. Edelson's test of likely inferior reconstruction mail produced penetration by both a narrowly pointed longsword and a pollaxe point. I tend to think two-handed thrusts with narrowly pointed weapons by at least strong wielders could penetrate mail and the fabric worn underneath. I suspect it'd take considerably more strength with a single-handed thrust. And also note that some period mail plus padding was heavier than those tests and some of it may have been made of better metal as well. (There are reference to hardening mail. Hardened steel mail might require about 36% more energy to penetrate than the mail Williams tested. We have period references to weapons piercing mail and to mail stopping even lance thrusts, so it's hard to say for sure.


b) Would a strike to the back or front of a coif-protected neck have sufficient power to collapse the wind pipe or break the neck, or would it only incapacitate the individual struck?

I imagine this would depend on the rigidity of the mail and the thickness of the padding, as well as of course the power of the strike.

[/quote]c) Would a strike from a longsword be able to break the foot or ankle of someone protected by a plate sabaton? What about the maille equivalent (there was such a thing, right)?[/quote]

As always, depends on the wielder of the longsword. Foot armor was generally pretty thin, so it's possible a powerful strike could disable the foot. From what I've seen in manuals, though, thrusting at armored feet was more common that strike at them. And yes, there was a mail equivalent.


d) Could a morte-strike penetrate a plate helm?

Depends on the helm. I wouldn't bet on it against hardened steel. According to Williams's tests, penetrating 2mm hardened steel with an 18-degree point requires approximately 262.5 J. I assume a strong enough morte-strike could stun or even kayo the person inside the helm, though.


Or similarly, if an opponent were laying on the ground, would a morte-strike defeat plate (penetration or create a sufficient dent to break ribs) assuming an overhead swing?

Against a breastplate? If hardened steel and reasonably thick, I doubt it.


Also, I believe I read someone stating earlier that plate likely wouldn't have become so widespread without the advent of firearms. What do others think of that? For instance, if in some fantasy world, civilization never adopted handheld or even field artillery (say magic or nobility putting sufficient pressure to prevent it from happening), would plate still be developed and used to anywhere near the extent it is?

I think so. While historically plate armor in Europe did evolve alongside firearms, I don't think it developed *because* of firearms. It certainly declined in use in part because of firearms, though this took a while. And it didn't develop in China despite the firearms. A host of factors lead to plate armor in Europe, but I suspect it would appeared even if firearms hadn't, because it's such an excellent defense against muscle-powered weapons and because Europe had the technology and social structure for its development.

The best circa-1500 plate harness of hardened steel made its wearer nearly immune to penetrate from muscle-powered weapons including the couched heavy lance and heavy crossbows. Those two weapons might pierce thinner plates, but some 16th-century military writers still considered aiming a lance at a man-at-arms a waste of time because of how unlikely it was to penetrate. Beyond that techniques for defeating folks in full plate thrusts to the face and mail-covered gaps such as the armpit, blows to the head from polearms, blows to the hands/fingers, and grappling. Repeated blows to wherever from cavalry swords and maces/axes/hammers could also eventually incapacitate one in full plate.

snowblizz
2015-06-28, 05:54 PM
Oops, sorry. Spoilerized it, could you do the same in your quote for what it's worth?

Not your fault. It thought it was a really funny joke... then I suddenly realise what I've seen in trailers and that it's a joke referring to something from the movie.

Like I told my friend when he explained the cinema was closed when I asked about going. "Whaddaya mean they open with 3 premiers, they aren't premiers 2 weeks after they have been released elsewhere dammit."

I've been staying clear of a lot places because we get stuff later. It's become a lot better than it used to be, imagine 6-12 month delays, this apparently is bad timing. I've also tried to not see the trailers either because they are often as bad.

SowZ
2015-06-28, 06:38 PM
I recommend Alan Williams's The Knight and the Blast Furnace and Michael Edelson's informal test (http://www.myarmoury.com/talk/viewtopic.php?t=11131).



Like most weapon vs. armor weapons, this depends on the person behind the weapon, the details of the weapons construction, and the details of the armor's construction. Williams's test of both reconstruction and period mail plus a rather serious quilted jack against a simulated arrowhead concluded that 120 J was required to defeat this combination. Edelson's test of likely inferior reconstruction mail produced penetration by both a narrowly pointed longsword and a pollaxe point. I tend to think two-handed thrusts with narrowly pointed weapons by at least strong wielders could penetrate mail and the fabric worn underneath. I suspect it'd take considerably more strength with a single-handed thrust. And also note that some period mail plus padding was heavier than those tests and some of it may have been made of better metal as well. (There are reference to hardening mail. Hardened steel mail might require about 36% more energy to penetrate than the mail Williams tested. We have period references to weapons piercing mail and to mail stopping even lance thrusts, so it's hard to say for sure.



I imagine this would depend on the rigidity of the mail and the thickness of the padding, as well as of course the power of the strike.

c) Would a strike from a longsword be able to break the foot or ankle of someone protected by a plate sabaton? What about the maille equivalent (there was such a thing, right)?

As always, depends on the wielder of the longsword. Foot armor was generally pretty thin, so it's possible a powerful strike could disable the foot. From what I've seen in manuals, though, thrusting at armored feet was more common that strike at them. And yes, there was a mail equivalent.



Depends on the helm. I wouldn't bet on it against hardened steel. According to Williams's tests, penetrating 2mm hardened steel with an 18-degree point requires approximately 262.5 J. I assume a strong enough morte-strike could stun or even kayo the person inside the helm, though.



Against a breastplate? If hardened steel and reasonably thick, I doubt it.



I think so. While historically plate armor in Europe did evolve alongside firearms, I don't think it developed *because* of firearms. It certainly declined in use in part because of firearms, though this took a while. And it didn't develop in China despite the firearms. A host of factors lead to plate armor in Europe, but I suspect it would appeared even if firearms hadn't, because it's such an excellent defense against muscle-powered weapons and because Europe had the technology and social structure for its development.

The best circa-1500 plate harness of hardened steel made its wearer nearly immune to penetrate from muscle-powered weapons including the couched heavy lance and heavy crossbows. Those two weapons might pierce thinner plates, but some 16th-century military writers still considered aiming a lance at a man-at-arms a waste of time because of how unlikely it was to penetrate. Beyond that techniques for defeating folks in full plate thrusts to the face and mail-covered gaps such as the armpit, blows to the head from polearms, blows to the hands/fingers, and grappling. Repeated blows to wherever from cavalry swords and maces/axes/hammers could also eventually incapacitate one in full plate.

My only contention is with the assessment of point B, where I would say very confidently you would almost never break the neck with blunt force trauma, no matter how poor the mail. Modern longsword fighters use real weight, real length steel longswords, (the point they come to is of course blunted, but still narrow,) and for the most part we only wear relatively thin rigid plastic coated in closed cell foam as neck protection. Far less protection than even crappy mail with thin padding. And we have very strong people who are much taller than the average medieval knight.

Mr. Mask
2015-06-28, 06:55 PM
Carl: Once they worked out how much of a threat the dragon was, they may be willing to use nukes. even while it's in range of a highly populated area (so that a few million people may die from the nukes). If the dragon kept to less inhabited areas, it would be a much quicker and easier decision.

With projectiles, even a few knots of wind will mess up long range fire. If an ICBM has to go through eighteen miles of hurricane, tornadoes, and hail, purposefully attempting to redirect it, it is not likely to be close to its target. If the dragon is capable of detecting and purposefully redirecting ICBMs, then sending them in groups isn't likely to help.

The yellow stone trick sounds like a plausible way to kill the dragon. I would be surprised if they gave the order to do that, but if the dragon pressed them too hard so that extinction seemed a possibility, they might. In this case, the dragon isn't likely to be lured, unless they were willing and able to use a major city as bait.



Oni: Well, that is moving the goal posts a bit. I never argued the dragon must know about technology, but that it is incorrect in assuming the dragon could not possibly know, in the context of it being a deific creature. I implied the dragon may know how to make use of nukes, to the extent that the possibility might confuse others as to whether that is possible. Since knowledge is hierarchical, why do you assume that technological knowledge is unnecessary in understanding how to make the winds obey your command? Human technology is applying matter and physics to get a result, and a creature that understands the metaphysical enough to act upon the physical seems liable to understand the physical to an extreme degree. If the dragon is a dumb beast that acts on instinct, it makes sense it may not understand anything outside of its instincts. But if it is a god, then I don't think you can presume on it's ignorance simply because it's from the past. "Oh, they're using radar waves as a form of active detection. How quaint."

In the example of a person with divine inspiration of wordplay. If the person's ability is restricted specifically to wordplay and riddles, then anything that is not wordplay or a riddle will naturally be excluded from their divine ability. But, if the complex algorithms were counted as riddles, a logic puzzle, then I would say there is good evidence that the person's divine abilities would allow them to answer it. If they can answer any riddle, then it means they can answer riddles beyond their experience and knowledge, unless they have experienced everything and know everything. And if they did experience and know everything, then the very existence of such an algorithm means they can answer it. If you put more limits on the person's divine inspiration, like they can't answer a riddle spoken in another language, or about a matter they aren't familiar enough with, then it becomes likely that they can't answer advanced cryptography. It also becomes more likely that their abilities aren't divine.


While I can understand physical objects getting picked up in the hurricane winds, causing radar fuzzing unintentionally, radar chaff is specifically masses of thin metallic strips and in a hurricane, they're likely to be scattered about very quickly, reducing or possibly negating their radar defeating effect.

It also begs the question, how does the dragon know about radar in the first place to be able to develop ECM? Well, the dragon is likely to have a pretty good perception and control of its winds, to be able to perform the other abilities with them. So I figure it could keep chaff and other objects within its hurricane if it wished to.

I expect that when they start hitting the dragon with radar waves, it'll see what they're doing. That is, that it can literally sense radar and other non visible electromagnetic spectra (at it's level of power, deities using invisible deathrays seems a logical concern).

Brother Oni
2015-06-29, 03:40 AM
Oni: Well, that is moving the goal posts a bit.

I think we have different definitions of the word 'god' and as such we're just arguing around in circles.

You're apparently making it both omnipotent and omniscient as a deific being - I've read enough of various mythology to not assign either of those attributes to deities.
If you want to assume those two characteristics then go ahead, but I have nothing further to contribute to this topic.


Since knowledge is hierarchical, why do you assume that technological knowledge is unnecessary in understanding how to make the winds obey your command?

I suspect it's because of my perspective of knowing that science and technological knowledge is made up of thousands of specialisations, all of which combine to a whole. If you want to handwave 'technological knowledge' as a single unified subject in the best Hollywood fashion then again go ahead, but I'll withdraw from the topic.



Well, the dragon is likely to have a pretty good perception and control of its winds, to be able to perform the other abilities with them. So I figure it could keep chaff and other objects within its hurricane if it wished to.

I expect that when they start hitting the dragon with radar waves, it'll see what they're doing. That is, that it can literally sense radar and other non visible electromagnetic spectra (at it's level of power, deities using invisible deathrays seems a logical concern).

Last bit of calculation for you as an indication to indicate how absurd you're making your dragon by giving it natural chaff making abilities:

Chaff has a minimum area/volume for it to be effective and the military ECM version would be the lowest ratio required, since the strips are perfectly designed to defeat radar. Using other physical objects would only increase the required density as they are not as reflective, nor of the right length to prevent back scattering.

You would also need a minimum volume of the covered area to prevent AOE effects blasting the dragon out of the sky - suppose we have a 50kton warhead and assuming that only a 20PSI overpressure or greater is required to injure your dragon, the missile would need to detonate over ~800m away.

This means it would need to generate a sphere covering 2.14km3 and based on this reference (https://books.google.co.uk/books?id=I1vksFcnQngC&pg=PA329&lpg=PA329&dq=chaff+density+ECM&source=bl&ots=EjmUI5qVBl&sig=gFl7TihV5_Vw0w5Lc-6ldz_EulM&hl=en&sa=X&ei=Jv2QVYj4O8qc7AbDm4oQ&ved=0CD8Q6AEwBQ#v=onepage&q=chaff%20density%20ECM&f=false) of 0.0024 m2/m3 of chaff required, that means you need 5.1 million m2 of chaff to provide complete coverage.

Assuming you have 5x2cm aluminium strips (if the dragon knows the military are using 10cm radar and not 5 or 3cm), I make that ~514 million exactly 5cm long strips in the air.

Incidentally, your dragon would be able to sense radar and the like, in much the same way as we sense air pressure on our bodies, since it's constantly bathed in the waves thus has to filter out the constant background stimulation to sense anything new.

Galloglaich
2015-06-29, 09:16 AM
1: Why don't you prove it. You can't - Nobody ever had significant trouble dealing with the Tiger. Not the British, not the Americans, not the Russians. It was a piece of garbage that hastened the fall of Germany by wasting resources. Every single account of Tigers inflicting serious harm has been disproven - either by Allied records showing there not having been a battle on that day at all, or by German records showing that there wasn't a Tiger within a thousand miles of where Allied troops claim to have fought one. Sure, it had a monstrous gun and could do some real damage if it managed an ambush, but it was never more useful than a towed 75, let alone a vehicle like the StuG that was actually good.

As for the T-34, in the early stages of the war successes were few, and the veteran German troops were excellent at simply falling back a little and leading the Soviets into a trap.

That is such a ridiculous outlier statement it almost beggars imagination. You might as well say nobody ever had any significant trouble dealing with tanks, period (you could in fact make this argument, but it would be disingenuous)

I assume that Gnomon is reacting to a certain unfortunate trend in WW II gaming circles in which the German kit of all kinds was exaggerated to a phenomenal degree and particular systems like the Tiger and the FW 190, Me 262 and so on were glorified above all others. For decades in the late 20th Century and early 21st, a lot of gamers were so enamored of German technology and alleged super-skills in WW II, so uncritically accepting of WW II era Nazi propaganda about the Heer and especially the Waffen SS, that you had a lot of people who could make you feel uncomfortable in the gamer world, including in the (otherwise wonderful) early desktop gamer world, and model builders and etc. and etc. - people whose politics was drifting far to the right in their glassy eyed love for all things with a bent cross painted on them.

Gnoman appears to be part of a backlash against some of this rather creepy fanboy craze, either individually or (I suspect) part of a little counter-culture of debunkers.

But I think he's taking it way, way, way too far. There is absolutely no doubt that the Tiger I in particular was a very dangerous weapon and on a tactical level at least, posed a major problem for allied war efforts. You can make an argument that it was a strategic blunder because it probably took more money, time and manpower to make and operate 10 Tiger tanks than it did to make and operate 50 Pz IV. But that was the bent of the entire German war machine at that time, a certain emphasis on quality, and there is no doubt they were a very difficult opponent to defeat and their defeat was only effected at enormous cost (mostly in Russian lives).


Kasserine pass? I don't know many Tigers were there, I wouldn't be surprised if it was no more than a single company, probably understrength. Could be as few as 10 or 15 Tigers. But you would be foolish to assume that meant they had little impact. We know in fact that they did. Almost all battles in North Africa had ridiculously small numbers of tanks which nevertheless had a major effect on the outcome. Tanks are dangerous! Tigers had a huge impact due to their near invulnerability at long range and their ability to kill at a very long range.


https://upload.wikimedia.org/wikipedia/commons/a/a6/Ju_87B_NAN1Sep43.jpg http://wiki.gcdn.co/images/thumb/2/21/Pz_III_Crew_out.jpg/314px-Pz_III_Crew_out.jpg
The Stuka was actually the critical component of many early German armored victories

Prior to 1942, including during the famous 'blitzkrieg' operations in 1940-41, German armored vehicles tended to have small guns and relatively light armor. German (and captured Czech) tanks had large well laid out, three man turrets, good optics and aiming systems, good radios, and well trained crews, as well as good ammunition and machine guns. But they were actually outgunned by and had less armor than many of the tanks their French and English opponents. They relied on well coordinated precision strikes by Stuka dive-bombers (especially), anti-tank guns, and artillery to punch through enemy lines or break up enemy attacks. Obviously this worked quite well.. for a while. Anti-tank guns in particular were devastating on defense but they instantly became highly vulnerable to mortars etc. once their position was known. The heaviest tanks could be knocked out by heavy anti-aircraft guns (88) pressed into antitank service.

https://upload.wikimedia.org/wikipedia/commons/2/28/Bundesarchiv_Bild_101I-297-1701-20%2C_Im_Westen%2C_Panzerj%C3%A4ger_%22Marder_I%22 .jpg
Marder

After increasing difficulties in Russia (including sobering encounters with the early T-34 and KV tanks) the Germans re-evaluated this strategy. The Stuka, though still useful with it's incredibly accurate pinpoint strike capability, was no longer able to always ensure breakthroughs or smash enemy tank columns, for one thing it was becoming too vulnerable as Allied fighters got better. The Russian front was the most important and Soviet VVS got an important boost from lend-lease fighters from the West, they made particularly good use of P-40 and P-39 fighters, but their own Yak 1 and La 5 series were coming online, which would become some of the best aircraft of the war. The Germans could no longer guarantee local air superiority and the Stuka was an old design by this point and couldn't handle these more potent fighters. So the Germans needed more firepower on the ground. From around mid 1942 the basic German tank or assault -un followed the design principle of a very nice, accurate and powerful gun which could pick off enemy vehicles from far away, and then presumably move off. This evolved from early improvisations on the Russian front, weapons like the Marder for example (a field expedient mashup of a - for the time- large anti-tank gun with an obsolete tracked vehicle) which proved extremely useful in preventing catastrophes from the increasingly aggressive Soviet counterattacks. The Marder was much better than an anti-tank gun because it could move - so it could sit in ambush, kill, and go away. But it as an 'open' vehicle it was very vulnerable to artillery and mortars and so on, and also it's armor was very thin and any tank could kill it, it's only real defense was that the range from it's (for the time) very big gun was such that it could often get away before enemy tanks got in range.

http://hosted.wargamer.com/panzer/stug05.jpg http://www.panzergeneral.de/images/stug3g.jpg
Early StuG / mid-war StuG

The StuG III which Gnomon likes so much was changed at this time from an assault gun with a low-pressure 75 howitzer to a tank-killer with a long range anti-tank gun mounted on it, designed for picking off T-34's from charging columns from vast distances across the steppe (miles away). This worked great at that kind of distance, (as was often though by no means always the case in Russia) but all the turretless AFV's proved very vulnerable to turreted tanks when the latter got close and in sticky territory. The StuG however had the advantages of the Marder plus good armor, and it also had useful HE ammo making the assault gun (anti-infantry, anti-anti-tank gun) features useful for when enemy tanks weren't around, so it remained a useful weapon.

But the Soviet tanks kept getting better and they had assault guns of their own. So the Germans needed to up the ante. They followed the strategy of bigger, longer ranged, accurate guns. The Soviets put more emphasis on speed, armor, mechanical reliability and most of all numbers, so it was a contest of snipers vs. knife fighters you might say. Same for infantry where the Germans were still mostly using Mauser K 98 bolt action rifles, very accurate out to 400 or 500 meters and hard hitting (but slow firing), augmented by the superb German light/medium/heavy machine gun of the MG 34/42 series; while the Soviets had switched to Burp guns which could massacre enemies at 100 meters but had pistol caliber ammo and couldn't hit anything at 300 meters (but still had pretty lousy light and medium machine guns through the whole war). In some battles German MG 42s and rifles picked off enemies from range before they could get close, in others Soviet SMG's utterly massacred helpless rifle bearing troops from point blank range (prompting the Germans to develop their own SMG's). The Germans were killing more and more Russians but the Russians kept coming and the Germans started suffering some major defeats and losing a lot of casualties. Most notably at Stalingrad of course which is where, incidentally, the Germans first experienced a really heavy shock from several 'new' Russian weapons systems including the T-34 and KV tanks (used in large numbers for the first time) and the Katushka rocket launcher, all deployed en-masse, on a scale the Germans had never before seen, to pinch off the 6th Army. Up to that point they had never seen more than a few score T-34's and KV's in one place, at Stalngrad in one day they dealt with hundreds of T-34's in waves, preceded by WW I scale bombardments of artillery and rockets. In the battles which isolated the 6th Army at Stalingrad German (and Italian, Bulgarian, Romanian etc.) troops did panic and did find themselves unable to kill T-34's and KV's with the weapons they had.

https://upload.wikimedia.org/wikipedia/commons/c/ce/Bundesarchiv_Bild_101I-313-1004-25,_Italien,_Panzer_%22Elefant%22.jpg

Most German tanks, tank destroyers and assault guns from this point followed the 'sniper' design method, some with heavy armor added, some retaining the simpler Marder strategy of almost no armor but a really dangerous gun (this making them more maneuverable). The Panther was designed for long range engagements, with a long barreled, very accurate and very high velocity gun, good maneuverability, and good frontal armor, but relatively weak side armor. This worked but had it's limitations, (vulnerability to side shots for example, and to rockets, aircraft and so on, and to being flanked by fast moving tanks) and the Soviets fine tuned their weapons and their strategies to exploit these limitations.

Where the Tiger I stands out is that it had good armor all around, not invulnerable but good enough that a T-34, or a Sherman, had a hard time killing it unless they got very close. It also had a lot of features making it hard for infantry to overrun it.

The reason a Tiger is so scary in a Kasserine pass situation is that it couldn't be killed from anywhere near the distance where it could kill you. None of the allied tanks or guns could kill it from anything like range, 6 pounders are good weapons but it's a small caliber gun, but even at (almost point blank) range of a hundred meters it will not cut through the front armor of a Tiger I, at 500 yards it can't even get through the side armor unless it gets a very lucky shot. Most of the American anti-tank guns at that time weren't even that powerful, but were only 37mm. The M3 Lee tanks they had didn't have guns that could kill Tigers either unless very lucky or very close. By contrast, the Tiger could kill an M3 or an anti-tank gun position easily from 2000 meters.

http://www.wwiiequipment.com/index.php?option=com_content&view=article&id=74:6-pounder-anti-tank-gun&catid=40:anti-tank&Itemid=58

https://en.wikipedia.org/wiki/8.8_cm_KwK_36

The Tiger did prove decisive in a number of major battles on the Russian front, they were used as a sort of 'Fire Brigade' there and rushed in to shore up collapsing armies. To suggest that it was actually not as good as a Pz IV or a StuG III is just absurdity. Tigers were not used much in North Africa (other than at Kasserine) and were misused in Sicily. Allied armies didn't learn to fear it until after the D-Day landings (and this is why Sherman tanks were unprepared for the Panther and Tiger and their evil friends in late 1944, leading to a lot of the scandal and media hype which still reverberates today - because the Allied commanders hadn't really grasped that this level of threat existed). The Tiger wasn't quite the uber weapon it's made out to be by some SS loving fanboys, but it was a dangerous critter and definitely one of the most effective armored vehicles of it's day.


But of course you can play with the numbers to make yourself believe anything. My recommendation for anyone who wants to have a sense of it is try out a realistic simulation game like Squad leader or Combat Mission: Barbarossa (or one of the newer ones) you'll get an idea of why they were so scary. Combat mission even simulates the effect of Allied (especially US) troops thinking almost any tank they ran into was a Tiger.

Galloglaich
2015-06-29, 09:28 AM
Depending on several factors, range, the specific weapon/armor, the right circumstances. plate could effectively deflect a musketball.



A.) A really solid stab with a good sword? Maybe, it depends on the sword and the armor, really. Some swords were really good at it. A spear? Sure.
B.) What, with a sword? Absolutely not. If so I should be dead right now. I wouldn't even say incapacitate. It might hurt like hell but I could keep fighting.
C.) A solid strike from a long sword that didn't penetrate would not have enough impact to break the bones of someone wearing a skiing jacket.
D.) An opponent on the ground takes significantly more blunt force, since their body cannot transfer momentum by moving with the strike. So, maybe, but I still wouldn't bet on it. The technique was used to fight armor, so clearly it gave you a fighting chance, but I still wouldn't expect it to work every time.

To give some general info. Plate made you all but immune to slashes and cuts. The blunt force trauma of a sword is minimal. A thick gambeson or arming coat is much harder to cut through with a slash than you would think. Shoot, cutting through clothing is harder than you might think. If you swung at me and missed and you stopped the sword for an instant, rather than keeping it in motion, I'm liable to just grab the blade and damn the consequences. A shallow cut to my palm is unlikely to kill me.

I do HEMA too, and have done for many, many moons, and while I generally agree with you here, (and I've been trying to explain to these folks that blades make very poor hammers for years now in this thread), you can also go too far with that; people do get broken bones from feders. I've seen people get broken ribs, hands, forearms, collar bones and I think even a foot from a feder before at tournaments. Through mail though I don't know - mail does have some protection against blunt trauma. As I'm sure you know we just use textle, foam and / or thin plastic protection (and fencing masks) and it seems to work very well to protect against their blunt impact. Injuries tend to happen when people don't have enough protective kit.

For that matter I'm not sure a spear or a sword could thrust though mail necessarily, from tests I've seen I have doubts, but it's hard to be sure because making the realistic type mail is so expensive and not enough testing has been done - most is done with horrible butted mail or really cheap, poorly made (but at least riveted) re-enactor mail made in India.

G

Brother Oni
2015-06-29, 10:13 AM
Galloglaich's reminded me of that source regarding the German target prioritisation of Sherman Fireflies over the regular M4.

It's complicated and very murky, to put it bluntly.

While the difference between a Firefly and a regular M4 is hard to see at range (the difference in cannon length and the radio bulge at the back of the turret), the British did start painting the outer underside of their Fireflies' cannons to help disguise the length, although whether this actually helped is unrecorded (one source suggested it was to reassure the Firefly crews into feeling safer and soldiers with higher morale, fight better).

That said, a Firefly looks vastly different to a Cromwell and any sane tank commander would prioritise the tank with the bigger cannon.

Adding to the issue, is that with the 17lber, the Firefly didn't need to get as close to knock out a Tiger as a regular Sherman did, thus there may have been proportionately more casualties in bocage country in Normandy (despite the bigger cannon, the armour of a Firefly was still comparatively weak) due to the shorter engagement distances, plus as the Fireflies became more numerous, there may have been a casualty increase for that reason, rather than the Germans specifically targeting them.

As a side note (since I enjoy little stories like this), I found one regarding the B squad of the Queen's Royal Irish Hussars losing three Cromwells to a single Tiger stood out in the open (they had no way of flanking it, thus did no damage to it). Despite this, the Royal Irish managed to fight off the German infantry and the Tiger withdrew (standard tactics for a Tiger was to provide supporting fire for their infantry attacks from outside of the enemy's tanks' range).
A couple of days later, the same situation happened to C squad and rather than go through the same nightmare again, they asked a Firefly from the North Somerset Yeomanry for help, which just drove up and took out the Tiger before the Tiger even saw it.

SowZ
2015-06-29, 11:47 AM
I do HEMA too, and have done for many, many moons, and while I generally agree with you here, (and I've been trying to explain to these folks that blades make very poor hammers for years now in this thread), you can also go too far with that; people do get broken bones from feders. I've seen people get broken ribs, hands, forearms, collar bones and I think even a foot from a feder before at tournaments. Through mail though I don't know - mail does have some protection against blunt trauma. As I'm sure you know we just use textle, foam and / or thin plastic protection (and fencing masks) and it seems to work very well to protect against their blunt impact. Injuries tend to happen when people don't have enough protective kit.

For that matter I'm not sure a spear or a sword could thrust though mail necessarily, from tests I've seen I have doubts, but it's hard to be sure because making the realistic type mail is so expensive and not enough testing has been done - most is done with horrible butted mail or really cheap, poorly made (but at least riveted) re-enactor mail made in India.

G

Sure, my instructor has broken ribs. People break hands and fingers all the time. But a lot of the worst injuries are, as you said, bad gear. Either a jacket with really thin forearm padding, crappy lacrosse gloves, an accidental cross guard punch, etc. My point is that if breaking bones is rare with what amounts to just a gambeson, how is it ever going to happen through both a gambeson and mail? The physics of a sword mean that no matter how solidly you hit, you only have a couple ounces behind your swing. But you are right that my answer to question C is an exaggeration. I don't think you could break a thick bone like a forearm or femur even with a winter coat if the sword was blunted.

I would guess a spear could thrust through mail in the right circumstances. The guy is moving towards you, you get him solidly in center mass so it can't glance off very easily, you really thrust in hard with both hands. I'd guess the strength of the wielder will matter less than the angle. But this is just a guess. Still, if even a spear couldn't thrust through mail, why did so many knights and soldiers still bother with shields? Blunt weapons? I guess that's possible. Still, these guys aren't using blunt weapons.
https://s-media-cache-ak0.pinimg.com/736x/de/12/98/de1298a4b99ac1ae83b9be86a562a373.jpg

Also, if it can't really be done, what is the point of an estoc? It seems that estocs could even pierce plate by targeting weak/thin spots in the armor.
http://2.bp.blogspot.com/_IYGc_MWwkfw/ST83iLCEu7I/AAAAAAAAAhE/eF981drISgc/s400/640_late_medieval_battle.jpg

This guy has a sword hundreds of years earlier than estocs but still looks like it comes to a pretty good point. It seems as though he is attempting to stab a mail covered man in the back. http://www.angelfire.com/mb2/battle_hastings_1066/bayeux_tapestry/c1BT45.jpg

There's quite a few drawings of armored guys killed by spears, even guys in plate, but it is very possible the strategy was, "knock him on his back, then stab him," so I wouldn't know about that.

Of course, period drawings weren't necessarily wholly accurate, and the average people of the time surely had misconceptions about war just as the average person now doesn't know much about modern warfare. But if you see things in multiple places it is suggestive.

Martin Greywolf
2015-06-29, 12:15 PM
Swordsmen of the British Empire by D. A. Kinsley. Go buy it and read it, it pretty much tells you what the answer is, and more.

Thing is, physics is a complex beast, and warfare even more so. Add art and human memory to the mix, and interesting stuff happens. The question of what exactly a given weapon could do against given armor doesn't have a simple answer. Even the oft-neglected gambeson can have from 15 to 40 layers of linen, wool or silk, giving you 75 different scenarios just from that.

The period accounts, even firsthand written ones, clearly show what could happen. British regulation sabres could thrust through the mail and gambeson, albeit not very often and mostly from horseback. When cutting, they did sometimes sever arms in mail, but I have yet to see any account of that happening while dismounted. The same sabres proved to be mostly unable to cut (or maybe chop, period accounts use cut for both, and only sometimes specify draw-cut) through turbans, but able to knock people wearing them out. Spadroons couldn't thrust through russians wearing thick coats, so they were used as bludgeons.

Then there's archeological evidence that shows skulls and collarbones fractured even under mail/helmet. Hard to tell how exactly it happened in most cases, but there you go, it did.

In the end, making statements like "sword can't penetrate plate" is not a good thing to do. Plate makes you almost immune to cuts and chops and very resistant to thrusts, but it doesn't offer full protection against either. You may argue it's because metal impurities or other armor defects, but remember what the H in HEMA stands for? When people had crap metal, they learned how to work around that.

One last thing. Injuries seen in HEMA and re-enactment are a poor indicator of what sword may or may not do. After all, most people who do them aren't sociopaths, and will soften their blow when aiming for, say, neck after opponent slipped. Then and again, modern equipment is a lot safer in general (modern tournaments didn't have any fatality as far as I know, unlike historical ones). Just too many variables to be able to really tell.

SowZ
2015-06-29, 12:30 PM
Swordsmen of the British Empire by D. A. Kinsley. Go buy it and read it, it pretty much tells you what the answer is, and more.

Thing is, physics is a complex beast, and warfare even more so. Add art and human memory to the mix, and interesting stuff happens. The question of what exactly a given weapon could do against given armor doesn't have a simple answer. Even the oft-neglected gambeson can have from 15 to 40 layers of linen, wool or silk, giving you 75 different scenarios just from that.

The period accounts, even firsthand written ones, clearly show what could happen. British regulation sabres could thrust through the mail and gambeson, albeit not very often and mostly from horseback. When cutting, they did sometimes sever arms in mail, but I have yet to see any account of that happening while dismounted. The same sabres proved to be mostly unable to cut (or maybe chop, period accounts use cut for both, and only sometimes specify draw-cut) through turbans, but able to knock people wearing them out. Spadroons couldn't thrust through russians wearing thick coats, so they were used as bludgeons.

Then there's archeological evidence that shows skulls and collarbones fractured even under mail/helmet. Hard to tell how exactly it happened in most cases, but there you go, it did.

In the end, making statements like "sword can't penetrate plate" is not a good thing to do. Plate makes you almost immune to cuts and chops and very resistant to thrusts, but it doesn't offer full protection against either. You may argue it's because metal impurities or other armor defects, but remember what the H in HEMA stands for? When people had crap metal, they learned how to work around that.

One last thing. Injuries seen in HEMA and re-enactment are a poor indicator of what sword may or may not do. After all, most people who do them aren't sociopaths, and will soften their blow when aiming for, say, neck after opponent slipped. Then and again, modern equipment is a lot safer in general (modern tournaments didn't have any fatality as far as I know, unlike historical ones). Just too many variables to be able to really tell.

Sure, but HEMA is a good example of how swords had very little blunt force trauma.

Lvl 2 Expert
2015-06-29, 04:25 PM
There's quite a few drawings of armored guys killed by spears, even guys in plate, but it is very possible the strategy was, "knock him on his back, then stab him," so I wouldn't know about that.

Well, spears and pikes and similar weapons were used a lot in the golden age of heavy armor, that's for sure. I guess the standard tactical idea behind it would be more along the lines of the spearmen bracing again cavalry charges and the guys with the warhammers/goedendagen (I can't help being a bit of a provincialist (https://en.wikipedia.org/wiki/Goedendag) sometimes)/some other blunt or hooked weapon clubbing or dragging the rider of the horse (or just flooring the horse itself) and finishing the job with a big smack. But since fighting in melee never ends up that organized I guess the spearmen must have found ways to pierce or circumvent armor as well, at least for finishing the fallen riders off. Maybe they just opened their visors? After all, a few centuries later pike and shot formations without hammers had to deal with even heavier armor on the cavalry, or at least the few who got that close without getting shot or impaled...

Carl
2015-06-29, 05:35 PM
Regarding the discussion about sword blunt trauma . Little somthing you you to do to demonstrate an important difference between re-enactors and the real thing. Get a towel or tea towel and a pen with a retractable nib. Fold the towel till it's 4 layers thick, i'd go with 6-8 layers of a tea towel), and place your hand under it. Not gently stab your palm through the towel with the pen with the nib out and with the nib in. There's a noticeable difference in experienced pressure and it is experienced shock pressure that will determine the fractures and break and the like.

Obviously this only works when both scenarios involve non-rigid materials. So not useful for plate, (and in the same vein if reenactors use plastic of sufficient stiffness in the neck guards it doubtless makes neck blows far less dangerous).

SowZ
2015-06-29, 06:18 PM
Regarding the discussion about sword blunt trauma . Little somthing you you to do to demonstrate an important difference between re-enactors and the real thing. Get a towel or tea towel and a pen with a retractable nib. Fold the towel till it's 4 layers thick, i'd go with 6-8 layers of a tea towel), and place your hand under it. Not gently stab your palm through the towel with the pen with the nib out and with the nib in. There's a noticeable difference in experienced pressure and it is experienced shock pressure that will determine the fractures and break and the like.

Obviously this only works when both scenarios involve non-rigid materials. So not useful for plate, (and in the same vein if reenactors use plastic of sufficient stiffness in the neck guards it doubtless makes neck blows far less dangerous).

I'd say using a real weight steel sword and swinging it full tilt and someone wearing a gambeson is a better bet. Also, stabbing is a whole other animal. If you took a kitchen knife or butcher knife, and put padding thick enough that it couldn't cut through, and you swung it at yourself, it wouldn't hurt much, (assuming your hand isn't on a solid surface.) Of course, don't do this. This would be incredibly stupid to actually try.

The neck guards are basically some plastic wrapped in closed cell foam. Certainly nowhere near metal plate.

Incanur
2015-06-29, 07:29 PM
The question of weapons versus armor definitely does depend on the period and texts in question. The 15th-century El Victorial, which describes circa-1400 events, suggests that even singled-handed swords could cleave helms and generally have some effect against armor, at least when wielded by an especially strong knight like the protagonist Pero Niño. And period art depicts tons of longswords used in apparent overhead swings against helmeted foes.

Carl
2015-06-29, 08:08 PM
I'd say using a real weight steel sword and swinging it full tilt and someone wearing a gambeson is a better bet. Also, stabbing is a whole other animal. If you took a kitchen knife or butcher knife, and put padding thick enough that it couldn't cut through, and you swung it at yourself, it wouldn't hurt much, (assuming your hand isn't on a solid surface.) Of course, don't do this. This would be incredibly stupid to actually try.

The neck guards are basically some plastic wrapped in closed cell foam. Certainly nowhere near metal plate.

You managed to miss every point i made in epic fashion.


Let me reiterate.

1. A blunt bladed sword will not transfer energy through flexible material in the same way as a sharp one. The pen test just demonstrates this in a simple easy to do experiment.

2. Non-flexible materials partially or wholly negate this effect, (and then some more besides), thus something doesn't have to match plate, merely be more rigid than mail and it will affect how effective a blow is at doing damage to something underneath.

SowZ
2015-06-30, 01:04 AM
You managed to miss every point i made in epic fashion.


Let me reiterate.

1. A blunt bladed sword will not transfer energy through flexible material in the same way as a sharp one. The pen test just demonstrates this in a simple easy to do experiment.

2. Non-flexible materials partially or wholly negate this effect, (and then some more besides), thus something doesn't have to match plate, merely be more rigid than mail and it will affect how effective a blow is at doing damage to something underneath.

A feder doesn't come to a sharp edge, but it certainly comes to an edge of sorts. The flat on the edge is very thin. Yes, it will have less focused force than a proper sword, but only just. Either way, the physics are clear. You only have a couple ounces behind the part on the blade that hits the enemy. If you don't cut or slice, you are unlikely to do much damage at all. You don't have enough weight or momentum to cause significant blunt trauma.

Also, the padding worn is thin and in most places entirely non-rigid. Some people actually wear gear modeled after gambesons. If feder vs. gambeson=light bruising, sharp longsword vs. gambeson+maille is not going to be very likely to inflict any long term damage.

Mr. Mask
2015-06-30, 05:08 AM
G: Excellent post, G. I have nothing to add.



I think we have different definitions of the word 'god' and as such we're just arguing around in circles.

You're apparently making it both omnipotent and omniscient as a deific being - I've read enough of various mythology to not assign either of those attributes to deities.
If you want to assume those two characteristics then go ahead, but I have nothing further to contribute to this topic. This seems to be a questionable understanding of omnipotence and omniscience. The traits I've described of the dragon do not require either of these aspects, which makes me wonder why you're questioning my understanding of mythology and asserting otherwise. If I have said something that denotes such features, tell me what it was I said.


I suspect it's because of my perspective of knowing that science and technological knowledge is made up of thousands of specialisations, all of which combine to a whole. If you want to handwave 'technological knowledge' as a single unified subject in the best Hollywood fashion then again go ahead, but I'll withdraw from the topic. I find this belittling of my opinion unreasonable.


Chaff has a minimum area/volume for it to be effective and the military ECM version would be the lowest ratio required, since the strips are perfectly designed to defeat radar. Tinfoil. Chaff is slices of tinfoil. It is true that the exact dimensions of it can have an exceptional improvement on its effects. There have been some studies about it, as to what size is best to keep it behind aeroplanes the longest, and working the best with the aeroplane, and as you mention in relation to what radar is being used.


Using other physical objects would only increase the required density as they are not as reflective, nor of the right length to prevent back scattering. Of the problems the dragon faced, I didn't think acquiring or producing a tinfoil equivalent would be one of them.


You would also need a minimum volume of the covered area to prevent AOE effects blasting the dragon out of the sky - suppose we have a 50kton warhead and assuming that only a 20PSI overpressure or greater is required to injure your dragon, the missile would need to detonate over ~800m away.

This means it would need to generate a sphere covering 2.14km3 and based on this reference (https://books.google.co.uk/books?id=I1vksFcnQngC&pg=PA329&lpg=PA329&dq=chaff+density+ECM&source=bl&ots=EjmUI5qVBl&sig=gFl7TihV5_Vw0w5Lc-6ldz_EulM&hl=en&sa=X&ei=Jv2QVYj4O8qc7AbDm4oQ&ved=0CD8Q6AEwBQ#v=onepage&q=chaff%20density%20ECM&f=false) of 0.0024 m2/m3 of chaff required, that means you need 5.1 million m2 of chaff to provide complete coverage. Well, that's certainly a ludicrous amount of chaff. I'm not sure what to equate that amount to (a small steel building carved into thin shreds?).


Assuming you have 5x2cm aluminium strips (if the dragon knows the military are using 10cm radar and not 5 or 3cm), I make that ~514 million exactly 5cm long strips in the air.


Incidentally, your dragon would be able to sense radar and the like, in much the same way as we sense air pressure on our bodies, since it's constantly bathed in the waves thus has to filter out the constant background stimulation to sense anything new. Mm, the dragon would need a pretty sharp awareness.



An interesting thing I found while looking this up, is that military radar does not work well with storms. Storms have lots of reflective rain, which is great for weather radar, but lousy for getting any kind of lock on a target. It seems chaff or steel shrapnel would be unnecessary.

Spiryt
2015-06-30, 09:38 AM
The question of weapons versus armor definitely does depend on the period and texts in question. The 15th-century El Victorial, which describes circa-1400 events, suggests that even singled-handed swords could cleave helms and generally have some effect against armor, at least when wielded by an especially strong knight like the protagonist Pero Niño. And period art depicts tons of longswords used in apparent overhead swings against helmeted foes.


That's the problem with 'Protagonists' though.

They tend to do awesome things by definition.


Some Polish knight lifting the bell that couldn't be raised by 40 men, for example.

Even assuming that those 40 men would be interfering each other, even 4 men bell sounds fishy.




Well, spears and pikes and similar weapons were used a lot in the golden age of heavy armor, that's for sure. I guess the standard tactical idea behind it would be more along the lines of the spearmen bracing again cavalry charges and the guys with the warhammers/goedendagen (I can't help being a bit of a provincialist (https://en.wikipedia.org/wiki/Goedendag) sometimes)/some other blunt or hooked weapon clubbing or dragging the rider of the horse (or just flooring the horse itself) and finishing the job with a big smack. But since fighting in melee never ends up that organized I guess the spearmen must have found ways to pierce or circumvent armor as well, at least for finishing the fallen riders off. Maybe they just opened their visors? After all, a few centuries later pike and shot formations without hammers had to deal with even heavier armor on the cavalry, or at least the few who got that close without getting shot or impaled...

All this assumes that all this 'finishing off', 'piercing' etc. like it usually is portrayed in media, which might very well not be the case.

Death tolls in the battles were often not very large in Medieval battles, and even in famous slaughters like during Battle of Golden Spurs there's strong indication that many death could have happened due to the fact that Flemish were ordered to take no prisoners.


Man doesn't have to be badly wounded to be defeated, in many cases he may simply get mentally defeated, physically restrained (grappled, pinned down with forks, whatever) or get tired to the point where he cannot stand back.

So it's not safe to guess effects of even 'mighty' blows from battle results alone, without more precise descriptions.

So while I also would guess that with heavy pikes or clubs some blows might have been even deadly despite armor, we cannot really use the word 'must'.



be more rigid than mail and it will affect how effective a blow is at doing damage to something underneath.

Can't see how mail wouldn't 'rigid' enough to be honest. No matter what is the thickness of sword blade, effective 'connection' of impact will get transfered to the few rings, and will thus have the pressure depending a lot on wire thickness.

Straybow
2015-07-01, 02:31 AM
@Straybrow: I know all that but you seem to be making the mistake of forgetting what the alternatives are. There is no "alternative" to the slow transfer rates of comparatively low energy waste heat. It is waste heat, meaning not hot enough to use for anything except making people uncomfortable. You can't "pump up" the laser to effective levels if the transfer rate is too low.

Given my point about how huge the surface area to volume ratio inside the laser can get and given the advantage of conduction in addition to radiation as a transfer method you'd need a very inefficient medium indeed for it to not easily provide quite good cooling per cubic meter relative to your available volume and surface area ratio's. All the limits in the world don't stop it still being almost certainly an incredible improvement over and above the alternative. The sole limitation right now is a lack of known medium for such a purpose. You seem to be thinking that just because a volume is available inside the ship it can automatically be used to efficiently take up the waste heat and turn it into IR laser. The means of efficiently exchanging thermal energy into a volume involve filling that volume with piping or baffle plating for conduction transfer. That means the volume can't be filled with lasing medium, it's already full of metal.

Carl
2015-07-01, 07:15 AM
@Straybrow:


There is no "alternative" to the slow transfer rates of comparatively low energy waste heat. It is waste heat, meaning not hot enough to use for anything except making people uncomfortable. You can't "pump up" the laser to effective levels if the transfer rate is too low.

You seem to have completely missed the point of the attached paragraph...


ou seem to be thinking that just because a volume is available inside the ship it can automatically be used to efficiently take up the waste heat and turn it into IR laser. The means of efficiently exchanging thermal energy into a volume involve filling that volume with piping or baffle plating for conduction transfer. That means the volume can't be filled with lasing medium, it's already full of metal.

i allready pointed out i mathed out a really basic setup with a lasing medium volume of 0.5m^3 and roughly 200m^2 surface area. It would require some seriously fine grained coolant channels, but it's not a bad example of how far you can push it.

Charlaton
2015-07-01, 03:54 PM
Thanks for all the responses and suggestions from everyone. I'll read through the material suggested to me (except Knight and the Blast Furnace...can't drop $450 on a fancy :/) and get come back if I have any further questions. Except this one...those of you in HEMA, how familiar are you with other branches? Are there any you'd suggest to someone in the St. Louis, MO area?

Straybow
2015-07-01, 06:59 PM
@Straybrow:

You seem to have completely missed the point of the attached paragraph...

i allready pointed out i mathed out a really basic setup with a lasing medium volume of 0.5m^3 and roughly 200m^2 surface area. It would require some seriously fine grained coolant channels, but it's not a bad example of how far you can push it. Not really. You said that radiators are vulnerable, that volume-based heat dissipation would be superior to surface-based heat dissipation, and, "The sole limitation right now is a lack of known medium for such a purpose."

There is room for heat exchangers, and for a large laser. You haven't demonstrated that any configuration of heat exchangers can efficiently move waste heat into a lasing medium. Nor that any laser can operate at the extremely long wavelengths of IR photons emitted by waste heat.

Straybow
2015-07-01, 07:07 PM
Sure, but HEMA is a good example of how swords had very little blunt force trauma.
A feder doesn't come to a sharp edge, but it certainly comes to an edge of sorts. The flat on the edge is very thin. Yes, it will have less focused force than a proper sword, but only just. Either way, the physics are clear. You only have a couple ounces behind the part on the blade that hits the enemy. If you don't cut or slice, you are unlikely to do much damage at all. You don't have enough weight or momentum to cause significant blunt trauma.

Also, the padding worn is thin and in most places entirely non-rigid. Some people actually wear gear modeled after gambesons. If feder vs. gambeson=light bruising, sharp longsword vs. gambeson+maille is not going to be very likely to inflict any long term damage.
Feders are not good examples to use. Not only are they blunted, but they are also much lighter and less stiff than a "real" sword. Nobody does HEMA tournament fights with full weight reproductions. A few crazy Brits fight with reproductions while wearing full plate.

No, the physics of the sword are that the whole weight of the sword, and the force of muscle behind it, hits the enemy.

Carl
2015-07-01, 08:01 PM
Not really. You said that radiators are vulnerable, that volume-based heat dissipation would be superior to surface-based heat dissipation, and, "The sole limitation right now is a lack of known medium for such a purpose."

I never said internal laser radiators where superior to external radiators. I said Laser radiators + external radiators will always be superior to just externals. That's blindingly obvious.



There is room for heat exchangers, and for a large laser. You haven't demonstrated that any configuration of heat exchangers can efficiently move waste heat into a lasing medium. Nor that any laser can operate at the extremely long wavelengths of IR photons emitted by waste heat.

Erm. What part of:

Larger surface area than external radiators, (by a massive amount for a small volume cost in percentage of available space terms), plus additional transfer rate via conduction is always going to give vastly higher transfer rates into a lasing medium than external hull area can achieve. ( 4 x 0.3m diameter lasing tubes per cubic meter with 10% volume usage on a 20m cube would give 25% more transfer area). Now whether the lasing medium can use all that. Genuine question.

As for the last sentence. That's the rub that i pointed out all the way back in post one on this issue. there's nothing in physics that says we can't do this AFAIK, but we haven't yet discovered something that can. Probably in part because ATM we have little good reason to look for it.

SowZ
2015-07-01, 09:53 PM
Feders are not good examples to use. Not only are they blunted, but they are also much lighter and less stiff than a "real" sword. Nobody does HEMA tournament fights with full weight reproductions. A few crazy Brits fight with reproductions while wearing full plate.

No, the physics of the sword are that the whole weight of the sword, and the force of muscle behind it, hits the enemy.

A medieval longsword was more than two pounds, but less than four. Anything within that range is possible. The tournament ready feders I know of mostly exceed three pounds. I don't know of any legal feders that are less than a pound and a half. They are not 'much lighter than a real sword.' They are exactly the same weight.

The whole weight of the sword doesn't go into the spot that you hit if your sword doesn't bite into the person's skin. Even then. The person moves because they aren't on the ground or up against a wall. The sword flexes and even wraps around the target a little. There is a reason that a four pound sword has less blunt force trauma than a one pound stick with a three pound ball on the end.

Also, we sometimes use blunts that are nothing more than blunted real swords. They don't weight more or hurt more than a feder.

Carl
2015-07-03, 12:40 AM
Question of my own for the unarmed experts out there. I've got a rough scenario in mind for a fight but i need an lower time limit that isn't completely ridiculous.

It's effectively a bar fight scenario but fairly large, (300 ish participants), and one side is fairly dosed up on Alcohol, not enough to seriously impair motor function, but enough to impair judgment, (they trecked across town on foot to start the fight, but they'd never have been so stupid as to do so without the edge of the alcohol for an idea of the level of intoxication), they also have roughly something like low level military training, their opponents are similar but sober, (trainee's actually, so use whatever term your branch of your countries military calls their trainee's), however one specific member of the trainee's due to prior training and a serious dose of natural talent is an absolute monster in unarmed combat and things turn ugly enough to make her go all in. She's on the kind of level you only ever get a handful of people to, the sort of person who makes black belt's, (to use the term colloquially), blanche at the though of facing, and she's explicitly trained to maim to disable and even kill with nothing but her bare hands. The fight gets nasty enough she feels the need to go all in on, and she explicitly goes after those who are representing a potentially lethal threat to her fellow trainee's, meaning she puts them down anyway she can, without hurting them if possible, without killing them ideally, but whatever it takes in the end.

The numbers are slightly biased in favour of the non-trainee's. Her count for the fight is 47 disabled, (19 with some form of permanent paralysis), and 17 dead. What's your estimation of how long the fight would have to go on for to see those kinds of numbers?

SowZ
2015-07-03, 01:02 AM
Question of my own for the unarmed experts out there. I've got a rough scenario in mind for a fight but i need an lower time limit that isn't completely ridiculous.

It's effectively a bar fight scenario but fairly large, (300 ish participants), and one side is fairly dosed up on Alcohol, not enough to seriously impair motor function, but enough to impair judgment, (they trecked across town on foot to start the fight, but they'd never have been so stupid as to do so without the edge of the alcohol for an idea of the level of intoxication), they also have roughly something like low level military training, their opponents are similar but sober, (trainee's actually, so use whatever term your branch of your countries military calls their trainee's), however one specific member of the trainee's due to prior training and a serious dose of natural talent is an absolute monster in unarmed combat and things turn ugly enough to make her go all in. She's on the kind of level you only ever get a handful of people to, the sort of person who makes black belt's, (to use the term colloquially), blanche at the though of facing, and she's explicitly trained to maim to disable and even kill with nothing but her bare hands. The fight gets nasty enough she feels the need to go all in on, and she explicitly goes after those who are representing a potentially lethal threat to her fellow trainee's, meaning she puts them down anyway she can, without hurting them if possible, without killing them ideally, but whatever it takes in the end.

The numbers are slightly biased in favour of the non-trainee's. Her count for the fight is 47 disabled, (19 with some form of permanent paralysis), and 17 dead. What's your estimation of how long the fight would have to go on for to see those kinds of numbers?

Even if she averages thirty seconds per target, which is pretty absurd, that's over an hour. For one person to get that kind of a count is going to require a huge battle. 100 some odd people on each side doesn't come close to cutting it, I'm sorry to say. This person personally takes out fairly close to half the opposition. It isn't that I am saying she isn't good enough. It is that she wouldn't have time to get to that many. I'd either lower her tally or up the number of combatants.

VoxRationis
2015-07-03, 01:20 AM
This isn't quite a weapons, armor, or tactics question, but I had to either post it here or post something on "world-building" and hope someone knowledgeable replies before the sun expands and swallows the earth.
I have read in many sources that the galleys of Classical Greece (and to a lesser extent, those of the rest of the ancient world) differed from the galleys of the Middle Ages and later periods by having oarsmen with superior individual skill and a higher oarsmen:oars ratio to take advantage of that. As empires expanded and found that their demand for galley crews exceeded the supply of highly-skilled rowers, they designed and built galleys with multiple men per oar. I'm currently building a setting inspired by the High Middle Ages, but am developing several cultures out of whole cloth. So my question to you is this: would older-style, stacked-rower galleys seem too anachronistic, or too implausible, in a High Medieval setting, even if we assume that the demand for rowers does not exceed the supply of highly-skilled professionals?

Brother Oni
2015-07-03, 02:09 AM
Even if she averages thirty seconds per target, which is pretty absurd, that's over an hour. For one person to get that kind of a count is going to require a huge battle. 100 some odd people on each side doesn't come close to cutting it, I'm sorry to say. This person personally takes out fairly close to half the opposition. It isn't that I am saying she isn't good enough. It is that she wouldn't have time to get to that many. I'd either lower her tally or up the number of combatants.

Or alternately, lower the number of combatants on her side, giving them more numbers to fight through. Alcohol is a double edged weapon though, depending on how tanked up they are - they have impaired judgement and co-ordination, but their pain response is reduced and their endurance is increased.

I'm not sure Carl appreciates how chaotic such a massive brawl is though and it's highly likely that your super soldier is going to get blindsided more than once - as an example, the church scene in the film Kingsmen (not linked due to board rules).
Bar fights do tend to involved improvised weapons and when weapons are involved, individual engagements can get down to a couple of seconds each.


So my question to you is this: would older-style, stacked-rower galleys seem too anachronistic, or too implausible, in a High Medieval setting, even if we assume that the demand for rowers does not exceed the supply of highly-skilled professionals?

Galleys persisted in some fashion well into the Age of Sail, but after the Battle of Lepanto (https://en.wikipedia.org/wiki/Battle_of_Lepanto) in the 16th Century, their main combat roles were supplanted by ships with sails. The Venetians used them extensively, if I remember correctly.

Some checking of the later designs indicate that they weren't of the stacked style though, although there are people here who are far more knowledgable about it (just don't ask when the ships had cannon).

SowZ
2015-07-03, 02:14 AM
Or alternately, lower the number of combatants on her side, giving them more numbers to fight through. Alcohol is a double edged weapon though, depending on how tanked up they are - they have impaired judgement and co-ordination, but their pain response is reduced and their endurance is increased.

I'm not sure Carl appreciates how chaotic such a massive brawl is though and it's highly likely that your super soldier is going to get blindsided more than once - as an example, the church scene in the film Kingsmen (not linked due to board rules).
Bar fights do tend to involved improvised weapons and when weapons are involved, individual engagements can get down to a couple of seconds each.



Galleys persisted in some fashion well into the Age of Sail, but after the Battle of Lepanto (https://en.wikipedia.org/wiki/Battle_of_Lepanto) in the 16th Century, their main combat roles were supplanted by ships with sails. The Venetians used them extensively, if I remember correctly.

Some checking of the later designs indicate that they weren't of the stacked style though, although there are people here who are far more knowledgable about it (just don't ask when the ships had cannon).

Weapons just make such a huge difference, though. When adrenaline is high, you can slam someone in the face repeatedly, slam them into walls, etc. etc. and there is no telling how long it will take to beat them dead or unconscious. It might be in the first few hits. It might take several minutes. Even a super skilled martial artist cannot reliably take someone out in just a few seconds.

Even if she is outnumbered, the fight isn't going to last an hour. Unless her side loses, she won't have the chance to take out so many.

If she has literal superhuman skill though, that is fine. It seems as though realism matters at least a little to the poster, though.

VoxRationis
2015-07-03, 02:16 AM
Galleys persisted in some fashion well into the Age of Sail, but after the Battle of Lepanto (https://en.wikipedia.org/wiki/Battle_of_Lepanto) in the 16th Century, their main combat roles were supplanted by ships with sails. The Venetians used them extensively, if I remember correctly.

Some checking of the later designs indicate that they weren't of the stacked style though, although there are people here who are far more knowledgable about it (just don't ask when the ships had cannon).

Yes, I am aware of this. Perhaps I did not phrase my question clearly. The one-deck, multi-man-oar designs in the medieval and Renaissance eras are what I was contrasting with, say, an Athenian trireme or a (much later) Liburnian bireme. But part of why that contrast exists is because of societal and strategic considerations, rather than those of technology or performance. I am wondering whether the technological considerations that do make the difference would make stacked designs implausible in a High Medieval time period, assuming the societal and strategic considerations are not relevant factors.

Kiero
2015-07-03, 03:26 AM
I have read in many sources that the galleys of Classical Greece (and to a lesser extent, those of the rest of the ancient world) differed from the galleys of the Middle Ages and later periods by having oarsmen with superior individual skill and a higher oarsmen:oars ratio to take advantage of that. As empires expanded and found that their demand for galley crews exceeded the supply of highly-skilled rowers, they designed and built galleys with multiple men per oar. I'm currently building a setting inspired by the High Middle Ages, but am developing several cultures out of whole cloth. So my question to you is this: would older-style, stacked-rower galleys seem too anachronistic, or too implausible, in a High Medieval setting, even if we assume that the demand for rowers does not exceed the supply of highly-skilled professionals?

The Hellenistic Age (only a couple of centuries later than the Classical Age) had a higher demand for oarsmen than the availability of the best professionals. They fixed that by oaring arrangements with more men per oar - only the guy at the inner-most end actually needs to know what he's doing, the others on the oar are just there for muscle-power. The pentere/quinquireme was the standard naval vessel by this period, it required two men per oar on the top two banks.

Then you could hire less-skilled men to fill out the numbers or levy the poorer citizens as Athens did. Use of slaves was pretty limited, for all that Hollywood liked to make out crews would often be.

Also bear in mind, stacked banks of oars with many men per oar is the only way to increase power. There's a logistical limit of 3 oars per bank, ships never got more than that, if anyone tells you differently, chances are they're either reading old research, or don't have a clue what they're talking about. Also the practical limit of men-per-oar is 8; ships like "twenties" cheated by having catamaran hull arrangements (so it was actually a pair of "tens" built together). This means you can't really get more than 24 rowers per bank.

If they're Mediterranean-style low-sided war galleys, you also need to emphasise the importance of taking them out of the water. I was just talking about all of this elsewhere, so I'll just quote myself:


Some of the earliest examples of galleys (not long after the ancient Egyptians) were designed for a group of oarsman-marines to be able to get to another ship fast, empty the benches, picking up weapons and shields as they went, then board. The dreaded Sea People who are written about in the time of the so-called Bronze Age collapse were often drawn travelling in pentekonter-style galleys, one bank of 25 oars on each side, totalling 50-odd men who would both row and fight.

It was only with the later, bigger galleys that there tended to be a specialisation of labour with lots of oarsmen and only a handful of marines. What this did was make ships much faster and more enduring under oars. Contrary to Hollywood myth, most oarsmen were not slaves (where do you keep them at night?), but professionals or mercenaries. In the sole case of Athens, they were citizens performing military service to their city.

Worth noting that generally an oared ship, particularly over a sprinting distance, is faster and more maneuverable than a sailing ship. Especially before the invention of the lateen sail.

The aforementioned Aetolians and Arcanarnians were notorious pirates, but any vessel in the water might turn it's hand to a little opportunistic piracy if presented with the chance. The Phoenicians, acknowledged as master sailors, weren't averse to doing so either.

It's worth remembering that captives taken during pirate raids was one of the major sources of slaves for the slave markets in antiquity. Moreso than defeated warriors taken in warfare or cities having all their inhabitants sold to pay debts, until the late Hellenistic/early Roman era.

In a fight, the mast and sail would be taken down, to avoid it being damaged, and reduce the risk of fire. Firepots being one of the weapons that might be deployed in a ship fight.

As has been said, until late into the Hellenistic era, and even then, artillery wasn't an important fighting tactic. Ramming and boarding, with the added tactic (mastered by the Rhodians) of shearing. Shearing is where instead of aiming your ram at the enemy ship, you aim at his oars. If he doesn't get them in fast enough (which is a standard procedure when an enemy closes), he risks having oarsmen killed and maimed by broken oars lashing about inside the rowing deck(s).

Something else that isn't shared by the Age of Sail: Mediterranean galleys were not totally waterproof, and took on water to become watertight. However, the longer they were in the water, the more waterlogged their boards got, making them heavier, thus slower and less maneuverable. To counter this, at night they were pulled up out of the water to dry out if possible. They weren't left moored in the water at docks. Because of their shallow draft and light construction, any beach would generally do. Rocky shores would actually be of no use whatsoever, since they'd inhibit your ability to get the ship out of the water.

Ships didn't stay at sea for days on end, or usually cross the open sea either. Most coasted, staying not far from land. Not just because many cultures didn't have the navigation skills to chart a course by the stars (though the Phoenicians did), but also because you couldn't carry enough water and food for several hundred oarsmen for more than a day or two.

One more thing, which could seem trivial, but will sort out those who know what they're talking about from those who don't. Oared galleys never had more than three banks of oars. Meaning if you look at one side of the ship, and count the oar-ports vertically, there are never more than three of them.

The polyreme naming convention (trireme, quadrireme, quinquireme, etc) does not refer to the number of oars in a file, but the number of oarsmen. You add power and endurance (and reduce the number of skilled oarsmen required) by adding more men per oar. Thus a five isn't five banks of one oar each, but three banks with the top two comprising two oarsmen each, and one on the bottom bank.

This is how silly-seeming numbers like deceres ("tens") are possible, lots of men per oar. Only the man on the end actually needs to know what he's doing and be able to keep to the oarmaster's time. Talking of whom, the oarmaster kept time with a drum or a stick they'd hit the deck with.

Brother Oni
2015-07-03, 03:53 AM
Weapons just make such a huge difference, though. When adrenaline is high, you can slam someone in the face repeatedly, slam them into walls, etc. etc. and there is no telling how long it will take to beat them dead or unconscious. It might be in the first few hits. It might take several minutes. Even a super skilled martial artist cannot reliably take someone out in just a few seconds.

Even if she is outnumbered, the fight isn't going to last an hour. Unless her side loses, she won't have the chance to take out so many.

Oh, I fully agree on the reliability of unarmed combat, just commenting on what bar fights typically turn into.

If I had to place an estimate on fight length, no more than 10-15 minutes (although subjectively, it's going to feel like an hour), after which one side will either have won, or they've fought themselves into a stalemate, both sides warily looking at each other across the room as they regroup and catch their breath (adrenaline will keep you going at peak performance for a while into the anerobic phase of exertion, but sooner or later, the oxygen debt is going to catch up).


Speaking of which, assuming a civilised setting, what's the local law enforcement's response time? That would probably put an upper limit on the fight time since people are likely to flee once folks with proper weapons and armour turn up.

fusilier
2015-07-03, 03:58 AM
This isn't quite a weapons, armor, or tactics question, but I had to either post it here or post something on "world-building" and hope someone knowledgeable replies before the sun expands and swallows the earth.
I have read in many sources that the galleys of Classical Greece (and to a lesser extent, those of the rest of the ancient world) differed from the galleys of the Middle Ages and later periods by having oarsmen with superior individual skill and a higher oarsmen:oars ratio to take advantage of that. As empires expanded and found that their demand for galley crews exceeded the supply of highly-skilled rowers, they designed and built galleys with multiple men per oar. I'm currently building a setting inspired by the High Middle Ages, but am developing several cultures out of whole cloth. So my question to you is this: would older-style, stacked-rower galleys seem too anachronistic, or too implausible, in a High Medieval setting, even if we assume that the demand for rowers does not exceed the supply of highly-skilled professionals?

The standard method of rowing a galley in the Middle Ages was known as the alla sensile method. This method developed over time but I think was established by the late 14th century. In it a typical galley had three men per bench (all on the same deck), but each man had his own oar. It was an efficient method, but required some skill as the outer men on the bench had their motions constrained by the inner men. Also it was difficult to put more men on a bench, as the oars started to interfere with each other too much.

During the 16th century this method was gradually abandoned in favor of the alla scalaccio method where the men on the bench all pulled on the same oar. This was less efficient but much easier to learn, and galley slaves could be pressed into service without having to rely upon paid professionals. The other benefit was that it could be scaled up fairly easily, by adding more men to a bench (although this had other repercussions on galley design). A galley rowed "4x4" in the alla scalaccio method was on par in terms of general performance with one rowed "3x3" alla sensile.

The Venetians clung to the sensile method for longer, and most of their galleys were still rowed in this fashion at the Battle of Lepanto in 1571.

So, it was the case, that most of the galleys in the Middle Ages and Early Renaissance were rowed with one man per oar (at least in the Mediterranean)-- just not stacked on different levels. However, there are other differences between Ancient and Medieval Galleys. While Ancient/Classical galleys were usually built shell first, Medieval Mediterranean galleys were being built frame first -- this made them stronger (and may be a factor that discouraged the use of the underwater ram), but also heavier.

Also, the physical rowing method was different. If I remember, in Ancient galleys they rowed while in a seated position. Medieval and Renaissance galleys they stood up when the oars were dropped into the water, then they pushed with their legs against a footboard, pulling with the arms and back until they basically fell onto the bench at the end of the stroke. I think the analysis was that the later method developed more force, and was better for shorter bursts of speed (in combat), while the old method was superior for rowing for longer periods of time.

Whether or not using a galley with stacked rowers is too anachronistic would be up to you. But historically, you don't *need* to stack the rowers to get them all using their own oars.

Carl
2015-07-03, 06:07 AM
To deal with the responses in no specific order, the authority response time is the reason i'm asking for a fight length estimate there's a fair amount of room to fudge it but i need to work numbers out. There also is a certain amount of outnumbering going on, that's why i said 300 ish+ total with maybe 200 on the opposite side. Blindsiding ehr is rather hard, she's psionic, she can't read minds at a distance, but she can in close proximity sense the presence of other beings so you might get behind her, but she's going to know your there and moving towards her.

I am surprised however at the 30 plus second per person estimate. I mean sure i don't expect her to rip through an opponent in say 3 seconds but i figured it would be quicker than that given she's perfectly willing to snap or dislocate all someone's limb bones if she has to to put them down, crush spine's and windpipes, snap neck's, (my understanding is you have to use various induced controlled fall techniques), and a person's nasal bones, (the most famous though i believe the real world preferred method is to break the nose first), or whatever other methods that i haven't thought of but might work. Maybe i'm overestimating the amount of force someone could apply, i'd always assumed that lesser variations of the fancy show techniques you often see martial artists doing could be applied to damaging effect to someone's anatomy provided you had the right kind of hold, or the right kind of angle, or such like, (you can assume she knows very accurately exactly how much force to apply at a given angle on a given joint/bone for human and near human targets).

Though to be fair several of the deaths aren't strictly from unarmed combat, their psionic touch attacks, but she doesn't have a non-lethal form of that and really doesn't like using it to kill unless she has no other choice.

Brother Oni
2015-07-03, 07:44 AM
To deal with the responses in no specific order, the authority response time is the reason i'm asking for a fight length estimate there's a fair amount of room to fudge it but i need to work numbers out.

Well in 2010, the London Met police averaged 10 minutes for an emergency response (link (http://www.met.police.uk/foi/pdfs/disclosure_2013/october_2013/2013010001787.pdf)). Something of this scale would take a little longer, but probably not too much longer - looking at the 7th July bombings, the first police officer was on the scene in 6 minutes and the main response arrived ~25 minutes after that.

That said, if it's a modern day setting, a crowd of drunk people that large moving as a single mass across town would have probably tripped police surveillance already and the riot police would be on hand much quicker (within minutes probably). More details would be appreciated if you'd like a more relevant response.


Blindsiding ehr is rather hard, she's psionic, she can't read minds at a distance, but she can in close proximity sense the presence of other beings so you might get behind her, but she's going to know your there and moving towards her.

If her abilities are such that she can distinguish individual intent when there's ~300 extrememly aggressive minds in her immediate area, then you're probably looking at sub 5-10 seconds per individual engagement as she'd ambush other people already in fights and stay out of any one-on-ones herself.

In a mass brawl, often a person doesn't see their next target until seconds after they've finished their current one (or just react without conscious thought). Again, I think you're underestimating how chaotic a mass brawl like this can be, especially if you're right in the middle of it (take a look at riots as an example).



Maybe i'm overestimating the amount of force someone could apply, i'd always assumed that lesser variations of the fancy show techniques you often see martial artists doing could be applied to damaging effect to someone's anatomy provided you had the right kind of hold, or the right kind of angle, or such like, (you can assume she knows very accurately exactly how much force to apply at a given angle on a given joint/bone for human and near human targets).

The fancy martial arts techniques you see and read about often only work from ambush or otherwise optimal conditions (one on one, space to move, etc). In reality, more often than not, you end up fluffing the technique (maybe they stumble at the last second and turn an otherwise incapacitating hit into a less severe one, or they're faster/stronger than she expected and that attemped arm lock goes completely pearshaped) at which point, you end up in the basics/grappling part of fights, where you end up trying anything and everything to get the upper hand.

When in this situation and unarmed, pure physical strength and weight counts for a lot - making the assumption she is an otherwise fit and healthy female soldier and her psionics don't grant her inhuman strength and reflexes (with regard to the latter, I mean that although she can know what they're going to do, her body won't react any faster than a normal human's), she's going to be at a disadvantage against bigger and stronger men due to simple biology (maybe this is where she causes her fatalities?).

If I wanted to emphasise her abilities, I'd have her use weapons (chair leg for example) and then go through incapacitating enemies already in combat, staying out of the mass groups and picking off isolated fights. Again, may I suggest the Kingsman film for an idea of how this might proceed (note that the scene has maybe 100 people at most and your scenario is proposing 300).

Edit: Did a better headcount of the scene in question

VoxRationis
2015-07-03, 12:03 PM
The standard method of rowing a galley in the Middle Ages was known as the alla sensile method. This method developed over time but I think was established by the late 14th century. In it a typical galley had three men per bench (all on the same deck), but each man had his own oar. It was an efficient method, but required some skill as the outer men on the bench had their motions constrained by the inner men. Also it was difficult to put more men on a bench, as the oars started to interfere with each other too much.

During the 16th century this method was gradually abandoned in favor of the alla scalaccio method where the men on the bench all pulled on the same oar. This was less efficient but much easier to learn, and galley slaves could be pressed into service without having to rely upon paid professionals. The other benefit was that it could be scaled up fairly easily, by adding more men to a bench (although this had other repercussions on galley design). A galley rowed "4x4" in the alla scalaccio method was on par in terms of general performance with one rowed "3x3" alla sensile.

The Venetians clung to the sensile method for longer, and most of their galleys were still rowed in this fashion at the Battle of Lepanto in 1571.

So, it was the case, that most of the galleys in the Middle Ages and Early Renaissance were rowed with one man per oar (at least in the Mediterranean)-- just not stacked on different levels. However, there are other differences between Ancient and Medieval Galleys. While Ancient/Classical galleys were usually built shell first, Medieval Mediterranean galleys were being built frame first -- this made them stronger (and may be a factor that discouraged the use of the underwater ram), but also heavier.

Also, the physical rowing method was different. If I remember, in Ancient galleys they rowed while in a seated position. Medieval and Renaissance galleys they stood up when the oars were dropped into the water, then they pushed with their legs against a footboard, pulling with the arms and back until they basically fell onto the bench at the end of the stroke. I think the analysis was that the later method developed more force, and was better for shorter bursts of speed (in combat), while the old method was superior for rowing for longer periods of time.

Whether or not using a galley with stacked rowers is too anachronistic would be up to you. But historically, you don't *need* to stack the rowers to get them all using their own oars.

Ah. That's what I'm looking for. I forgot about alla sensile rowing (probably because it kind of rubs me the wrong way, on account of the different oar lengths and clustered oarlocks). Now, I heard (read, really—this isn't gossip on the street) that the frame-first method was developed because people had to re-learn a lot of shipbuilding skills after the collapse of the Roman Empire, and learned it in a different way from the ancients. Given that the major naval area of my campaign setting never had such a collapse, do you think that continuity would make the difference between staying with the old designs and adopting the medieval styles?

That's interesting about the stroke technique, though I've heard the old triremes were far faster than even the fleetest of the medieval galleys.

Brother Oni
2015-07-03, 01:32 PM
Ah. That's what I'm looking for. I forgot about alla sensile rowing (probably because it kind of rubs me the wrong way, on account of the different oar lengths and clustered oarlocks).

The wiki page has this nice model of a Venetian alla sensile galley if it helps you visualise it:


https://upload.wikimedia.org/wikipedia/commons/2/29/Venice_galley_rowing_alla_sensile1.jpg

fusilier
2015-07-03, 02:23 PM
Ah. That's what I'm looking for. I forgot about alla sensile rowing (probably because it kind of rubs me the wrong way, on account of the different oar lengths and clustered oarlocks). Now, I heard (read, really—this isn't gossip on the street) that the frame-first method was developed because people had to re-learn a lot of shipbuilding skills after the collapse of the Roman Empire, and learned it in a different way from the ancients. Given that the major naval area of my campaign setting never had such a collapse, do you think that continuity would make the difference between staying with the old designs and adopting the medieval styles?

That's interesting about the stroke technique, though I've heard the old triremes were far faster than even the fleetest of the medieval galleys.

I thought that the Byzantines kept the classical construction methods going for longer, but even they eventually changed. I've heard various theories for the change in construction style. As for speed you might be right -- but there's a lot of factors. Classical ships may have been more lightly constructed, which would have helped with speed -- also 3x3 rowing developed later in the middle ages, replacing 2x2 rowing. Finally the size of the ship mattered -- smaller galleys were usually faster. I think, although could be wrong, that in terms of burst speed a Classical galley and a Renaissance one were roughly the same.

Did some more research -- most Medieval galleys were being rowed with two oars per bank on different decks throughout the 11th century, by about 1200 they were putting them on the same bench, all on one exposed deck. By 1300 three men on a bench were common.

Note: when talking about number of men per bench, etc., I'm usually referring to a "standard" galley. Smaller galleys had fewer men per bench, larger ones usually had the same but not necessarily, and there were other variations.

Kiero
2015-07-03, 02:47 PM
That's interesting about the stroke technique, though I've heard the old triremes were far faster than even the fleetest of the medieval galleys.

Probably because they were significantly lighter and much sleeker in profile. Ancient galleys were made to be able to be pulled out of the water by their oarsmen at the end of every day.

Look up the reconstruction of an ancient trieres, the Olympias (https://en.wikipedia.org/wiki/Olympias_%28trireme%29), it's long, low and narrow.


She was subject to sea trials in 1987, 1990, 1992 and 1994, but one of the most informative was an exercise in 1987 when crewed by 170 volunteer oarsmen and oarswomen. Olympias achieved a speed of 9 knots (17 km/h) and was able to execute 180 degree turns within one minute, in an arc no wider than two and a half (2.5) ship-lengths. These results, achieved with an inexperienced crew, suggest that the ancient writers were not exaggerating about the capabilities of such vessels

Archpaladin Zousha
2015-07-03, 03:54 PM
Feels like my question kinda got lost in the shuffle of the current conversation. Is it it okay for me to reiterate it here?


I was interested in creating a character similar to the Cohors Sagittariorvm Levantinorvm (Imperial Eastern Archer Auxilia) unit from Rome: Total War's famous Europa Barborum mod:

http://europabarbarorum.com/EB1/i/units/romani/roman_archer_auxilia.gif

Specialised archer units for the imperial army can be enrolled in all provinces in which skilled bowyers and a population familiar with the traditions of archery can be found. These are mainly in Asia and the fringes of the steppes. Archers now have become an integral part of any well composed roman army. Normally they are placed behind a wall of protective heavy infantry in the battle formation. From there they can provide a supporting barrage fire above the infantry's heads to weaken the enemy and break his charge before the main lines engage. The imperial army's archers are more heavily armoured than their counterparts, with shirts of lorica squamata or hamata (scale or chain mail), conical iron or bronze helmets and small shields protecting their left arm. They use long ranged recurved composite bows with bone ends and bracers to protect their forearms from the sinew, together with multiple types of arrows: three bladed heads to inflict heavy wounds at un armoured targets, thin needle like, pyramidal shaped armour piercing heads and flaming arrows, carrying an ignition load in a kind of small metal cage incorporated into the arrowhead. Additionally the archers are armed with a gladius for self defence, but despite this and their armour it should not be expected that they fight well at close quarters for any longer time.
It's clear from the description that these are composite bows, but I'm not certain if they're longbows or shortbows as per D&D and Pathfinder gear conventions. Generally longbows seem to be associated with wood and more popular in Northern Europe, as indicated by the Celtic and Germanic factions longbow units. By contrast, shortbows were more associated with the horse archers of the east, like the Parthians and Huns.

So, do I go short or long for this foot archer?

Brother Oni
2015-07-03, 04:13 PM
Feels like my question kinda got lost in the shuffle of the current conversation. Is it it okay for me to reiterate it here?

Looking at the SRD20, it doesn't distinguish between a straight bow and a recurve bow, yet differentiates between a self bow and a composite bow (despite the bow shape having a greater effect on performance).

From the image you've posted, that's a 'short bow' since it doesn't appear to be the height of the archer, so if you want visual accuracy, a composite shortbow would be the weapon. For performance accuracy, a composite longbow would be better.

Perhaps talk to your GM and have a fluff composite recurve represented mechanically as a composite longbow?

Mr. Mask
2015-07-03, 07:37 PM
Speaking of combat, I have a question as to counters. Can anyone describe what setting up and executing a really good counter is about? I'm trying to design a combat mechanic around them, and currently it's a little like a straight from poker (you see a chance to line up a straight, so you decide to put your chips down).




Carl: It does partially depend on how bloody she wants to get, and how much the others want to stick around for it. I remember that when the Marines got into bar fights, they intentionally avoided using any deadly move, going more with kickboxing kinds of hits. Moves designed to kill can be distinguished, and you can get charged if others witness you trying to murder people in a bar fight. Under normal conditions, several people ending up dead or paralysed would result in someone getting shot.

Though, it sounds like this case is not normal, as it seems the other side are willing to stick around even when things get bloody and are trying to murder the trainees to the extent it gets the protagonist(?) to react very violently. Another question that comes up under those conditions, however, is why the other group didn't bring weapons if they came over to murder the trainees.

Her Abilities
Anyway, onto the kill count. If the others only have basic training, and she's one of the most dangerous people in the world, she can take each man out within a couple of seconds. Smash one's windpipe, another's collarbone, another one's sternum, break someone's neck, gouge an eye, crush testicles, displace their kneecaps with a kick, she'll be very good at setting up these highly lethal, disabling strikes. If she wants to disable people without permanent or lethal injury, it gets a lot harder, as it starts to lower her moveset down to kickboxing. In a kickboxing situation, the larger, stronger, faster person tends to come out ahead, and it would become a lot more about her endurance and ability to ambush and blindside people, as she's likely to get hit a lot and need to hit a lot more.

Semi-Lethal
There are various means of injuries that are questionably lethal and permanent. Like, smashing a person's windpipe, some people have survived that and recovered fully, others have died. It partly depends on how nearby medical aid is. Good medical aid can do amazing things, such that it's generally rare for people to die near to a hospital, unless you really make an effort to kill them. Back to semi lethal strikes, she can theoretically disable a lot of kneecaps, smash windpipes, kick people in the groin, knock them unconscious by choking the arteries in the neck (stop blood flow to the brain), thumb an eye so the person gives up and goes home, or even just take a blade and cut someone open and let the medics sort it out. If she's doing this and holding back, this is dangerous both to the person she's attacking, and to herself--as there is a chance someone will shrug off one of these injuries and hurt her.

Improvised Weapons
As someone mentioned, barfights tend to have improvised weapons. A good improvised club will make it much easier for her to disable people without killing them. If she gets a really nice club, then she could break bones reliably in such a way that's (relatively speaking) unlikely to kill, with medical aid nearby. That being said, improvised weapons don't make for very nice clubs, generally. Too light, badly balanced, too heavy, etc.. She could try something like go about cutting people, as bleeding is scary to many people and it may make them back out of the fight (cut someone's windpipe a little, and all they'll think about is how weird breathing feels--though you'll most likely cut an artery and kill them).

Tactics
I mentioned before about blindsiding people and ambushing them. If there area similar number of combatants, she can move around and look for people to catch offguard. This is what brings a combatant from formidable to terrifyingly dangerous, their tactics. Even with a bar stool, if she is able to smash someone's foot, to lead her enemies inward so her trainee friends jump on them from two sides before she moves forward again and helps them mop up, even if she manages to distract a number of the enemy so her side has a chance to turn the tide, or do something like take a hostage, she could make a big difference. Her personal kill-count might be harder to track this way, as she'll be doing some kill-assists, kill-stealing, and helping her allies get kills ("kills" being enemies down).

Psionic Touch Attacks and Unarmed
One thing to note, is that unarmed combat does reduce her edge against less skilled, but physically capable opponents. So weapons are handy. As for a psionic death attack, she could kill everyone in the room quite easily, depending on its limitations. I'm not quite sure about those limitations, but if she is able to kill someone with it, and is willing to use lethal force, it does imply she'd be using it quite a bit, unless there are bad side-effects (even if there are, she's likely to use it whenever she gets in trouble, or to just avoid getting into trouble at all).


I'll give this some more thought.

SowZ
2015-07-03, 07:43 PM
Speaking of combat, I have a question as to counters. Can anyone describe what setting up and executing a really good counter is about? I'm trying to design a combat mechanic around them, and currently it's a little like a straight from poker (you see a chance to line up a straight, so you decide to put your chips down).




Carl: It does partially depend on how bloody she wants to get, and how much the others want to stick around for it. I remember that when the Marines got into bar fights, they intentionally avoided using any deadly move, going more with kickboxing kinds of hits. Moves designed to kill can be distinguished, and you can get charged if others witness you trying to murder people in a bar fight. Under normal conditions, several people ending up dead or paralysed would result in someone getting shot.

Though, it sounds like this case is not normal, as it seems the other side are willing to stick around even when things get bloody and are trying to murder the trainees to the extent it gets the protagonist(?) to react very violently. Another question that comes up under those conditions, however, is why the other group didn't bring weapons if they came over to murder the trainees.

Her Abilities
Anyway, onto the kill count. If the others only have basic training, and she's one of the most dangerous people in the world, she can take each man out within a couple of seconds. Smash one's windpipe, another's collarbone, another one's sternum, break someone's neck, gouge an eye, crush testicles, displace their kneecaps with a kick, she'll be very good at setting up these highly lethal, disabling strikes. If she wants to disable people without permanent or lethal injury, it gets a lot harder, as it starts to lower her moveset down to kickboxing. In a kickboxing situation, the larger, stronger, faster person tends to come out ahead, and it would become a lot more about her endurance and ability to ambush and blindside people, as she's likely to get hit a lot and need to hit a lot more.

Semi-Lethal
There are various means of injuries that are questionably lethal and permanent. Like, smashing a person's windpipe, some people have survived that and recovered fully, others have died. It partly depends on how nearby medical aid is. Good medical aid can do amazing things, such that it's generally rare for people to die near to a hospital, unless you really make an effort to kill them. Back to semi lethal strikes, she can theoretically disable a lot of kneecaps, smash windpipes, kick people in the groin, knock them unconscious by choking the arteries in the neck (stop blood flow to the brain), thumb an eye so the person gives up and goes home, or even just take a blade and cut someone open and let the medics sort it out. If she's doing this and holding back, this is dangerous both to the person she's attacking, and to herself--as there is a chance someone will shrug off one of these injuries and hurt her.

Improvised Weapons
As someone mentioned, barfights tend to have improvised weapons. A good improvised club will make it much easier for her to disable people without killing them. If she gets a really nice club, then she could break bones reliably in such a way that's (relatively speaking) unlikely to kill, with medical aid nearby. That being said, improvised weapons don't make for very nice clubs, generally. Too light, badly balanced, too heavy, etc.. She could try something like go about cutting people, as bleeding is scary to many people and it may make them back out of the fight (cut someone's windpipe a little, and all they'll think about is how weird breathing feels--though you'll most likely cut an artery and kill them).

Tactics
I mentioned before about blindsiding people and ambushing them. If there area similar number of combatants, she can move around and look for people to catch offguard. This is what brings a combatant from formidable to terrifyingly dangerous, their tactics. Even with a bar stool, if she is able to smash someone's foot, to lead her enemies inward so her trainee friends jump on them from two sides before she moves forward again and helps them mop up, even if she manages to distract a number of the enemy so her side has a chance to turn the tide, or do something like take a hostage, she could make a big difference. Her personal kill-count might be harder to track this way, as she'll be doing some kill-assists, kill-stealing, and helping her allies get kills ("kills" being enemies down).

Psionic Touch Attacks and Unarmed
One thing to note, is that unarmed combat does reduce her edge against less skilled, but physically capable opponents. So weapons are handy. As for a psionic death attack, she could kill everyone in the room quite easily, depending on its limitations. I'm not quite sure about those limitations, but if she is able to kill someone with it, and is willing to use lethal force, it does imply she'd be using it quite a bit, unless there are bad side-effects (even if there are, she's likely to use it whenever she gets in trouble, or to just avoid getting into trouble at all).


I'll give this some more thought.

I'm just pointing out that hand to hand combat isn't nearly that reliable. Adrenaline, human willpower, and a hundred other factors make it not nearly so simple. I'm sorry, I don't care how well trained you are, you cannot reliably disable an enemy in a few seconds with your hands. You can do it sometimes. But one guy might go unconscious after one strike and another might keep fighting even after taking dozens of strikes. This is how a real fight works. Even with knives, sometimes one stab kills somebody. Sometimes it takes literally dozens. Even with a gun. Sometimes shooting someone once drops them. Other times, you put half a dozen slugs in somebody and they are still fighting. This whole, "with enough skill, you can dance from guy to guy nerve pinching and throat breaking consistently" is pure Hollywood. Fights do not work that way, especially in a chaotic melee. You might be able to do a fancy disabling strike on someone who doesn't see it coming that will drop them more often than not, but human beings will surprise you.

With adrenaline, even an untrained foe can be dangerous. Even in swordfighting, historical master swordsman talk about the dangers of fighting untrained stupid people fighting with abandon. People get vicious. They are all human beings designed by evolution to survive. And skill doesn't matter more in a massive melee, it matters less since luck became more important. There is a reason the best martial masters in history tended to prefer duels to battles; they realized that with enough skill they could go undefeated in a hundred duels. The same could not be said for big battles.

Now, if it is magic, okay, that is something else. Just realize this isn't how real humans fight. There are real people who can kick through baseball bats and break femurs with a single punch. But even they would know that real fights are not so clean and choreographed. It is chaotic as hell, and things happen so fast but also things can seem to take forever.

There are a bunch of stories where heroes have supernatural capabilities and that is fine. Just realize it isn't realistic. Which is perfectly fine.

Hjolnai
2015-07-03, 09:16 PM
Even if someone could reliably defeat opponents hand to hand in just a few seconds, fighting burns energy very quickly. I'd find it difficult to believe that anyone could continue to do so again and again and again, without rest in between; after the first twenty or so (which is still an unbelievable number), fatigue would sap too much strength and focus to be able to so precisely defeat people quickly, and it only gets worse once that efficiency is gone. If applicable, a height and weight disadvantage will also make this even harder.

Quite frankly, in the absence of particular weaponry (guns, explosives, incendiaries etc), I couldn't imagine a scenario where one person can take down 47 people no matter the skill difference. Unarmed, or with improvised weapons, really doesn't seem possible.

SowZ
2015-07-03, 09:41 PM
Even if someone could reliably defeat opponents hand to hand in just a few seconds, fighting burns energy very quickly. I'd find it difficult to believe that anyone could continue to do so again and again and again, without rest in between; after the first twenty or so (which is still an unbelievable number), fatigue would sap too much strength and focus to be able to so precisely defeat people quickly, and it only gets worse once that efficiency is gone. If applicable, a height and weight disadvantage will also make this even harder.

Quite frankly, in the absence of particular weaponry (guns, explosives, incendiaries etc), I couldn't imagine a scenario where one person can take down 47 people no matter the skill difference. Unarmed, or with improvised weapons, really doesn't seem possible.

I could believe a particularly skilled mounted knight taking down fifty men in a very large battle using melee weaponry. Crazier things have happened and been documented in warfare. However, that is still a far cry from improvised weaponry and hand to hand combat. And to your point; the knight would be mounted and there could be little rests between engagements. Yeah, I also don't buy that a human being would have that much endurance.

Carl
2015-07-03, 10:07 PM
Damn it your giving me even more to reply to :(. I am writing a big reply to Oni's stuff that should cover most of it but i may have to add expansion paragraphs. But yes Mask, very non-standard situation, it starts as a bar fight but finishes as something a lot uglier the collective dead and seriously injured count not including her total is probably as high or higher than her own as a minimum. So no the Authorities may not like that they've got a bunch of dead and seriously injured but they're not going to go charging anyone for defending themselves anyway they can.

Roxxy
2015-07-03, 10:19 PM
How much of a problem are large height disparities within a modern military force? Like, let's say we have a modern technology fantasy setting where a nation has large numbers of humans, dwarves, elves, and orcs. In this country, the average human ranges between 4'9" and 6'4" (it is an immigrant country with a wide range of different ethnicities who have different average heights), the average dwarf ranges between 4'5" and 5'3", the average elf ranges between 5'3" and 5'9", and the average orc ranges between 7'3" and 7'9". If all of these people are allowed to enlist, how much does the fact that height varies between 4'5" and 7'9" effect the logistics of the military and the use of military vehicles?

Mr. Mask
2015-07-03, 11:53 PM
SowZ:
I'm going to disagree, and my disagreement may sound sharper than I intend. I don't blame you for being under these impressions. They're sadly common. Again, I apologize if I sound sharper than I mean to.

Well, I'm assuming she's a very dangerous person, as that is the description given. When people get to the level where they can kill someone then go to eating, they're past the point of needing to build up willpower or psyche themselves up, like normal people need to. Adrenaline also takes some experience to get under wraps, but it isn't too hard--you need to improve control of that just to aim and shoot effectively in a fight (hands shaking would be a serious problem). Many of the other factors you allude to are also concerns in fighting with weapons. The concerns of unarmed combat are less killing power and the greater usefulness of physical strength.


"I'm sorry, I don't care how well trained you are, you cannot reliably disable an enemy in a few seconds with your hands."
I've known people who could reliably take me down in seconds. It's really not that unusual, even in sports circles. Try to fight a black belt judo expert, and you're on the ground in a second, with a throw that could've broken your neck. Tell a CQB combat instructor for the USMC he can't take someone out in a second, and you might be in the hospital as an example (though, funnily enough, the reverse happened to an army instructor). And believe me, those cute judo throws don't just hurt, they can kill someone. And the gouging eyes, knee cap trick, breaking the windpipe, sternum or collar bone? I was not kidding, you can really can do that stuff, and it disables people, and you don't do it by hitting them for five minutes till their HP reaches 0.


"You can do it sometimes. But one guy might go unconscious after one strike and another might keep fighting even after taking dozens of strikes."
You're thinking of gloved boxing. Bare-knuckle was known to last seconds till someone was out, and they had ten minutes for you to get back up. Judo matches with professionals will last a few seconds. And smashing someone's confused head on the pavement lasts as long as you want it to.


"This is how a real fight works."
You may have some experience with brawling and the like. What you describe is true for those instances, that is real fighting and it tends to work that way. The cases and people I've known didn't have much experience with brawling, they were combat survivors. People who killed to survive. And what I described is also how real fighting works.


"This is how a real fight works. Even with knives, sometimes one stab kills somebody. Sometimes it takes literally dozens."
This is where skill comes in. If you're good with a knife, you don't just stab in and out. You grab them, stab in, and cut your way out. Opening up the throat and abdomen, so even if they're not mentally incapacitated, they're literally too busy holding their organs in (there's a guy who survived one instance of that. There was no twenty stabs, only a couple of cuts). A lot of people have no idea how to use knives; they jump in, try for a quick stab, and jump out--and look down in surprise that their opponent stabbed them too while they were jumping around (I'm thinking of a particular case, couple of highschool idiots, drove off to the hospital shortly after a few stabs).


"Even with a gun. Sometimes shooting someone once drops them. Other times, you put half a dozen slugs in somebody and they are still fighting."
The FBI did some interesting statistics, on that. Three shots will incapacitate a person 90% of the time, but some people who are hyped up on drugs or even adrenaline will remain functioning after dozen of shots. The brain can keep going for several seconds without blood-flow. With battle rifles, you don''t see that problem so much unless you fail to hit centre of mass, as the exit cavity gets to be enormous (liquefy their insides). Assault rifles are pretty good, but the military was pretty unhappy to lose that extreme firepower, almost universally.

This is actually a mistaken correlation. Are there cases of people taking absurd amounts of damage and not going down? Yes. Does this mean you can't take people down reliably? 90% of the time, you can take them down with three shots centre of mass (that's why the M16 uses a 3 round burst). There are also some tricks you can use, to incresae your chances. One cop who was around in a bad part of town when there was lots of PCP going about, he learned to shoot the pelvis--not for a groin shot (pain really doesn't matter if you're on enough drugs), but to shatter the pelvis. You can't fight if you can't stand, and losing the ability to walk is very shocking.


"This whole, "with enough skill, you can dance from guy to guy nerve pinching and throat breaking consistently" is pure Hollywood. Fights do not work that way, especially in a chaotic melee. You might be able to do a fancy disabling strike on someone who doesn't see it coming that will drop them more often than not, but human beings will surprise you. "
Only nerve pinching trick I know of that works is a hit to a certain part of the arm, which stuns the nerves. I think they might use that in something, but I've never been interested in it. Grabbing a guy by the shirt, unbalancing him so he can't fight back, then hammering his collarbone or kneeing his sternum are better, or just a good old judo trip to send their head slamming into the concrete. You are correct that movie portrayals of this are hollywood.

"With adrenaline, even an untrained foe can be dangerous."
You have it a bit backwards here. Adrenaline can be useful, but a lot of learning to fight is learning to keep it under wraps. It makes your hands shake, which is bad for swords and guns, and has some other effects you generally want to avoid. Another annoying one is tachypsychia. I heard one guy who said he found it useful, but another said it really messed him up, that everything was happening slower but his brain wasn't any faster, and that he kept telling his feet to move, but there was a delay--which made it confusing as heck. Getting those physical reflexes under control is important for soldiers of the past and present. People who are really practised at killing, it's really no different from any other activity. Like a practised skydiver, who doesn't feel any stress, even though they might die.


"Even in swordfighting, historical master swordsman talk about the dangers of fighting untrained stupid people fighting with abandon."
I'm having trouble thinking of an example of this. Musashi never said anything of that nature, Sun Tzu said if your opponent is stupid and easy to anger, just anger him and lure him into a trap. You might be thinking of men who have nothing to lose, like men in desperate straights? Someone backed up against a wall with no means of escape, committed to fighting you, that is a dangerous opponent to watch out for. That's why it's wise to allow them a means of escape, so their thoughts are divided. Then you cut them down.


"People get vicious."
Vicious has limited use. Some self defence teachers have taught women to scratch at eyes, bite, etc., in self defence. This succeeds in making your opponent angry, so that an assault or robbery may turn into a homicide when they get their gun or knife. If you learn how to properly gouge an eye though, then sure, get as vicious as you can to survive (though be careful of going too far, as that can turn self defence into murder).


"There is a reason the best martial masters in history tended to prefer duels to battles; they realized that with enough skill they could go undefeated in a hundred duels."
They did? Martial arts masters are more associated with duels, certainly, as that's where they show off their skills. But I have trouble thinking of a named preference for duels. Musashi commented that it was the same for duelling as it was for war, and he won sixty duels (not sure if anyone has managed one hundred). He also participated in a couple of battles, but there wasn't that much war in his time. There's Talhoffer, who learned what he did from judging judicial duels?


"And skill doesn't matter more in a massive melee, it matters less since luck became more important. There is a reason the best martial masters in history tended to prefer duels to battles; they realized that with enough skill they could go undefeated in a hundred duels. The same could not be said for big battles."
Melees and wars are less discriminate, certainly. A skilled fighter can be killed by an oddly placed artillery round just as a green recruit. But skill definitely does help, even in the chaos of war. Some guys survived to be old veterans, because knowing when to duck your head and how to shoot people before they shoot you is an important skill. And then you get people who drive off a few hundred Germans from the top of a burning tank, or who rush the German line and cause the enemy force to surrender, or rout the Japanese after killing one of them with his own sword and taking it for yourself as your stump-arm is still bleeding. You also get heroic actions from fellows like someone Barcelona. They do a good job of adapting his actions in The Pacific. Skill does pay off, even if you can get unlucky. And if you had an unskilled soldiers in those same situations, they wouldn't have succeeded.

The USMC close combat manual has some good information on unarmed techniques, and group tactics.




"Even if someone could reliably defeat opponents hand to hand in just a few seconds, fighting burns energy very quickly. I'd find it difficult to believe that anyone could continue to do so again and again and again, without rest in between; after the first twenty or so (which is still an unbelievable number), fatigue would sap too much strength and focus to be able to so precisely defeat people quickly, and it only gets worse once that efficiency is gone. If applicable, a height and weight disadvantage will also make this even harder.

Quite frankly, in the absence of particular weaponry (guns, explosives, incendiaries etc), I couldn't imagine a scenario where one person can take down 47 people no matter the skill difference. Unarmed, or with improvised weapons, really doesn't seem possible."
Oh, they did a test of this. Someone in full armour, trying to fight with a longsword for as long as he can. He lasted a minute or two. Put them in a life or death situation without the armour, and you'd get a sizably better amount of time. But in that case, he was fighting non stop against three guys taking turns. In an actual melee, there's a little break while you look for your next target, and you can always pull back for a while to regain your breath. Some battles lasted for a day or more, just by people taking turns resting and fighting. Height and weight disadvantages are serious problems in unarmed combat.

47 would be a lot. It might not be technically impossible, but it'd be a feat for the ages. I didn't comment much on that, but 47 is about as many as John Wick kills in one of the biggest shootouts, which seems too many for barehanded fighting.



"I could believe a particularly skilled mounted knight taking down fifty men in a very large battle using melee weaponry. Crazier things have happened and been documented in warfare. However, that is still a far cry from improvised weaponry and hand to hand combat. And to your point; the knight would be mounted and there could be little rests between engagements. Yeah, I also don't buy that a human being would have that much endurance."
Depends what you count. Knights sometimes chased down routing troops, so that would make it easy to rack up a lot of kills. In the modern day, there are some impressive examples of people taking out about that many men, even stopping whole battalions almost single-handed.



Carl: Damn it your giving me even more to reply to :(. I am writing a big reply to Oni's stuff that should cover most of it but i may have to add expansion paragraphs. But yes Mask, very non-standard situation, it starts as a bar fight but finishes as something a lot uglier the collective dead and seriously injured count not including her total is probably as high or higher than her own as a minimum. So no the Authorities may not like that they've got a bunch of dead and seriously injured but they're not going to go charging anyone for defending themselves anyway they can.

OK. With that, I'd say things were starting out pretty normal, lots of ineffective punching, main character probably stays out of it--then someone does something stupid and pulls out a knife or a gun, or breaks a bottle and tries to stab someone. This causes more people to bring out weapons, but instead of it bringing the fight to a standstill, more stupid stuff happens (not stupid plot, just stupid stuff people can do) and suddenly everyone is trying to murder each other in a drunken rage (you'd need them to really hate each other, beforehand).



Roxxy:
I wouldn't say too much. You'd have a lot of different sizes of equipment sent in, and if you didn't have that many halfling recruits or something, they might be relegated to something more peaceful or otherwise neglected to make things easier on logistics. Vehicles would probably be annoyingly tight for the orcs, but the smaller creatures would do just fine (they might have some modular seating, so they can transport twice the dwarves if need be). In a war, you could get a lot of cases of orcs complaining that there isn't any armour or other gear in their size, because only human/elf sized stuff was delivered (depending on the enlistment numbers, women didn't have properly fitting body armour for quite some time, so they might just get neglected).

SowZ
2015-07-04, 12:32 AM
SowZ:
I'm going to disagree, and my disagreement may sound sharper than I intend. I don't blame you for being under these impressions. They're sadly common. Again, I apologize if I sound sharper than I mean to.

Well, I'm assuming she's a very dangerous person, as that is the description given. When people get to the level where they can kill someone then go to eating, they're past the point of needing to build up willpower or psyche themselves up, like normal people need to. Adrenaline also takes some experience to get under wraps, but it isn't too hard--you need to improve control of that just to aim and shoot effectively in a fight (hands shaking would be a serious problem). Many of the other factors you allude to are also concerns in fighting with weapons. The concerns of unarmed combat are less killing power and the greater usefulness of physical strength.


"I'm sorry, I don't care how well trained you are, you cannot reliably disable an enemy in a few seconds with your hands."
I've known people who could reliably take me down in seconds. It's really not that unusual, even in sports circles. Try to fight a black belt judo expert, and you're on the ground in a second, with a throw that could've broken your neck. Tell a CQB combat instructor for the USMC he can't take someone out in a second, and you might be in the hospital as an example (though, funnily enough, the reverse happened to an army instructor). And believe me, those cute judo throws don't just hurt, they can kill someone. And the gouging eyes, knee cap trick, breaking the windpipe, sternum or collar bone? I was not kidding, you can really can do that stuff, and it disables people, and you don't do it by hitting them for five minutes till their HP reaches 0.


"You can do it sometimes. But one guy might go unconscious after one strike and another might keep fighting even after taking dozens of strikes."
You're thinking of gloved boxing. Bare-knuckle was known to last seconds till someone was out, and they had ten minutes for you to get back up. Judo matches with professionals will last a few seconds. And smashing someone's confused head on the pavement lasts as long as you want it to.


"This is how a real fight works."
You may have some experience with brawling and the like. What you describe is true for those instances, that is real fighting and it tends to work that way. The cases and people I've known didn't have much experience with brawling, they were combat survivors. People who killed to survive. And what I described is also how real fighting works.


"This is how a real fight works. Even with knives, sometimes one stab kills somebody. Sometimes it takes literally dozens."
This is where skill comes in. If you're good with a knife, you don't just stab in and out. You grab them, stab in, and cut your way out. Opening up the throat and abdomen, so even if they're not mentally incapacitated, they're literally too busy holding their organs in (there's a guy who survived one instance of that. There was no twenty stabs, only a couple of cuts). A lot of people have no idea how to use knives; they jump in, try for a quick stab, and jump out--and look down in surprise that their opponent stabbed them too while they were jumping around (I'm thinking of a particular case, couple of highschool idiots, drove off to the hospital shortly after a few stabs).


"Even with a gun. Sometimes shooting someone once drops them. Other times, you put half a dozen slugs in somebody and they are still fighting."
The FBI did some interesting statistics, on that. Three shots will incapacitate a person 90% of the time, but some people who are hyped up on drugs or even adrenaline will remain functioning after dozen of shots. The brain can keep going for several seconds without blood-flow. With battle rifles, you don''t see that problem so much unless you fail to hit centre of mass, as the exit cavity gets to be enormous (liquefy their insides). Assault rifles are pretty good, but the military was pretty unhappy to lose that extreme firepower, almost universally.

This is actually a mistaken correlation. Are there cases of people taking absurd amounts of damage and not going down? Yes. Does this mean you can't take people down reliably? 90% of the time, you can take them down with three shots centre of mass (that's why the M16 uses a 3 round burst). There are also some tricks you can use, to incresae your chances. One cop who was around in a bad part of town when there was lots of PCP going about, he learned to shoot the pelvis--not for a groin shot (pain really doesn't matter if you're on enough drugs), but to shatter the pelvis. You can't fight if you can't stand, and losing the ability to walk is very shocking.


"This whole, "with enough skill, you can dance from guy to guy nerve pinching and throat breaking consistently" is pure Hollywood. Fights do not work that way, especially in a chaotic melee. You might be able to do a fancy disabling strike on someone who doesn't see it coming that will drop them more often than not, but human beings will surprise you. "
Only nerve pinching trick I know of that works is a hit to a certain part of the arm, which stuns the nerves. I think they might use that in something, but I've never been interested in it. Grabbing a guy by the shirt, unbalancing him so he can't fight back, then hammering his collarbone or kneeing his sternum are better, or just a good old judo trip to send their head slamming into the concrete. You are correct that movie portrayals of this are hollywood.

"With adrenaline, even an untrained foe can be dangerous."
You have it a bit backwards here. Adrenaline can be useful, but a lot of learning to fight is learning to keep it under wraps. It makes your hands shake, which is bad for swords and guns, and has some other effects you generally want to avoid. Another annoying one is tachypsychia. I heard one guy who said he found it useful, but another said it really messed him up, that everything was happening slower but his brain wasn't any faster, and that he kept telling his feet to move, but there was a delay--which made it confusing as heck. Getting those physical reflexes under control is important for soldiers of the past and present. People who are really practised at killing, it's really no different from any other activity. Like a practised skydiver, who doesn't feel any stress, even though they might die.


"Even in swordfighting, historical master swordsman talk about the dangers of fighting untrained stupid people fighting with abandon."
I'm having trouble thinking of an example of this. Musashi never said anything of that nature, Sun Tzu said if your opponent is stupid and easy to anger, just anger him and lure him into a trap. You might be thinking of men who have nothing to lose, like men in desperate straights? Someone backed up against a wall with no means of escape, committed to fighting you, that is a dangerous opponent to watch out for. That's why it's wise to allow them a means of escape, so their thoughts are divided. Then you cut them down.


"People get vicious."
Vicious has limited use. Some self defence teachers have taught women to scratch at eyes, bite, etc., in self defence. This succeeds in making your opponent angry, so that an assault or robbery may turn into a homicide when they get their gun or knife. If you learn how to properly gouge an eye though, then sure, get as vicious as you can to survive (though be careful of going too far, as that can turn self defence into murder).


"There is a reason the best martial masters in history tended to prefer duels to battles; they realized that with enough skill they could go undefeated in a hundred duels."
They did? Martial arts masters are more associated with duels, certainly, as that's where they show off their skills. But I have trouble thinking of a named preference for duels. Musashi commented that it was the same for duelling as it was for war, and he won sixty duels (not sure if anyone has managed one hundred). He also participated in a couple of battles, but there wasn't that much war in his time. There's Talhoffer, who learned what he did from judging judicial duels?


"And skill doesn't matter more in a massive melee, it matters less since luck became more important. There is a reason the best martial masters in history tended to prefer duels to battles; they realized that with enough skill they could go undefeated in a hundred duels. The same could not be said for big battles."
Melees and wars are less discriminate, certainly. A skilled fighter can be killed by an oddly placed artillery round just as a green recruit. But skill definitely does help, even in the chaos of war. Some guys survived to be old veterans, because knowing when to duck your head and how to shoot people before they shoot you is an important skill. And then you get people who drive off a few hundred Germans from the top of a burning tank, or who rush the German line and cause the enemy force to surrender, or rout the Japanese after killing one of them with his own sword and taking it for yourself as your stump-arm is still bleeding. You also get heroic actions from fellows like someone Barcelona. They do a good job of adapting his actions in The Pacific. Skill does pay off, even if you can get unlucky. And if you had an unskilled soldiers in those same situations, they wouldn't have succeeded.

The USMC close combat manual has some good information on unarmed techniques, and group tactics.




"Even if someone could reliably defeat opponents hand to hand in just a few seconds, fighting burns energy very quickly. I'd find it difficult to believe that anyone could continue to do so again and again and again, without rest in between; after the first twenty or so (which is still an unbelievable number), fatigue would sap too much strength and focus to be able to so precisely defeat people quickly, and it only gets worse once that efficiency is gone. If applicable, a height and weight disadvantage will also make this even harder.

Quite frankly, in the absence of particular weaponry (guns, explosives, incendiaries etc), I couldn't imagine a scenario where one person can take down 47 people no matter the skill difference. Unarmed, or with improvised weapons, really doesn't seem possible."
Oh, they did a test of this. Someone in full armour, trying to fight with a longsword for as long as he can. He lasted a minute or two. Put them in a life or death situation without the armour, and you'd get a sizably better amount of time. But in that case, he was fighting non stop against three guys taking turns. In an actual melee, there's a little break while you look for your next target, and you can always pull back for a while to regain your breath. Some battles lasted for a day or more, just by people taking turns resting and fighting. Height and weight disadvantages are serious problems in unarmed combat.

47 would be a lot. It might not be technically impossible, but it'd be a feat for the ages. I didn't comment much on that, but 47 is about as many as John Wick kills in one of the biggest shootouts, which seems too many for barehanded fighting.



"I could believe a particularly skilled mounted knight taking down fifty men in a very large battle using melee weaponry. Crazier things have happened and been documented in warfare. However, that is still a far cry from improvised weaponry and hand to hand combat. And to your point; the knight would be mounted and there could be little rests between engagements. Yeah, I also don't buy that a human being would have that much endurance."
Depends what you count. Knights sometimes chased down routing troops, so that would make it easy to rack up a lot of kills. In the modern day, there are some impressive examples of people taking out about that many men, even stopping whole battalions almost single-handed.



Carl: Damn it your giving me even more to reply to :(. I am writing a big reply to Oni's stuff that should cover most of it but i may have to add expansion paragraphs. But yes Mask, very non-standard situation, it starts as a bar fight but finishes as something a lot uglier the collective dead and seriously injured count not including her total is probably as high or higher than her own as a minimum. So no the Authorities may not like that they've got a bunch of dead and seriously injured but they're not going to go charging anyone for defending themselves anyway they can.

OK. With that, I'd say things were starting out pretty normal, lots of ineffective punching, main character probably stays out of it--then someone does something stupid and pulls out a knife or a gun, or breaks a bottle and tries to stab someone. This causes more people to bring out weapons, but instead of it bringing the fight to a standstill, more stupid stuff happens (not stupid plot, just stupid stuff people can do) and suddenly everyone is trying to murder each other in a drunken rage (you'd need them to really hate each other, beforehand).



Roxxy:
I wouldn't say too much. You'd have a lot of different sizes of equipment sent in, and if you didn't have that many halfling recruits or something, they might be relegated to something more peaceful or otherwise neglected to make things easier on logistics. Vehicles would probably be annoyingly tight for the orcs, but the smaller creatures would do just fine (they might have some modular seating, so they can transport twice the dwarves if need be). In a war, you could get a lot of cases of orcs complaining that there isn't any armour or other gear in their size, because only human/elf sized stuff was delivered (depending on the enlistment numbers, women didn't have properly fitting body armour for quite some time, so they might just get neglected).

I'll try and respond to the rest later, but I'll comment on the swordfighting for now. Keep in mind that Mushashi lost multiple battles, but never a duel, because you have limited control in a duel. As for the references to the danger of an aggressive, stupid opponent, I don't know about the Japanese or Chinese Masters. But the Germans had a term for it. The Buffalo. The problem with an untrained but angry opponent is that it is likely they will never stop attacking. If you are good, you can wait for an opening and get a kill. The problem with that is that if your opponent doesn't have self-preservation, you are likely to both hit each other. It's difficult to protect yourself against someone who isn't trying to do the same.

Sometimes you make a swing forcing the enemy to block, expecting them to save themselves, but if they decide to just stab you instead you are both screwed. So there were different tactics to fighting such a person to ensure you didn't both gank each other.

Janus
2015-07-04, 01:28 AM
Just want to confirm something:
Shields are absolutely redundant, if not cumbersome, for someone wearing full plate, right? I keep seeing it in games and other media, and I just want to make sure that I'm not crazy (at least on this topic).

....on a similar note, would it be dumb of me to up the AC of D&D 5e's plate armor while making shields useless when you're wearing it?

Mr. Mask
2015-07-04, 02:11 AM
Janus: Well, not entirely, but generally yes. There were still some knights and lances around at that time, and they used shields to increase their chance of survival in the clash. Part because anything you can do to increase your survival in those mayhem charges is good, and because being on a horse the weight of a shield is less cumbersome, and also because it's really awkward to use two handed weapons from atop a horse so the main disadvantage of a shield is gone. Which brings us to the other point, that polearms and two handed weapons were a big deal by the time full plate had taken root, and you can't use those with a shield.

So, a plated fighter could still use a shield (and I bet some did in some instances in Europe at the time), but normally there are better options since your armour is good enough. So, making shield redundant while in full plate wouldn't be necessary.



SowZ
Huh? I don't think Musashi commanded any battles. It is true that you have more control over your faculties than a random body of men, so unless you get their discipline just right and they're of a similar skill to yourself, your chances may well be better in a personal duel than in a battle. At the same time, you could have the skills of a body of men make up for the insufficiencies of a bad commander. Either way, I'm not sure what you're saying, as I didn't say otherwise to this. If you mean that this disproves what Musashi says about war and duels being the same, then you need to read the rest of his writings.

"As for the references to the danger of an aggressive, stupid opponent, I don't know about the Japanese or Chinese Masters. But the Germans had a term for it. The Buffalo. The problem with an untrained but angry opponent is that it is likely they will never stop attacking. If you are good, you can wait for an opening and get a kill."
Which German masters said that? If you go crazy against a good practitioner today, swinging wildly, he'll figuratively cut you to pieces, displacing your guard and stabbing you without allowing you to injure himself. Defending and then attacking rather than going for a more master-stroke approach isn't anything I've seen a reputable master recommend. You don't generally wait for openings when the enemy comes to blade range, repeatedly guarding like in the movies till he lifts his blade sky-high as in dark souls, but make your own opening based off minor mistakes every human makes. Displace their guard, unbalance them, run them through or throw them. Step back, step forward with a cut as their strike misses if they're really unorthodox. If someone was a real duellist, someone without any genuine fighting experience, they might be so flabbergasted by someone not fighting in proper rhythm and technique that it throws them off the way a southpaw can, but that's the best I can think of for why someone's stupidity might be an effective weapon (well, except in convincing them to fight you in the first place, or swarm you like dying lemmings).


The problem with that is that if your opponent doesn't have self-preservation, you are likely to both hit each other. It's difficult to protect yourself against someone who isn't trying to do the same.
This was a common problem with duels. I think it was George Silver who said that the Italians prided themselves in their skill based on how many men died in duels, and not by how many survived them? Some duelling schools taught really horrible ideas, and some of that resulted in drawn out fighting where both men ended up skewering each other (like the aforementioned highschoolers with pocket blades). It is more difficult to protect yourself against a really committed attacker, and them zerg rushing you might help them in the short run... but not by that much. Look up some longsword techniques, there are a lot of ways to displace your opponent's weapon while killing them this way and that.

Actually, someone reminded me before that George Silver said something about that. A good test for any duelling school is to have one of their masters fight three resolute men half-drunk. But the advantage isn't so much their idiocy or drunkenness, it is in their resolution and lack of fear of death. Like in the Hagakure, a man must accept to die so that he may live and succeed. But that isn't anything to do with poor defence, or advantages of ignoring defence, but commitment to your task (you may theoretically get some chance to kill your opponent instead of defending, but not statistically often enough to matter).

Sometimes you make a swing forcing the enemy to block, expecting them to save themselves, but if they decide to just stab you instead you are both screwed. So there were different tactics to fighting such a person to ensure you didn't both gank each other.
Well, that should be common technique regardless for who you're facing. If it's not, the school is teaching posturing, not fighting. Poking at belligerent beggars with long rapiers to scare them off, not dealing with committed assassins.

Spiryt
2015-07-04, 02:19 AM
Just want to confirm something:
Shields are absolutely redundant, if not cumbersome, for someone wearing full plate, right? I keep seeing it in games and other media, and I just want to make sure that I'm not crazy (at least on this topic).

....on a similar note, would it be dumb of me to up the AC of D&D 5e's plate armor while making shields useless when you're wearing it?

I don't think anyone can say that, since we still see people in more or less 'full plate' carrying some shields here and there.

Mainly in all kinds of visual art, but I think you can find some army registers describing them too.

Brother Oni
2015-07-04, 05:40 AM
Even if someone could reliably defeat opponents hand to hand in just a few seconds, fighting burns energy very quickly. I'd find it difficult to believe that anyone could continue to do so again and again and again, without rest in between; after the first twenty or so (which is still an unbelievable number), fatigue would sap too much strength and focus to be able to so precisely defeat people quickly, and it only gets worse once that efficiency is gone. If applicable, a height and weight disadvantage will also make this even harder.

Depends on a person's anaerobic endurance and how quickly they're building up oxygen debt. Given that professional MMA fighters have 5 minute rounds, you can probably argue that to be close to the upper limit before fatigue starts to become too debilitating.


How much of a problem are large height disparities within a modern military force? Like, let's say we have a modern technology fantasy setting where a nation has large numbers of humans, dwarves, elves, and orcs. In this country, the average human ranges between 4'9" and 6'4" (it is an immigrant country with a wide range of different ethnicities who have different average heights), the average dwarf ranges between 4'5" and 5'3", the average elf ranges between 5'3" and 5'9", and the average orc ranges between 7'3" and 7'9". If all of these people are allowed to enlist, how much does the fact that height varies between 4'5" and 7'9" effect the logistics of the military and the use of military vehicles?

Physical size has a significant effect on crew sizes - if you're too big, you can't function in the limited space you have. In some WW2 tanks, you couldn't even get in/out of the crew compartments if you were too big.

Things like logistics also become an issue- part of the height requirements is that the military don't want to be custom making uniforms, plus various weapon systems are designed for people of a minimum height (AT weapons for example).
That said, you could very well have specific racial regiments, each with their own requirements. For example, the British Army (http://www.army.mod.uk/documents/general/AFCO_Form5-U.pdf) has a minimum of 4'10" (depending on the role) whereas the Brigade of Gurkhas need to be at least 5'2".


Sometimes you make a swing forcing the enemy to block, expecting them to save themselves, but if they decide to just stab you instead you are both screwed. So there were different tactics to fighting such a person to ensure you didn't both gank each other.

To reinforce this, masters hated fighting amateurs as they were unpredictable. This often led to the simultaneous killing of both parties in duels.

Kiero
2015-07-04, 10:22 AM
Depends on a person's anaerobic endurance and how quickly they're building up oxygen debt. Given that professional MMA fighters have 5 minute rounds, you can probably argue that to be close to the upper limit before fatigue starts to become too debilitating.

And even then, they aren't fighting non-stop for 5 minutes, that's too tiring even for the fittest of people.

If anyone isn't aware just how tiring combat is, I'd suggest you pop along to a martial arts class that does actual contact sparring. Without armour or carrying weapons, you can still burn through an incredible amount of energy and shed a lot of sweat in 60 seconds of focused, intense effort.

Brother Oni
2015-07-04, 05:23 PM
If anyone isn't aware just how tiring combat is, I'd suggest you pop along to a martial arts class that does actual contact sparring. Without armour or carrying weapons, you can still burn through an incredible amount of energy and shed a lot of sweat in 60 seconds of focused, intense effort.

Indeed. It's why extended fights tend to have lulls - it's an opportunity for both sides to catch their breath.

Straybow
2015-07-05, 03:46 PM
Just want to confirm something:
Shields are absolutely redundant, if not cumbersome, for someone wearing full plate, right? I keep seeing it in games and other media, and I just want to make sure that I'm not crazy (at least on this topic).

....on a similar note, would it be dumb of me to up the AC of D&D 5e's plate armor while making shields useless when you're wearing it? Shields are defensive weapons, not just static hunks of wood used for mobile cover. Even in plate you use a shield or buckler to control your opponent's weapon to create an opening for attack if you aren't using a two-handed weapon. A shield is very useful to block heavier missiles that won't necessarily penetrate armor, but a strike to the helm can leave you disoriented and vulnerable for a few seconds. That may be all your opponent needs to put you on the ground and disarm you.

Kiero
2015-07-05, 04:54 PM
Shields are defensive weapons, not just static hunks of wood used for mobile cover. Even in plate you use a shield or buckler to control your opponent's weapon to create an opening for attack if you aren't using a two-handed weapon. A shield is very useful to block heavier missiles that won't necessarily penetrate armor, but a strike to the helm can leave you disoriented and vulnerable for a few seconds. That may be all your opponent needs to put you on the ground and disarm you.

Yep, people tend to forget that a shield has an edge. Even a larger one can do nasty things like break the instep of someone's foot or some toes if you slam it down on it. Jaws are always a good target for the edge of a shield, too, if you're in close.

Incanur
2015-07-05, 05:15 PM
As others have mentioned, soldiers wearing considerable plate armor did at times use shields. Fourquevaux's 1548 manual wanted pikers in three-quarters harness plus mail hose and a shield on their backs to sling down in the press once the pike became useless. Some of Maurice of Nassau's early 17th-century targeteers wore considerable plate armor in manual illustrations. Etc.

Spamotron
2015-07-05, 05:41 PM
Fourquevaux's 1548 manual wanted pikers in three-quarters harness

That three-quarters harness part is also noteworthy. Leg armor appears to be by far the most common piece of kit infantry ditch before a long march, and if your legs are unarmored a shield is excellent protection for them.

NRSASD
2015-07-05, 06:45 PM
So I've got an extended question for you guys. It's starts a little fuzzy but comes down to real-world physics I promise.

In a campaign I'm running, I've got a group of blood-mage/swordsmen (inspired by ciphers from Pillars of Eternity), who have extensive training in lightly armored sword combat. They use long slashing swords designed to inflict bleeding injuries, no shields, and lighter-than-steel chain shirts. The swords have a specially-designed fuller(?), running almost the entire length of the blade and designed to catch blood shed from bleeding wounds. The idea is that they step in, melee, disengage, and use the blood they drew to power their close combat magic. That being said, here's my questions:

1. Does the fuller being deeper or longer affect the odds of catching spilled blood?

2. Does the fuller being deeper or longer affect the integrity of the sword?

3. Should this sword still do as much damage as the typical D&D longsword? I know that the D&D longsword did not exist in the real world (unless I'm wrong, please correct me) and was an amalgam of several actual designs, but it's a useful reference point.

4. Are there any good examples of real world, straight, light, slashing swords with crossguard hilts? In my campaign, these guys are at the cutting edge of sword development, which has been mostly stuck somewhere between bastard and broad swords. More complex handguards have not yet developed.

Thanks for your help!

Mr. Mask
2015-07-05, 09:35 PM
Generally, people try to not get blood on them or down their blades in combat, as it makes them really slippery and is generally grotesque (not to say they succeed, it's difficult to avoid blood if you're getting good hits on unarmoured opponents [this makes me wonder how much people bleed through armour]). If you're trying to maximize the amount of blood contact, I'd either go with a big clever or a short knife. Keeping blood off you is especially hard when you're holding the guy and gutting him, etc.. A powerful cutting blade can just sever more arteries and make bloodied wounds (bigger weapons helps, too). You can also open a few more unnecessary cuts to draw more blood.

In general, they could try to get closer to their enemies (short, powerful cleavers, or dagger fighting), but I don't think the fuller of a sword will be the main cause of how bloody you get. If they only need as much blood as would drip down the blade, they could try running their hands down or even into the wounds of people they've injured or slain, or hug a corpse quickly. If there's pavement, then there could be a pool of blood they could have a quick roll in.

If they really only need a small amount, the fuller may not matter, as their swords should get bloody enough either way (they can lick their blades or something for greater effect, if it's ingested/vampire based).

Fullers are (sometimes) to reduce weight and increase the durability of a blade, I believe? I don't think it'd effect the damage of a sword if you had a different fuller (aside from making it a little lighter?).

I think the closest to a DnD longsword would be a one-handed sword, or maybe an "arming sword", which is a similar catchphrase for just about any sword (normally one handed).

"4. Are there any good examples of real world, straight, light, slashing swords with crossguard hilts? In my campaign, these guys are at the cutting edge of sword development, which has been mostly stuck somewhere between bastard and broad swords. More complex handguards have not yet developed."
A little unsure of which you mean. There are examples like this: http://fabri-armorum.com/english/foto-basic/1apr-mece.gif


Hope this helped. I'l have to get around to trying Pillars of Eternity (I guess it's as good as I've heard?).